Download as pdf or txt
Download as pdf or txt
You are on page 1of 93

2

FACULTY OF BUSINESS STUDIES


University of Dhaka
Executive MBA Admission Test, July 2023

Duration : 90 minutes Total Marks 80


MCQ : 55 minutes MCQ Marks 55
Written : 35 minutes Written Marks 25

Instructions
Check the question paper and the answer sheet before the start of the examination. You are advised
to change
the question paper or the answer sheet should the printing be illegible or missing. Any complaint
at a later
stage shall not be entertained.
Use of calculator or electronic device of any kind is not permitted.
Use black ball point pen only to fill up the circles.
°

Each question in the MCQ part carries one mark.

MCQ Part

1. ee English (Questions
1 - 20)
tn Se the blanks with appropriate words (Questions 1-5) =
1. Your supervisor is not the person to take your failure och mama Peden
fe kindly B. mistakenly C. timely D. deeply
2. When something tastes disgusting, it is
A. tasty B. awful C. spicy D. expensive
| 3. The selease of the film with the 35th birthday
of the actor.
A. coincided B. harmonized C. timed D. patronized
4 People have become under the burden of heavy taxes.
A. kindy B. isolatedy C. reckless D. impatient »
5. Hridoy was as he raced to a hundred in 60 balls.
A. unable B. unstoppable C. unplayable D. unpredictable
Fill in the blanks using appropriate prepositions (Questions
6 - 10)
6. They wanted to hear the end the story.
A. for B. to 2 of D. with
7. Business students are good analytical exercises.
K in g. at C. on D. to
i %. She had an accident, but is now recovering her injuries.
A. from B. at Cc. of D. with
9. Iwill take my driving test a week,
A. about B. in C. over D. on
10, Leave your shoes the door when you enter the house.
A. over “. C. by D. above
identify the correct sentences (Questions 11-13)
Iv Twenty minutes are allowed to each speaker.
B. Twenty minutes is allowed to each speaker,
C. Each speakers allowed twenty minutes.
D. Twenty minutes allowed each speaker,
*t seen Soma since six years.
12. A. Ihaven ars

B.
C.__
_Thaven't seen SO for six yeas.
Ihaven’t seen Soma semily sit years.
D. I haven’t seen Soma M

13, A. More you read, understand.


B. The more you rea the more you understand,
C. The more you read, the more YOU "rand.
D. More you read, the more you understa! .
14-15)
Fill in the blanks with the correct options provided (Questions
4. Y to come unless you want to.
~ An i don't need C. have need (D) need
The small child likes to do whatever her father.
¥ A. hasdone B. did C. was done B. does
16. Which word is wrongly spelt? Geni
A. Recreate B. Receive G. Reciept D. Recede
the pair FANS: GALLERIES
17. Choose the appropriate pair of words matching the relationship suggested in
A. Teams: Goalposts B. Tickets: Counters
C. Players: Referees RB. Audience: Seats
18. Which one is a synonym for ‘Exquisite’?
A. Plain B. Ugly C. Beautiful D. Painful
19. The word ‘Hospitality’ is related to
A. Clinics and patients B. Reception and entertai:
C. Trade and commerce D. Management and discipline
20. Which use of the apostrophe is correct?
A. This isa girl’s hostel.~_
B. The womens’ cricket team won the championship.X
C. The automobile shop sells cars’. K
RP. Our neighbours’ houses were silent.

Current Affairs (Questions 21 - 35)


21. Which is the first financial institution in the country with Mobile Financial Services?
zs A. BRAC Finance Ltd B. Nagad Finance PLC
C. bKash Finance PLC D.. Rocket Finance PLC
22. What did we celebrate this year for Joliot-Curie award to Bangabandhu Sheikh Mujibur Rahman?
A. 60* Anniversary B. 50% Anniversary C. 40" Anniversary D. 45" Anniversary
23. What was the proposed minimum amount of tax for an individual assesse having taxable income less than
TK.350,000?
A. 1,000 B. 1,500 € 2,000 D. 2,500
24. What amount has been approved by the International Monetary Fund (IMF) as loan to Bangladesh?
A. $4 Billion B. $4.5 Billion C.# $4.7 Billion D. $4.9 Billion
25. What is the mission of United Nations Conference on Trade and Development (UNCTAD)?
A. Peaceful economy for all B. Prosperity for all
C. Development is priority D. Results, Partnership and accountability
26. Who was the first Governor of Bangladesh Bank?
A. A.K.N. Ahmed B, M.Nurul Islam
Cc. AN, Hamidullah D. Khoshed Alam
27. 7 is the full form of SLR in banking?
- State Liquidity Ratio B. Statutory Liquidity Ratio
C. Social Liability Rate D. Statutory Loan Ratio
28. Aig ened caved Country is directly regulated by-
- Investment Corporation of Bangladesh B. Dhaka St ock Exchan
C. ae and Exchange Commission
Bangladesh Securities a: _D.. Ministry of Finance ige
29. Who is the architect of Jatiya Sangsad?
“ A. Moinul Hossain B. Hamidur Rahman & LouIis
Kahn D. Nithun Kundu
30, Who can pass and enact Ordinance?
A. Speaker B. Prime Minister Cr President D. Chief Justice
31. What is the length of the Padma Bridge?
- AC 6.15km B. 6.5km C. 6.75km D. 7.15km
32. In which sector of Bangladesh the demand of water is the highest?
A. Residential B. Transport C, Agriculture D. Industry
33. What is the area of Saint-Martin Island?
A. 8sqkm B. 10sqkm C. 12sqkm D. 14sqkm
34. High quality coal is found at in Bangladesh.
A. Jamalgonj B. Jokigonj C. Bijoypur D. Ranigonj
35. Which is the smallest district of Bangladesh by area? ‘
A. Narayanganj __ oe en . = a4 C. Satkhira = D. Gazipur
oS x aa Sar
—-~ ~~ Logical Reasoning (Questions 36 — 40)
36. 4" date of month comes after 2 days from Tuesday, then which would be the day on the day before 18® of that
month?
A. Monday B. Wednesday C. Thursday D. Sunday
In a row of trees, a tree is 7" from the left end and 14" from the right end. How many trees are there in the row?
A. 18 A -B. 19 £. 2 D. 21
38. Qis rae as old as P but twice younger than T. R is half the age of P but is twice older than S. Who is the second

A. Q B. T Cc. R a:A4 D. S
397" “ina garden, the ratio of the number of coconut treées*t6"that Of m is'S:6*respectively. If the total number of
trees is 121, then how many coconut trees are there in the garden? ‘ 22144 wrt )a uy i
A. 38 B. 55 Ce ee oe —
40. If the first four numbers of an arithmetic sequence are 167, c. 159, 155; then hat would be the sixth number?
A. 161 B. 143 : 51 BD. 147
aN e SN LAWNS
at Mathematics (Question. 54 yo ug 7
41.—
ALAN
The total age of three members in a family is 27 years:
;
What will be the total of
$I
their ages after three years?
a -
A. 33” B. 36 Peer His: 39 D. 42
42- A line segment is formed joining at least : “B
A. _ three points Br two points C. four a D. more than four eat
43. Aplane travelling 600 miles per hour isseule puny fro eet a a 58pm see
what time will it arrive at the airport? OF eg ott
os
i s
A. 5:30pm B. 5:03 pm JC. a pm 5.01 pm
(44. 1faman were to sell his chair for Tk. 720, he would lose 25%. To gain 25%, he should sell it for:
ze A. Tk. 1200 28. Tk. 1000 C. Tk. 950 D. Tk. 900 ag ©
P is two years thetthan RK who is twice as ve as R. If the total of the ages of P, Q “= Rbe 7 then how old is Q?
A. 7 Q2zeR B. 8 Cc 9 0-5 0°
Which of the following is the highest?_,” vit > ange 1) \ ;
A. LOdsSR B. mis” Wie c. asno0?' 126250 15 pst
Find the value of x and y from the following eq fiona ek. Ad \
2x+3y=26 Bxt lor eon aw 15 Ita( a o> yo
3x-4y=05 A — wy?KS ca
2. Y Ww
A. x=4; y=6 B. x x =7; y=4 © ew fhh y=5 D. x=8y=3
—z 48. \ A company wants to ship some
ne laptops. If the weight
Stott the +r plus one laptop
is 6 pounds and the weight of the
x plus two laptops is 10 pounds, then what is the weight of the box and what is the weight of the laptop?
(2, 3) B. (4,5) y 4)
(2, 4) D. (4,6)

kyle

An
TH ~AEARAR 2 2E
Zz SRe- 25 3 R25 a
ks
rny-*e
7
wt :, bie
49. 1
1£0.13 x P? = 13, then P is equa rs

50. if: 2.14x+49 = Ee


7
a aX ak B. 5 AS | 6 D.
cs fee
Sl. x f= a
If 2x7= 7 and OIE

is WeaIyP > B
Ke + 2)° =?» BEA
What is the value of (5+10
B. 2 wwe (OC. (CO D. 10
a. ; 7" e\
53. Opposite vertical angles are
A. half of one another “ 1 0 Bone third of one another
C. equal
to the other D. two times larger than the other
Lt
¥ Any angle greater than 180° but smaller than 360° is called
A. straight angley BD. reflex angle C. right angley
a
D. acute angle
5p: A cricket ground has a length of 7.5 meters and a width of 4 meters. The area will be:
A. 300m? B. 15m’ Cc 25 D 30m

gsr

jo"
i" 4“
———e

~ MBACEVEN
ISS
.)
, . * IN G) Program (33-83 stb)
M {

DU, Admission lest Summ


er-2016
S e

gs 1: En gl ishanLa ngua n & Current Affairs (1-3. 9)


Reead the fo ll ow
w
B ks
in vie and sw er he
question
5 3

in the World a th S that foll f


s Pade has
of cing a g
? ra es ti on
ss deshr’sen competit ors suc each as India.
a
e ease
ed lac
of do nela
sing
inrmg. buBa
re gu l
led all i atory ;
alve re fo
tom, stha Bangladesh below
Oo

caammbb od ia
al e an d“s Vi et
aet e na ra nk in g on i
1
over al l
Ir aq . Ba ng la de sh Caan in the gu la to ry sy st em and a chaned ge 10 NS
to a"
Yemen and rg in its re comp ar
on the number of en te d on the sub-innidi e cators oxides
ranking depends Every ye ar ms, the
im plW em Repo rt pr
: in the list.
D
World Bank’s oing Business
the one cova complying “3
, the
. si ne ss
ea sy or diffi ca rt um si ze d bu
a small to medi es>>
how
an 1dea of cu
is to s t a r t bus ine ss sta rti ng 2 busini
significant re gu la ti on s aff ect
ct i It it
in the liferi of a
e ngcycleproperty. getting credit. pr ot ec ti ng
°
eons , electricity,in are as reg ist reso lv in g
ing
settin g co ns tr uc ti on en fo rc ‘ g contracts. and
in
1 g across borders,
minority inve
stors > payin g taxe , tradin
s
to rs wh il e th e other five
‘adica
insolvency. In th de sh lo st grades on five for st ar ting 4 busine
ss.
"Te 2 Bang la the in de x
ed uber, . di dr op pe d the most in property and prot ion in
ecting
eanr y in re gi st er in g ct
credit in equal parts taxes due to the
redu
followed & by HIBES, and then s impr ov ed on pa yi ng
cies
or s. Wh il e Bangladesh lia be ca use the others did
better.
minority in ve st
l rank in g dr op pe d
the corporate tax rate, the overal
stem
01. The passage 1S
about:
rl d b. Wo rl d Ba nk’s regulatory sy
around the wo
a. Doing business overall World Bank
ranking
pl ac e in th e ss
¢. Bangladesh’s nk in g in th e ea se of doing busine
global ra .
d. Bangladesh’s si ne ss in Ba ng ladesh
of doing bu ement in the past
year?
e. The life cycle sh ow n im pr ov s
ar ea ha s Bangladesh g co ns truction permit rms
02. In wh ic h b. Ge tt in
rd er s ©. Ca rrying out refo
siness ss bo
a. Starting a bu d. Trading acro
ness?
co un tr ie s in doing busi sier
th e W o r l d Bank rank To ma ke do ing business ea
es b. available
03. Why do ke credit easily
d. To ma
ies
c. To enforce
complia n c e
o m i c pr os pe ct s of the countr B angladesh's po
sition
futu re e c o n si ne ss ha s
e. To show the ng a bu
in th e li fe cycle of starti
ny areas
04. In how ma in the past y
ear?
nchanged
remained u d?
a.5 B a n g l a d e sh droppe
th e o v e r a ll ranking
of
b. O t h e r s did better
s missing
05. Why ha o r s w e r e
c a t
a. Key indi cators remaine
c. 5 key indi
Iraq fe
d. Yemen and
s regulato
e. Bangladesh’
497
EMBA # 63
Find the correct sentence (Questions 6-10)
6. a One of the boy’s is meeting me today. b. One of the boys are meeting me today.
. One of the boy is meeting me today. d. One of the boys is meeting me today.
Q
e. One of boys meeting me today.
7. a. He is too important for tolerating any delay. b. He is too important to tolerate any delay.
c. He is too important at tolerating any delay. _d. He is too important with tolerating any delay.
e. He is too important for tolerate any delay.
8. a. The intruder stood quietly for few moments.
b. The intruder stood quietly for a few moments.
c. The intruder stood quietly off a few moments.
d. The intruder stood quietly over few moments.
e. The intruder stood quietly in a few moments.
a. A person’s true character, it seems, is revealed in such a situation like this.
b. A person’s true character, it seems, is revealed in a situation such as this.
c. A person’s true character, it seems, is revealed from a situation such as this.
d. A person’s true character, it seems, is revealed from a situation such as this.
e. A person’s true character, it seems, is revealed at situation similar to this.
10. a. Between you and I, I doubt that he will come.
b. Between you and I, I doubt that he would come.
c. Between you and me, I doubt that he will come.
d. Between you and me, doubt that he would come.
e. Between you and me, I doubt he comes.
Select the appropriate words (questions (11-13):
11. The release of the movie was timed to with the director’s sixtieth birthday.
a. reflect b. coincide c. celebrate d. harmonize _ e. patronize
12. the northern frontier of India is the Himalayan range.
a. Aside b. Along c. Beside d. Above e. Between
13. He has many awards and certificates under his
a. profile b. folder c. pillow d. table e. belt
For each sentence, find the correct option that expresses the meaning of the underlined
words (questions 14-16):
14. Ambiguous newspaper headlines are sometimes funny.
a. Hilarious b. Ambivalent __ c. Inappropriate d. Uncivil e. Inept
15. He travels to his home village once in a blue moon.
a. frequently b. every fortnight c. every month d. rarely e. regularly
16. Incessant rains for the last few days mean trouble for the farmers.
a.Infrequent § b. Unseasonal _c. Continual _d. Intermittent e. Unwanted
Select the correct prepositions (Questions 17-20):
17. He is suspicious my motive.
a of . b. from . c. in d. . with
wi e. to
18. He will, all probability, become the next Prime Minister.
a. at b. under c. with d. on e. in

498
| MBA (Evening) Program, DU Admission Test Summer, 201 6 |

19, After getting _ the plane, we walked to the immigration counter.


a. on b. out c. from d. off e. of
90. You are still alittle new____all this.
a. to b. in c. of d. about e. off
91. Which player has first scored 500 runs ina first class cricket match?
a. Bradman b. Hobbs c. Hanif d. Lara e. Sobers
9). The country that has recently taken up a plan to use mineral resources surrounding the
solar system is:
a. USA b. France c. Luxemburg d. Belgium __ e. Russia
93. The current chair of the G20 countries is:
a. UK b. Germany c. Italy d. Australia =e. Japan
Note: 46aIcy Indonesia (2022)
24. The highest tax payer in Bangladesh in the year 2014 — 15 is:
a.RMG sector b.ICB c.Grameenphone d.SquareGroup _e. Bata Shoe
25. VSAT stands for:
a. Virtual Satellite b. Volatile State of Airbag Technology
c. Visual Sonic Ari Time d. Very Small Aperture Terminal
e. Virtual Site Automation Technology
26. The present position of Teletalk in terms of market share in Bangladesh is the:
a. 3" b. 2™ o. 5" d. 6" e. 7"
27. ‘Skyrocketing’ is aterm used in: a.airsports _b. space industry
c. expressing high growth d. moving high up in the mountains
e. bombing from rockets in the sky
28. One of the instruments used in temporary and ‘automatic halting’ of share trading is:
a. embargo b. halt-trade c. stop-sale d. circuit breaker e. lock-in
29. Which of the following countries is not a member of South Asian Free Trade Association
(SAFTA)?
a. Afghanistan b. Myanmar c. Bhutan d. Maldives __ e. Nepal
30. According to current FIFA world ranking (February 2016) the country in the 1* position is:
a. Spain b. Brazil c. Argentina dd. Belgium e. Netherlands
Section II: Logical Reasoning/Critical Reasoning & Mathematics (31-55)
31. The homeowners aged 40 to 50 are more likely to purchase ice cream and are more likely
to purchase it in large amounts than are members of any other demographic group. The
popular belief that teenagers eat more ice cream than adults must, therefore, is false. The
Wea

argument is flawed primarily because the author.


a. fails to distinguish between purchasing and consuming
b. doesn’t supply information about home owners in age group other than 40 to 50
+e

C. depends on popular belief rather than on documented research findings


Bm

d. doesn’t specify the precise amount of ice cream purchased by any demographic group
€. discusses ice cream rather than more nutritious and healthy foods
BG

499 LPrx>!
32. A company is considering changing its policy
regarding daily working hours. Cur
this company requires all employees to arrive at work at 8 a.m. The proposed ny
would permit each employee to decide when to
arrive — from as early as 6 a.m. to 5 a
as 11 am. The adoption of this policy would be most likely to decrease ethistegreed?
productivity if the employees’ job functions required them to
a. work without interruption from other employees
b. consult at least once a day with employees from other companies
c. submit their work for a supervisor’s eventual approval
d. interact frequently with each other throughout the entire workday
e. undertake projects that take several days to complete
. Eating beets significantly lowers the risk of cancer, according to an article in a nutritional
magazine. The article refers to a study that people who consumed one or more beets per
dav were half as likely to be diagnosed with the disease as people who did not. Which of
the following, if true, most weakens the argument in the magazine article?
a. Another study found that people who consumed one tablespoon of flax seed oil per day
were more than four times less likely to be diagnosed with cancer as those who did not.
b. Participants in the study reported consuming no vegetables other than beets.
c. The study was only conducted in one city.
d. In another experiment, cancer patients who ate one or more beets per day were no
more likely to recover than those who ate no beets.
e. The participants in the study who ate beets were more likely to exercise regularly than
those who did not eat beets.
Ali is twelve years old. For three years, he has been asking his parents for a dog. His
parents have told him that they believe a dog would not be happy in an apartment, but they
have given permission to have a bird. Ali has not yet decided what kind of bird he would
like to have. Find the statement that must be true according to the given information.
a. Ali’s parents like birds better than they like dogs.
b. Ali does not like birds.
c. Ali and his parents live in an apartment.
d. Aji and his parents would like to move.
e. Ali does not iike pets.
35. Fruit known as amla in certain parts of Asia is an excellent source of vitamin C. A small
quantity of the fruit grated and added to salads provides almost all the daily requirement
of this vitamin. However, the fruit is very sour. A new process designed to remove most
of the sour taste wili make the fruit acceptable to American tastes. We are therefore
starting to grow this fruit for sale in the United States. The argument above assumes all of
the following except
a. Americans generally won’t eat very sour foods.
b. The new process does not remove a significant part of the vitamin content.
c. That a market exists for a new source of vitamin C.
d. The fruit can be used only in salads.
€. Apart from being sour there are no other objections to eating this
fruit.

500
[MBA (Ev
(Eveeni
ni ng)

peo
Program, DU Admission Test Summer, 2016 |

36. The average (arithmetic m W What


a4" obtained by a student on 3 subjects is 78.
- e i

must the silicate


be to be 80?
wubjects #, aoe e subject for the student’s average marks on the 4

ei ; b. 81 c. 82 _-d. 88 €. 86
37. wo M and N
friends After some time a third
and 60% shares in a business.

cant 85 ES
a
friend joined the cain
ee eee ae
What is the share of M in the business sc
a.The20%differene b. 24% c. 28%% d. 32% e. 36%
square of the smaller number Is equal to
38. Fourts erence between two numbers is 3. If the
oe a the larger number, find the two numbers. e Bothaandc
or b.4,5 c.6,9 d.7,8
‘ into two parts. Each part is bent to form a
square. Given
a0. th 44cm long is cut
at
the of wire find the perimeters of the two squares.
the total area of the two square is 65cm’, 10cm
a. l6cm, 28cm _b. 16cm, 24,cm__. 14cm, 12cm d. 20cm, 24cm ©. 12cm,a loss of 16%,
40. If a merchant offers a discount of
30% on the list price, then he/she makes
of 10%
an ae will the merchant make if he sells at a discount
of profit or loss
of the list price? ©. 6% loss _
_b. 6% profit c.0.8% loss d.8%profit
a.6.25% loss miles a minute. How far will the car
hour, and a plane travels 10
41. A car travels 50 miles an
travel when the plane travels 500 miles? d.39.7miles_ ©. 56.8 miles
miles b. 37.5 miles c.41.6miles once. How
a. 504
each of six players will play every other player exactly
42. In a chess tournament
the tournament?
many matches will be played during d. 36 e. 18
b. 15 c. 30 of 65
a. 12 a motorbike travelling at a speed
at 100 km per hour overtakes
43. A train travelling train in meters?
onds. What is the length of the
km per hour in 40 sec Le
d.550 meters ©. 600
b. 450 meters c.500 meters
a. 400 meters ch one of th
of boy s in a clas s is thr ee times the number of girls. Whi
The number children in the class?
following numbers cannot represent the total number of
d. 40 e. 36
b. 44 c. 42
a. 48 Tk. 132. A customer
of 54 ora nge s is sol d for Tk. 162 and a box of 27 is sold for not
45. A box 2 box es as one box with 54 oranges was
ora nge s but he had to buy
needs only 54 tomer had to
available. Approximatel y, how much was the ‘excess’ money that the cus
pay for each orange? c. Tk 1.10 d. Tk 6.0 e. Tk 3.0
a. Tk 3.30 b. Tk 1.90
school 7 km away. He cycles at 10 km/h
_ A cyclist leaves home at 10.00 hours to cycle to
has to push his bicycle the rest of the way at 3 km/h. He
until he has a puncture, then he How
s. far did he walk?
arrives at school at 11.10 hour c.4km. d.1km. e
a. 2 km. b. 3 km. After four day s two “ .S km.
47. Four persons can complete
a job in 8 days.
wou ld it take for the rem ain ing per sons to c persons have left the
y days
work. How man ee the work?
b. 6 c. 4 18
a. 10
ee

501
t EE |
48 There are 125 chips on the table. If as many of the chips as
Possible are to be arranged
into an equal number of 3 chip and 4 chip stacks and the remaining
chips are to be
removed, how many of the chips are to be removed?
a. One b. Two Cc. Pe d. Six e. Seven
49. Solve the equation of x: —+ =—
x xta x+b
i ab b. ab . ab d ab e ab
a+2b a—2b a+b -a-b 2a+2b
. Ifx and y are non-negative, simplify (8 1x" yy
a2 ae Is 9 7 9 Ww?
a 8ix*yv? bOoxty? c.3xty? d3xty? se. 3x*y?
Si. Which one of the following is the least value of ae x ?
x
al b 2+x a2 q its et
3 — 6 6 "3 6
_ Ifa + 2b =6 ab = 4, what is the value of a4?
a
1 3 5
_ b. 1 c. — d. 2 e. —
™'3 2 2
53. Determine the value of (5”)’.
a. . -50 b. -20 c. -100 d —] eHZ
625 25
A circle has an area of 45cm”. What is the radius of the circle?
a. 3.78 cm b. 5.43 cm c. 3.21 cm d. 4.35 cm e. 2.10 cm
5)- A circular logo is enlarged to fit the lid of a jar. The new diameter is 50 percent larger
than the original. By what percentage has the area of logo increase?
a 50 b. 80 c. 100 d. 125 e. 250
MBA(EVENING) Program (34-84 U1) Waal ettot
B Pave #516
DU, Admission Test Fall-2016

Section I: English Language & Current Affairs (1-30)


Read the following paragraph carefully and answer the questions below AL kee
British oil giant BP will merge its Norwegian assets with Det Norkse, Norway's biggest
independent oil company, in exchange for a minority stake of 10.8 billion kroner ($1.3
billion) in the new group, the two sides announced recently. Named Aker BP, the new entity
will hold 97 licenses on the Norwegian continental shelf, including 46 as the operator. It will
Produce around 122,000 barrels of oil equivalent per day, based on 2015 figures,
with
reserves estimated at 723 million barrels. It will be 40 per cent owned
by Norwegian group
Aker, the holding group that owns Det Norkse, and 30 per cent owned by
BP. The remaining
30 percent will be held by Det Norkse’s other shareholders. “With this transaction, we
Provide Det Korkse with operational strength, a robust capital structure and two
solid
industrial owners, thereby creating a platform for further growth
on the Norwegian
continental shelf and near term capacity to pay out quarterly dividend,”
Kjell Inge Rokke,
Aker’s main owner, said in a statement. In practice, Det Norkse will issue
135.1 million new
shares at 80 kroner apiece which will go to BP, which will
in exchange turn over its
Norwegian assets. Aker will then acquire 33.8 million of these
shares.
01. A suitable title of the passage would be
a. Explorations for oil in Norwegian continental shelf
b. The merger of BP with Aker
c. Exploration of Norwegian oil by BP
d. The merger of Det Norkse and BP’s Norwegian assets
e. Acquisition of Norwegian oil assets by BP
02. What will Det Norkse get if BP’s Norwegian assets are merged with the
company?
a. The brand name BP
b. 97% license as a stakeholder
c. Shares of value $1.3 billion in the merged company
d. Shares of value $10.8 kroner in BP
e. 122,000 barrels of oil per day
03. What will be the total stake of BP and Det Norkse’s other shareholders (in%) in the
merged company?
a. 70 b. 40 c. 80 d. 60 e. 100
04. Approximately how much of the estimated total reserves of oil is planned to be
extracted per day by the merged company?
a. 5% b. 7% c. 0.05% d. 0.017% e. 0.12%

510
S
a. 10808 h pas in the process of the proposed merger? >i04
Find the correct sentence (questions é10 _ 1 d. 101.3 e
06. a.A basket of
c apples are rotten. b. A basket of apples get rotten.
, a or a Possibly rotten. —_d._ A basket of apples is rotten.
orf oO

07. .‘Binal
Bina left! fo orb
Re Ool pon
after the ee
rain "has stopped.
€it tor school after the rain had stopped.
‘Bina leaves for school after the rain
has stopped.
ao

. Bina had left for school aft


er the rain stops. re
. Bina left for school after the rain
had stopped.
f O

08. - As we crossed the street, a car almost struck


us.
ao

. Crossing the street, a car almost struck


us.
- A car almost struck us, crossing the street.
d. While a car, crossing the street, almost struck us.
e. When having crossed the street, a car almost struck us.
09. a. He worked and sang from moming then to night.
b. He worked and sung from morning into night.
c. He worked and sang from morning till night.
d. He worked and sang from morning through night.
e. He worked and sang from morning towards night.
10. a. The team comprises of members. b. The team comprising six members.
c. The team comprises six members. d. The teams comprise of six members.
e. The team comprise of only six members.
Select the appropriate word (Questions 11-13):
11. Catching the earlier train will give usthe__ to do some shopping.
a. need b. luck c. possibility —_d. occasion e. chance
12. Isawa of cows in the field.
a. group b. herd c. swarm d. flock e. school
13. The grapes are now enough to be picked.
a. ready b. mature c. red d. advanced. ripe
that best replaces the underlined portion of the fo
Choose the alternative llowing
sentences (questions 14-16):
14. He is not diplomatic.
a. He is not intelligent. b. He is not cunning. c. He is
e. He is not naive. 1s not a fool |
d. He is not tactful.
15. I met him after a long time, but he i me the cold shoulder.
. insulted me 2
a. abused me
€. quarreled with me . Scolded me
d. ignored me

S11
aleeee
16.

a. was reget as ie was looked upon as |


C. was thought to be d. pretended to be e. looked like

Choose the appropriate preposition (Questions 17-20)

17. He is completely absorbed ___his research work.


a. at b. of c. on d. in e.up
18. On the way we called __a friend’s house.
a. over b. in c. at d. of e. to
19. Do you believe__ ghosts?
a. of b. on c. at d. in e. to
20. In hts mother’s old age, he looked ___her.
a. after b. on c. at d. upon e. towards
4i, Bangladesh has the highest trade deficit with )
a. India b. Bhutan c. Pakistan d. China e. Saudi Arabia
22. This year’s Copa America title went to Chile. Which country won the title in 2012?
a. Argentina __b. Brazil c.Colombia _d. Uruguay e. Chile
Note: 2012 >I Copa America
afew Safe | 2011 cet one BOTH |
23. The immediate effect of BREXIT Referendum results is:
a. Decline in the price of gold b. Rise in the exchange rate of British pound
c. Rise in both the price of gold and the exchange rate of pound
d. Decline in both the price of gold and the exchange rate of pound
e. Rise in the price of gold and decline in the exchange rate of pound
24. In June 2016, the price of palm oil per ton in Malaysia was US $562. How much was
the price per liter in Taka if the $-Taka conversion ratio is 1 : 79?
a. 46 b. 82 c. 76 d. 89 e. 58
25. What is the name of the central bank of China?
a. Central Bank of China b. Bank of China c. China Central bank
d. People’s Bank of China e. Reserve Bank of China
26. What is the name of the place in the Bay of Bengal where BAPEX is now elanning tot
explore offshore oil and gas through drilling?
a. Santos b. New Mooring c. Sangu d. Talpatti e. Magnama
27. Which national airline stopped carrying direct cargo from Bangladesh in the last week
of June 2016?
a. Cathay Pacific b. British Air — c. Emirates d. Lufthansa _ e. Japan Airlines

512
[MBA (Evening) Program, DU Admission Test Fall, 2016 |
Which ancient city of Syria has been destroyed by the anti-Asad forces in the country?
a. Palmyra b. Damascus c. Aleppo d. Bel e. Tadmur
What is the population density per sq km in Dhaka city according to the April 2016
Report of the Demographia World Urban Areas?
a. 44100 b. 4120 c. 3120 d. 29200 e. 11080
30. Name the country, a significant number of politicians of which recently demanded that
English should be scrapped as the EU language.
a. Germany b. Italy c. Russia d. France e. Belgium

Section II: Logical Reasoning/Critical Reasoning & Mathematics (31-55):

31. The Faculty of Business Studies, University of Dhaka opened a food court on the gr ound
floor of the EMBA Building a few years ago. If a researcher wants to conduct a survey of
students about their view of the food court and its role in bringing change in their life in
the campus, which students may be considered the most representative sample?
a. Students getting admission in the Faculty after the food court was opened
b. Students taking part in the management and running of the food court.
c. The current, past and prospective new students of the Faculty.
d. Students studying in the University but not in the Faculty.
e. Students who used canteens, cafeteria, street foods shops etc before opening of the
food court.

32. Photography is no longer an art form. Nowadays digital cameras are used for generating
perfect images by simply pointing at the subjects. The argument apparently assumes that
a. selection of subjects or learning the art of photography is not important.
b. digital cameras will continue to improve in quality.
c. digital cameras can never go wrong.
d. photography with all other types of cameras is an art form.
e. art is not perfect.

33. A doctor asked his patient, “Do you have high blood pressure”? The patient replied, “I
have not seen a doctor”. What can be concluded from the above conversation?
a. The patient does not have high blood pressure.
b. The patient visited the doctor because he had high blood pressure.
c. The patient has no idea about high or low blood pressure.
d. The patient had never visited a doctor.
e. The patient did not answer the question directly.

EMBA # 65
513
Le es ge ae eg es ae
34, On weekends, Mr. Karim spends many hours working in his vegetable and flower
gardens. Mrs. Karim spends her free time reading and listening to classical music, Mr,
and Mrs. Karim like to cook. Which of the following statement 1s true? :
a. Mr. Karim is a gardener. b. Mr. Karim does not like classical music,
c. Mrs. Karim cooks vegetables that Mr. Karim grows.
d. Mr. Karim enjoys planting.
e. Mrs. Karim enjoys reading nineteenth century novels.
35. Assume that a distinguished professor of the Faculty of Business, University of Dhaka
claimed in a meeting of the teachers of the Faculty that “None of us is going to vote to
make the FBS premise a smoking permitted area, so neither are you, right”? Which of
the following is true about the speaker?
a. He is stating his personal opinion only. b. He is using circular reasoning.
c. He is using peer pressure. d. He is advocating a specific cause.
e. He is presenting a no in-between argument:
36. A shop has a “buy 2 and get 1 free” offer. Sakib has Tk. 300 with him; he needs to buy
6 dolls for his daughter. If each of the doll costs Tk. 55, how much does he saver?
a. 30 b. 45 c. 60 d. 75 e. 80 :
37. Marks of a student were wrongly entered in computer as 83; actual marks of that student
were 63. Due to this mistake average. marks of the whole class got increased by half
be ?
G ). Find the total number of students in that class.
a. 25 b. 30 c. 35 d. 45 e. 40
38. In a hotel, 60% had vegetarian lunch while 30% had non-vegetarian lunch and 15% had
both types of lunch. If 96 people were present how many did not eat either type of lunch?
a. 23 b. 24 Cr 28101. d. 26 e.21
39. The sum of three numbers is 98..If the ratio of the first to the second is 2: 3 and that of
the second to the third is 5:8, then the second number is:
a. 30 b. 13 c. 17 ., 0.25 e..10
By selling a pen for Tk. 119, gain is as much percent as it costs. What is the cost of a pen?
a. 20 b. 27 c. 33 d. 40 e. 70
41. Naizel is twenty years older than Neketa. In three years Naizel will be twice as old as
Neketa will be. How old is Neketa now? .
a. 24 b. 16 c.17 d.8 e.3
42. Plane A leves City X at 11:00 a.m, and flies due east at an average speed of 120 mph.
Plane B leave City X at ten minutes past noon and flies due south at an average speed of
160 mph. How far apart are the two planes at 1:40 p.m.?
a. 400 miles _b. 380 miles c.360 miles 4.340 miles _e. 320 miles
43. A container holds 40 gallons which is 80% water and 20% acid. How much of the water
must be evaporated to produce a solution containing 50% acid?
a. 22 gallons _—b. 28 gallons c.26 gallons d.24 gallons — e. 32 gallons

} 514 | cee
Oa

| MBA (Evening) Program, DU Admission Test Fall, 2016 |

The tens’ digit of a two-digit number is 3 less than Urersnibnstlllla


44. 1s 1
reversed, the sum of the original number and the new number
original number? -
a. 69 b. 58 c. 47 d. 25 e. 14
n is divided
45. If n divided by 7 has a remainder of 2, what is the remainder when 3 times
by 7?
a. 2 _ b.6 “5 d. 4 e.7
46. How many times is the digit ‘4’ used in writing numbers from 10 to 100?
a. 10 b. 11 c. 13 d. 15 e. 19
isx m ore
The ratio of boys to girls in a class is m : n. If the number of boys
than the
47.
number of girls then the number of boys is:
nx m-n mx m-n m+x
a. b. c. d. é.
m—n nx m-n mx m+n
1

If4 20
2=then 48
the value of n is:
a7 b.6 costs d. 8 e.9

49. If Jm+JVm-5 = 5, what is the value of m?


a8 b.7 c.6 d.9 e.5
? l Pane ;

Simplify the expression: Sate


3n

a. = b. 2.5" eoseDPmis s d. . e. 10

51. If one root of the equation x” +'3x =k = 0 is 2, what is the value of k?


b. 12 ad Ved ah - > =d16 =v e. 10»
a. 20
d to
For x/y if 3 is added to the numerator, the resulting fraction is /%;'while if 4 is:adde
the denominator the resulting fraction is,4, What is the value,of x4?
a 5/16 b.2/10. CAB de 4/14 5 > ©1120
53. If the diagonal of the square is 6 2. in., what is the area:of the square in square-inch?
a. 78 b. 72 c. 54 d. 64 e. 36
If the perimeter of a rectangular flower bed is 30 feet, and its area is 44 square feet,
what is the length of each of its shorter sides? TENS 3
a4 b. 5 c. 6 d.7 e.8
5S. In the following figure, ASB is a quarter of a circle. PQRS is a rectangle with sides PQ
=§ and PS = 6. What is the length of the arc AQB?
Ai

pi Q
|
S B
R
| a. Sx b. 82 c.10n d. 15x e. 207

515
MBA(EVENING) Program (35-84 %)
watery

DU, Admission Test Spring-2017


INSTRUCTIONS
Test Duration : 55 mins (Section I and section II) + 35 mix (Section III) = 90 Minutes
Total Marks —: 55 (SectionI and section II) + 25 (Section HI) = 80 Marks
The test is divided into Three sections:
Section I — English Language & Current Affairs (Questions 1-30) ;
Section II — Logical Reasoning, Critical Analysis and Mathematics (Questions 31-55)
Section III — Writing Ability Test (Questions 56-60)
Each question of the MCQ part carries one mark
For every wrong answer to a MCQ 0.20 mark will be deducted
Writing Ability test contains 5 questions carrying 5 marks for each (5 x 5 = 25).
| Write answer to a question only in the space left after it, no additional paper will be supplied.
e Admit Card and photograph of each candidate would be checked
© No one will be allowed to sit for the test without admit card
© Use of calculator is not permitted
e Carrying a mobile phone or similar device during the exam will cause cancellat
of theion
script
e Check the question and answer script before start of the examination. It is advised to
change the script in case the printing is not clear or any blank page is found.
Section 1 : English Language (Questions 1-20) & Current Affairs (Questions 21-30)
Read the following paragraph and answer the questions that follow (question 1-5):
Climate is dynamic and changing. Now-a-day, climate change is one of the most debated
issue in the world. Human activity is the prime cause of global warming along with natural
disasters. The scientists who are studying and doing research in climate
change all over the
world are finding evidence from tree rings, pollen samples, ice cores and
sea sediments of
such changes. Climate change is a global problem, so we need a global
solution. The youth
can play a vital role in making the people aware of the effects of
climate change. They can
make a network to launch a movement for sustainable development globally.
Sustainable
development, which can successfully address the issue of climate
change, should be the
model everywhere. The youth, with their knowledge and
awareness building activities, can
drive their communities and countries towards a Sustainable
as well as peaceful future. Thus,
the youth can make our planet safe for themselves and for the future
generations as well.
01. The subject of the passage is:
A. Climate change B. Evid :
C. The role of human activity in climate shana ences of climate change
D. How the youth can contribute to climate
E. The effects of climate change itigatigation
change miti
on sustainable development

522
Ue. MUVUIUINE tO the Passage ee

C. Both human activities and natural disasters are responsible for climate change
D. The youth ar © responsible for climate
change
E. The youth can discover evidences of climate change from
sea sediments correct?
03. If we follow the logic of the paragraph, which one of the following ee tl Wonton
A. A global problem needs a local solution. B. A local problem needs a loc bal solu
tion.
C. A global problem needs no local solution. D. A local problem needs a glo
E. Local and global problems solve each other.
04. A global problem is one which
A. All the countries of the world share
B. A problem that demands attention from developing countries.
C. A problem that the west creates for the east
D. A few countries of the world share
E. A problem dealt with by the United Nations
05. According to the passage
A. The youth of today are changing the climate.
B. The youth are a strong force behind global warming. =" :
C. The youth can be leaders of their communities in finding evidence of climate change.
D..The youth can ensure a peaceful future.
E. The youth cannot guarantee a sustainable future.
Choose the dicrnatioe that best replaces the underlined portion of the following sentences
(questions 6-8) |
06. Using it wisely, leisure promotes health, efficiency and happiness. rent,
A. Using it wisely B. If used wisely - C. Having used it wisely
D. Because it is using wisely _E. Because of usefulness
07. The reason I came late to class today is because the bus broke down.
A. I come late to class today is B. I came late to class today is that —
C. that I was late to class today as because D. I was late'to class today is because
E. why I came late to class today is because
08. Since we are living in Dhaka for five years, we are reluctant to move to another city
.
A. Since we are living B. Being that we are C. Being that we have been
D. Since we have been living E. Since we were living
Find the correct sentence (questions 9-12);
09. A. I don’t like when she tells me that I have to study,
B. This guide gives useful informations about the City,
C. The law states that smoking is forbidden in
most public areas.
D. He cleaned the house and after he ironed the clothes.
E. The team are doing good work.

523
[MBA (Evening) Program, DU Admission Test Spring, 20 17 |
10. A. The boy who does best he will; get a prize.
'
B. One should not waste will
his time,
be :
C. Everyone have seen that movie D. If we will be late, they m
i tching TV.
11 i. cain ba ed a B. Could you tell me why did I fail?
C. I look forward to see you next month.
D. IfI were taller, I'd play basketball. :
E. | am not agree with an =
12. A. What means this word? B. I suggested pane *
C. Surprising is that no one was hurt. D. I have not an iPhone
E. I need to concentrate myself on my homework.
Select the appropriate word (questions
13-1 6)
13. Th ey are offering to the fire victims.
A. contribution —_B. compensation C. continuation D. confiscation E. consultancy
14. The bus arrived
A. late B. lately C. belated D. latterly E. latter
15. The taxi drivers in Dhaka driv
e
A. dangerous B. danger C. dangerously D. dangers
16. The patient Says he is now E. dangerousness
for the operation.
A. serious B. ready C. prepare D. nervous E. expectant
Choose the appropriate preposition (que
stions 1 7-20):
17. He was accused theft.
A. in B. by C. at D. of E. with
18. Rahim is endowed natural talents.
A. with '
B. of C. by D. on
19. I detest prying _ E. after
someone’s secrets.
A. for B. after C. into D. about E. at
20. Joya apologized her brother’s misbehaviour.
A. of B. for C. to D. in E. at
Current Affairs (Questions 21-30)
21. What is the position of Bangladesh
among remittance-earning countries in the world?
A. Seventh B. Fourth C. Sixthh D. Fift E. Third
22. The Nobel Prize in economic science was given
in 2016 for
A. Welfare Economics B. Contract Theory
C. Theory of Allocation D. Asymmetric information
23. The first woman to climb the Mount Everest was E, Portfolio Theory
A. Karoline Mikkelson B. Valentina Tereshkova
D. Promila Thapa C. Junko Tabei
E. Nishat Mazumder |
24. When was the last amendment in er
the Bank companies Act made in Ban
A. 1991 B.2 012.. = €. 2013 gla desh?
25. Which one of the follow D. 2014 E. 2015
ing is the newest scheduled bank
A. NRB Commercial Bank in Bangladesh?
B. Jumuna Bank
D. Meghna Bank
E. Shimanto Bank
C. Union Bank

524
~
26. WhICA Country has becom ——ie e ion?
dete te See te ty

A. Denmark scome the world’s first country to ban deforentat —_


an What amount of fare C; Netherland "Di Poland Seni for tack iN
| aid was pledged by the Worl d Bank to Bang
climate change?
A. $2 billion B.$4 billion C.$89millionD. $1 million E. ¢ | billion
28. Which of the following is considered | country?
a Eurasian E, Turkey
alae : B. Saudi Arabia C. Albania —_D. Greece
a). Where is the “Bangladesh Square’ situated?
A. Mozambique B. Burundi C. Sudan D. Senegal E. Liberia
30.
Who among the following women has been honored as “Freedom Fighter” in 20 IP
podah Imam B. Ferdousi Priyabhashini
C. Begum Rowshan Ara D. Ivy Rahman E. Sufia Kamal

31. Read the Statements (1) and (II)


in
I. The prices of petrol and diesel in the domestic market have remained unchanged
past few months. Ee oo tas
Il. The crude oil prices in the international market have gone up substantially 10
few months.
The relationship between the two statements can be stated as:
A. Statement (1) is the cause and Statement (II) is its effect.
B. Statement (II) is the cause and Statement (1) is its effect.
C. Both Statements are independent causes.
D. Both Statements are effects of independent causes.
E. Both Statements are effects of some common causes.
32. People of Generation Xers born between 1965 and 1981 tend to be more challenged when
they can carry out tasks more independently and this makes them the most
entrepreneurial generation in history. Which of the following best supports this
statement about the generation Xers?
A. They work harder than people of other generations.
B. They have a tendency to be self-directed workers.
C. They have an interest in making history.
D. They tend to work in jobs that require risk taking behavior,
E, They like to challenge their bosses’ work attitudes.
33. Amloki, an Asian sour fruit, is an excellent source of vitamin C and a small quantity of it
grated and added to salads provides almost all the daily requirement of this vitamin. A
new process designed to minimize the fruit’s sour taste will make the fruit more
acceptable to Americans and it will be profitable to grow the fruit for sale in the Unite
States. The argument assumes all of the following except nited
A. Americans generally won’t eat very cn! tab
B. The new process does not remove a significant part ds Wirt all 3
C. The ket exists for a new source of vitamin : ofthe fruit’s vitamin content.
D. The fruit can be used only in salads,
E. Apart from being sour there are no other objections to cating the fruit.

525
: or neem conn HnenyPPELT LETT PFT
VI) } 7
; , eT TUT f
|MBA (Evening) Program, DU Admission Test Spring, 2017]
34. In Survey of job applicants, two-fifths admitted to being at least a little ST eeadend
may however, underestimate the proportion of job applicants who are dishonest
oe
arguments A to E best explains the reason?
A. Some dishonest people taking the survey might have claimed to be honest. —
B. Some generally honest people taking the survey might have
penne * ad |
e. Some people claiming in the survey to be a little dishonest may actually
D. Some people be very stl
claiming in the survey to be dishonest might have peated Teena
ee E. Some people who are not job applicants are probably a
little dishonest.
. People buy prestige when they buy a premium product. They want to e associated with
P duction strategies
something special. Why, therefore, mass marketing techniques and price
re
should not be used for such products?
A. Affluent purchasers currently represent a shrinking portion of the population orem on haser
B. Continued sales depend directly on the maintenance of an aura of exclusivity.
oducts
C. Purchasers of premium products are concerned with quality and price of eee
wi
D. Expansion of the market niche to include a broader spectrum of consumer gous, wn
increase profits.
manufacturin
E. Manufacturing a premium brand is not necessarily more costly than =
standard brand of the same product.
36. What is the value of 3°?
A.l B. 27 ee ms E.1/3
Zi 9
unit of pencil.
37. A person needs to pay Tk. 500 to buy 100 pencils and Tk. x for any additional
If the customer pays a total of Tk. 4700 for 1200 pencils, what is the value of x?
A. 4.33 B. 4.0 C. 3.91 D. 3.85 E. 3.5
38. Find the value of: 6(-3) (1/3) (-0.25)
A.9 B. 6 C. 4.5 D. 1.5 E. -0.5
39. If 12a + 3b= 1 and 7b —2a= 9, what is the average ofa and b?
A. 0.1 B. 0.5 C.1 D. 2.5 E.5
40. Which number will continue the following series? 3,8, 15,24, 35, 48, 63, 80, __
A. 92 B. 91 C.99 D. 87 E. 85
41. Three numbers are in the ratio of 3:4:5 respectively. If the sum of the first and third numbers
is more than the second number by 52, then which will be the largest number?
A. 50 B, 65° C. 62 D. 72 E. 85
42. A shopkeeper purchased 200 bulbs for Tk.10 each. However, 5 bulbs were fused and had to
be thrown away. The remaining were sold at Tk. 12 each, What will be the percentage profit?
A. 13 B, 15 C,17 D. 22 E. 25
43. The number of boys in a class is three times the number of girls, Which one of the following
numbers cannot represent the total number of children in the class?._. .
A. 48 B. 44 C, 42 D. 40 E. 52
44. In 10 years’ time a company produces 500,000 units of commodity. From the second year, the
company produces further 1000 additional units of the commodity in each year than the
previous year. What would be the quantity that the company should produce in the first year?
A. 50000 B. 50500 C. 40000 D. 45500 D. 40500

526
4). A Tam :
rm
travelling at & speed Of OF Rat
isan - “<i per hour overtakes motorbike
hour in
» Wha t is the leng th of the trai nersin mete rs?550 PD. 350 meters
A.A ten ers
metkt
400n toc B. 450 met ers CC. 500 met _D. meters
46. he hads(0
oan from a bank at the rate of 13% p. a. simple interest. After 3 year
16,000 D.Tk.20,000 ET. 15,000

ROE
Tk. 10, 000 C. Tk.
mee age of a group of 12 students is 20 years, If $ more students join the group, the
B.
47 tsThe avear rag

TTA
e age of the new SHIR te
ee Yom . Tis ave rag
A224 years years ‘iD. 23.4i years E. 25. y
26
48. Ifx+ y= 8pands—y=6q, then xis : 5 Cc

B.4pt+ 3qC. D. 43.2p- E. &p+6q


‘Set 10 over’s of a crc kct game: osu rate was only What should be the run ra
Int he fir st
49. in the remaining 40 over’s to reach the target of 282 runs?
B.6.5 C.6.75 D.7 E. 68
A. 6.25 the spe ed of the cur ren t is 5 kph. The beat
speed 5 ofhouarsboaint favo hour
is 10 km per curr (kp h) and starting point, What was the
The elle
50. | trav d r of the ent and then retu rned to the
rete
tom
ti urn?
A. 5 hours B.10hous C.1Shours © D.75hours” E. 12.5 hours
51. A rectangular solid measures 4 units by 5 units by 6 units. What is the surface area of the solid?
A. 60 square units : B, 74 square units C. 110 square units
D..120 square units E. 148 square units
52. If Kabir loses 8 pounds, he will weigh twice as much as his sister. Together they now weigh
278 pounds. What is Kabir’s present weight in pounds?
B.135 (eee Grlook D. 147 E. 188
A. 131
square meters,
53. The len room is 1.5 times of its breadth. If the area of the room is 216
ofgth
what is the perimeter of the room? B50
B54 — —C.48 D. 42 -
A. 60
2x+x7 :
54. What is the least value of 6 9
.
Ox

vere
2+
c= arnt. 5B”
gs 2+x
6 6. . o-
3
+ 3x+ 10=1+2, thenx =
55. Ifx2 D7
A.0 B.1 C4 D.9
Writing Ability Test
Pr ee Cad Sa MLS Labial oot GS pat ern Sacica ter:
01. Bengali to English Translation 5x1=5
02. English to Bengali Translation 5x1=5
03. Précis writing
04. Short Note
AW

0S. Short essay

527
u DUM iD oaaren
m)"March-2017 fa Page# 542
section I: English Language (Questions 1-20) and Current
Affairs (Questions 21-3 ».
BeBe.
Read the following passage and answer the questions that follow (Questions 1-5)
Japan’s*s factory
factory output
output unexpectedly slipped 0.8 percent in January, the first fall in six months
and the latetest red fl ag for the world’s> number two economy, official data showed recently.
-month figure, whi i .
The on ich missed market expectations for a 0.4 percent expansion, comes a
week after Japan logged its first trade deficit in almost half a year. A weak yen has helped
prop UP the economy by driving exports, but inflation and consumer spending remain weak as
cautious firms avoid big pay hikes. Production of electronics parts and devices expanded in
January, lifted by strong demand for smartphones and other gadgets. But that was offset by a
decline in passenger vehicle production. The data dampen hopes for Japanese Prime Minister
Shinzo Abe's efforts to produce meaningful growth with a policy blitz.
Ol. The subject of the passage is
A. Japan's economic recovery B. Loss of factory production in Japan
C. Gains in factory production in Japan D. Japan's deficit economy
E. Japan's efforts to produce meaningful growth
02. The passage suggests that
A. Japan has never faced any trade deficit until January of this year
B. consumer spending is important for bridging trade deficit
C. Japan's weak currency helped the economy
|
D. A weak currency drives imports
to dampen growth
E. Japanese Prime Minister Mr. Abe wants policy decisions
xt of the passage?
03. What is the meaning of the term 'red flag in the conte
about the economy
A. Japanese flag with a red sun in the middle B. Optimism
C. An accelerated economic growth D. A sign of caution for the economy
E. A sign of expansion of the economy
04. The passage suggests that
production
A. Everyone expected a drop in Japan's factory
market expansion
B. There was an expectation about a
C. Consumer spending was robust D. Inflation was strong
uary
E. Smartphone prices were high in Jan
|
05. According to the passage many firms
A. were cautious about raising wages and salaries
B. provided salary and wage increases which contributed to inflation
C. declined to produce passenger vehicles D. contributed to inflation
E. raised red flags on their roof tops

535
7
: March
— P
program:
Admission Test of DU MBA Evenin
Choo
E. over
06. | will te ProPriate preposition (questions 6-7):
my driving test a week. p. on e
07 4 about B. in C. for ou enter e Te ee : above
- You have to leave your shoes the door when " i
es A. over Bin —_C. by ‘ i
- Fate smiles ______ those who work hard. D. on Re
F A. over B. round C. with : oe
9. He was driving his car a great speed. a E. ov’
A. on B. at C. of wi
‘Select the appropriate word (questions 10-30):
10. He will not work he is compelled. E. unless
A. till B. after C. if D. lest
11. After the initial setback, all programmes ran ren: uently E. consequential
—_B. subsequently C. consequent D. conseq
A. subsequent
12. Mahin’n career has taken some twists and turns. ‘ z - i
_ introspective
, pets E i
A. interactive B. incentive C. intuitive D. interesting
13. Doctors should as many patients as possible.
A. heel B. haul C.health _~Dz heal ee,
Choose the alternative that best replaces the underlined portion of the following sentences
(questions 14-16):
14. In Bangladesh, gender discrimination deters population control and, poverty alleviation.
A. justifies B. helps C. impedes D. stimulates _ E. accelerates
15. The celebrations went on for a whole week.
A. continued B. were disruptedC. picked up _D. were turned down E. muted
16. The country is passing through a critical phase.
A. unhappy times B. impasse _ C. crisis. D. (an) economic downturn . E. happy times
Find the correct sentence (questions 17-20):
17. A. Much efforts bring their reward.’ B- Many villagers cannot write name. 7
D. It takes a lot of money these
C. He is coward man.
E. We should love the nature. y days to pay for utilities.
18. A. I would not hear you. B, All day I was wearing a s
C. He took our his shoes. D. He came putting on peblitoait
jacket.
E. The ship was drowned
B. The more you read WO 3 a Pe
19. A. More you read, less you understand.
D! Mo
C. The more you read, the less you understand. u under
re you read, the > less you understand.
E. More reading, less understand.
20. A. The man who knocked at the door was a stranger
B. The man who is knocking at the door was a stranger
)
C. The man knocking at door is a stranger.
D. The man knocking door is a stranger.
E. The man who knocks at the door is stranger,

536
ads
current Affairs (Questions 21-30) TO alt gl; e A RRD Dhaka
OMIEA ;

91. How many government colleges have been bought under the Jurisdiction of University
A. 5 B.7 C. 10 D. 6 B.9
9, Human Rights Watch is an advocacy group located in:
A. New Delhi B. New York C. Washington D.C D. London E. Geneva
93. Cricket player Shakib Al Hassan is a/an
A. all rounder B. offspinner —C. coach D. pacer E. wicket keeper
94, Where is the headquarter of Asian Infrastructure Investment Bank located? '
B. China C. Japan D. Thailand _E. Indonesia i"
A. India
25. The tallest building in the world is located at the ‘Jewel in the desert’, that is in i
B. Istanbul C. Sahara D.Morocco __ E. Saudi Arabia }
A. Dubai
26. In which Indian city Bangladesh recently played a test match? \f
A. Mumbai B. Kolkata C. Hyderabad D.Chennai _ E. Bangalore
27. Name the most recently inaugurated commercial bank in Bangladesh?
A. NRB Clobal B. NRB Commercial C. Modhumoti Bank
D. Midland Bank E. Shimanto Bank
28. The targeted electricity production capacity of Rampal Power Plant is to be
A. 1100 MW B.1240MW —-C. 1290 MW__ OD. 1320 MW_sE.. 1420 MW
29. Which country is considered to be the Emerging Tiger?
A. Bangladesh _B.. China C. India D. Russia E. UAE
30. When did Payra Sea Port start its journey?
A. 26 March 2014 B. 13 August 2016 C. 16 December 2005
D. 12 July 2010 E. 10 Sept 2012
Section II — Logical Reasoning Critical Analysis and Mathematics (Questions 31-35)
31. During the past 20 years, computer scientists focused increasingly on startmg and
running successful business. However, since business must be profitable, computer
scientists must focus on developing products that generate profit. Consequently,
computer has lost its creative aspect.
Which of the following assumptions is most necessary for drawing a conclusion from
the argument presented above?
A. All computer programs must lack a creative aspect in order to be well received.
B. Some computer scientist entirely disregarded creativity and chose instead to pursue profit.
C. A program cannot be both creative and profitable.
D. Computer scientists are obsessed with the profitability of their work.
E. Non-profit institutions use large amounts of software.
32. Agriculture economists in Bangladesh argue that subsidy should be given on agricultural
inputs to induce small farmers. |
Which of the following arguments will weaken the statement above?
A. Large farmers are the biggest recipients of subsidies of agricultural inputs and small
farmers are least benefitted from subsidy programs.
B. Input subsidy is necessary for food security in the country.
C. Input subsidy will increase the competitiveness of local farmers.
D. The government has developed a good distribution network of agricultural inputs.
E. Development of agriculture is a priority of the present government. |

E MBA # 68 | 537
tn

arch,
‘M ara 20 17 |
20 an
|Admission Test of DU MBA Eyes 25% every year. Market alynls
33, The demand for android phone experie nced a growth
future. ’
predicts that this growth will continue in the neat the state ent above Kick HN Hidled
Which of the following statements will strengthen”. the market Wi :
A. There is increasing threat of fake android phone dabl
customers’ confidence. droid phone more affordable than
B. A large segment of people in the country find ancr
ever before due to its decreasing price. c wastes and is a threat to environment.
C. Android phone increases the burden of electroni android phone.
D. The government is planning to impose import duty on android phones.
E. Older people are not receptive to the changes in technology in fica, if yor att pking
34. Arshad has twenty years of typing experience behind him; ther Da wok no further,
for an efficient typist to enter your data into the new system, ror
The speaker assumes that
A. Twenty years of pracice ensures typing efficiency arehad :
B. The ontat ies required for ie ae is identical to what has boen:doing
C. Arshad’s job profile is the best that the new employer is going or
D. Arshad is an outstandingly fast and accurate typist.
E. Arshad will fit well into the new office.
35. Jamal’s grade point average puts him in the top third of the graduating class of college A.
Noman is in the top tenth of the same class. Ethica has the same grade pomt average as
Noman. Nancy has a lower grade point average than Ethica.
If the information above is true, which of the following must also be true?
A. Noman and Jamal both have a higher grade point average than Nancy.
B. Jamal has a higher grade point average than Ethica.
C. Jamal has a higher grade point average than Nancy.
D. Ethica and Nancy both have a higher grade point average than Jaml.
E. Noman has a higher grade point average than Nancy.
Mathematics (Questions 36-55) :
36. Three-fourth of the number of female students in a department is equal to half of the
number of male students. If the department has 1420 students, how many of them are
male students.
A. 852 ‘B. 720 C, 568 D. 284 E. 364
37. Hasan sold an article for Tk. 56 which cost to him Tk. x; If he had gained x% on cost,
what was the cost of the article?
A. Tk, 45 B. Tk. 40 C. Tk. 36 D. Tk. 28 E. Tk 38
38. If a train runs at 40 km per hour, it reaches its destination late by 11 ‘wets bat if it runs
at 50 km per hour, it is late by 5 minutes only. The correct time for the train to complete
its journey is:
A. 14 minutes B16 minutes C.19 minutes D 29 minutes “E.25 minutes

538
©” 49. Ifw is 10% less than x, ¢ #30%:ess thang, then wy.is- roent-less-t
A. 25% B. 30% ” C.37% i 41% om
40. A company employs 15 men working 44 hours a week. If 4 men are ill, how many hour
a week would the rest have to work to make up the time lost?
A. 50 hours B. 55 hours C.60hours E. 70 hours
__D. 63 hours
41. Ifa company can increase sale by 15% and selling price increased by 10%. What will be
the percentage of increased revenue?
A. 56.2% B. 26.5% C.625% D.150% E.256% i
I
42. During a certain season, a team won 80% of its first 100 games and 50% of its remaining
games. If the team won 70% of its games for the entire season, what was the total number
of games that the team played?
A. 180 B. 170 C. 156 D. 150 E. 190
43. Out of three given numbers, the first one is twice the second and three times the third. If
the average of these numbers is 88, then the difference between first and third is:
A. 32 — B. 48 C. 56 D. 72 E. 96
44. Mr. Sakil purchased 12 mangoes for Tk. 10 and sold 10 mangoes for Tk. 12, what was
his profit margin? |
A. 44% B. 144% C. 120% D. 100% E. 335%
45. a Surface area ofa cube is 1176 square inch, what is the length of the edge of the
cube?
A. 14 inch B. 196 inch C. 64 inch D.192inch E.96imch
46. Two pipes A and B can fill a cistern in 375 minutes and 45 minutes respectively. Both

pipes are opened. The cistern will be filled in just after half an hour, if the B is tamed off
after ;
A. 5 minute B. 9 minutes C.10 minutes’ D.'15 minutes E. 20 minutes
47. Today is Fahim’s birthday. One year from today he will be twice as old as he was 12
years ago. How old is Fahim today?
A. 20 years B. 22 years C.24years D.25 years © - E.27 years
48. If 8°"*4= 4° | then the value ofx is: eS
A.1 B. 3 C. 6 D.9 E. 10
49. One factor of x” — y —2y-1 isx+y+ 1, Another factor may be:
A.xty-l B.x-yt+l C.x-y D.y-x+l E.x-y-1

50. If y“= 64, x =6 anda= =» Whats the value of

A.2 B. 8 C.3 D.9 E.4


51. Which of the following is equivalent to the pair of inequalities x + 6 > 10 and x-—3 < 5?
A.2<x<16 B2<x<4 C.2<rs8 D4<x<s8 E4<x<16

539
“arch 2017
Admission Test of DU MBA (Evening) Prok
oes Find the value
ofx if 1 + < S72" 2
x x E.3
A.~1 pL c 2 D. 2
3 ae
33. If x(2x + 1) = 0 and ete 1 (2x — 3) =0, thenx==? 1
E. =
Bend pt C > D.0 2
2 a 9
54. The lines AB and CD are parallel. Calculate the size of *!
A —_p— e
134 x?

tc 48° —D —
A. 90° B. 92° C. 94° D. 96 then find out the value of
53: O is the center of the circle ABC. If OAB = 30° and OCB = 20°
X and y.

(>o x° IB

A. 100° and 50°_ BB. 130° and 20°C. 20° and 130° D. 50° and 130° E. 50° and 100°

Section III: Writing Ability Test (56 — 60)

Write answers in the space provided after each question, no extra paper will be supplied
56. Translate the following sentences into English. 5
A, PRs Met fac BHAT OTT PACHA |
B. arate ages fe mcw SATS Mf?
C. a8 taale caer free AAA AT AMT FACT |
D. cerrrecont araitinl FarcHE CHPTSR METS |
E, Boaicy ora Mesa DT ACR |

57. Translate the following sentences into Bangla.
:
A. I shall wait until you come back. |
B. The growth in the export sector this year surpassed al! expectations
C. Lam tired of saying the same thing again and again.
|
D. Our banks are financing the poor. et
E. The transport worker’s strike affected the vegetable mark

540
— SS et Tea

58. Define/Explain (in English) the meaning of the following terms in 1-2 sentences
r wi chagie
- ee es ‘eit \

A. Cyber crime:
B. Sustainable development:
C. International Mother Language Day:
D. Digital Bangladesh:
E. Black money:

59, Make a precis of the following passage and provide a title 5


A rising population beings about a number of challenges in such diverse areas as climate
change, food security, infrastructure development and health and hygiene. To feed the
fast growing population of our earth, scientists and planners have to discover new ways
to produce more. One possible way this can be done is to bring more land under
cultivation. But this can only be done in places where vast stretches of land are lying
unused. However, in a country like Bangladesh, where land use is optimum, not much
additional land is available for agriculture. A second way is to make use of new types of
seeds to produce more. Already a number of new strains of paddy and wheat have been
developed in different parts of the world. Bangladesh is one of the countries where a lot
of useful work has been done in the field of agricultural research and food sustaimability.
We have a huge population that is also gaining more buying power as economy
improves. This is putting pressure on our farmers to produce more varieties of crops and
vegetables in larger quantities. So far our progress in food production has been
commendable.

Title:
Précis:

60. Write a short essay in about 250 words on “The expected contribution of Padma
Bridge in Bangladesh economy’. : 5

541
Read the following passage and ans : uestions 1-5)
It is hard to be a farmer. Cold Swaitier aati naedaeibal and insects can eat
your crops. Your crops may need more rain than what they get. Fruits and vegetables can go
bad before they are sold. Some people say farmers can fix all of this with GM or genetically
modified food. What is GM food? All living things have DNA. DNA tells living things how
to grow. These days, people can change the DNA that tells food how to grow. When people
change the DNA of food it is called GM food. Some GM food can grow in cold weather. GM
food can stop insects from eating it. One da y GM good may feed people w ho do not
have
much food. But we still don’t know the risks GM food poses. Does GM food make people
sick, or kill insects we need? We do not know.
1. What is DNA?
A. It is something everyone needs to change.
B. It is a part of every living thing that tells it how to grow.
C. It part of every living thing that turns it into GM food.
D. It is something that is found only in GM food.
E. It is something that grows food.
2. According to the passage
A. GM food may on day feed the hungry. B. GM food can grow under water.
C. We do not know that GM food is. D. GM food makes people sick.
E. GM food kills insects we need.
3. A suitable title of the passage could be.
A. The uncertainty about GM food. B. How farmers can grow food in cold weather.
C. How we can grow GM food. D. GM food.
E. Feeding the hungry mouths.
4. What is the purpose of the passage? |
A. To educate the reader about GM food. | B. To suggest that DNA change is good.
we-need.
C. To prove that GM food can destroy only the insects that
D. To convince the reader that GM food is bad.
change.
E. To convince the reader that farmers need DNA
this’?
5. What is the meaning of the phrase ‘farmers can fix all of
A. Farmers can decide what to do. B. Farmers can solve all the problems.
ch all this. D. Far mer s can grow all the food
C. Farmers can atta
E, Farmers can depend on GM food.
Choose the appropriate preposition (questions 6-9):
6. The professor _ Bangladesh wrote a book on London.
C. to D. at E. by
A. from B. of

551
2017
DU Admissioey n Test Jul
[MBA (Evening) Program,
7. 1am indebted °
B
after
mii ae on D. to
8 ” esbyno ki ing herelh>= ePee pee D. of E. at
9. Wecantalk about it__ dinner tonight. E. under
A. in B. over C. while oe . of the senten
Choose the alternative that best replaces the underlined portion ¢
below (questions 10-12):
8

the invest
marker __madeC. buy ors D.nervous.disruption ne
upheaval in the B.stock
10. The slow- upswi ng out E. heavy investment
A. down
11. She was an unlikely candidate for the C.position.
improbable D. unqualified E. undefeated
A. losing B. difficult
12. He is intimidated by his boss. ee
A. frightened B. promoted _C. made intimate D. encouraged E. disliked
Find the correct sentence (questions 13-16)
13. A. Many villagers cannot write his name B. He is a coward man.
C. People often spend their leisure on cinema. D. I was second in my class.
E. Many a men have tried to scale the Everest.
14. A. It takes longer to reach the town by bus than train.
B. It takes longer to reach the town in bus than train.
C. It takes longer to reach the town by bus than by train.
D. It takes longer to reach the town in bus than in train.
E. It takes longer to reach the town on bus than train.
15. A. I here they’re coming for the six o’clock show.
B. I hear they coming for show at six o’clock.
C. I hear they’re coming to show the six o’clock.
D. I hear their coming for the six o’clock.
E. I hear they’re coming for the six o’clock show.
16. A. Neither are the right answer. B. I don’t know what here problem is
C. The streets get flooded every time it rains. D. I can’t bare it any longer
E. The book is one of the worse I’ve read. -:
Select the appropriate word/words (questions 17-20):
17. She likes detective books,
A. reads B. reading C. read D. havi U
18. My uncle, who is a cardiac surgeon, deals with ing tead E. of reading
A. kidney patients B. patients with vocal cord r oble
C. heart patients D. patients with gastric problems E. liver patients
19. The release.of the film __ with the 35" birthday of the actor.
A. coincided B. harmonized C. timed D. yi
20. It’s not possible for most people to things that ha ae E. celebrated
A. memorize B. recall C. dream D. tinaghie eir pllahoog: :
- €xpunge

552
sett ac a i a tte ae A

Current Affairs (Questions 21-30)


_ gf Bob Dylan was awarded the Noble prize in : |
A. Music B. Peace "Ch iterature.....D. Physics E, Medicine
92. Which country has recently stopped using online encyclopedia “Wikipedia”?i; age
A. Russia B. China C. Iran D. France E. Turkey
93. The first Nano satellite launched recently by Bangladesh is named as
A. Bangabandhu satellite B. Grameen Kheya C. Mohakash bhela
D. Banglar doot E. BRAC Onnesha
24. ADP stands for:
A. Annual Development Plan B. Academic Development Policy
C. Annual Development Program D. Automated Development Plan
E. Annual Dividend Policy
25. The highest production of power (in MW) by BPDB till date is
A. 6987 B. 9471 C. 13179 D. 56879 E. 7895
26. Proposed Rooppur nuclear power station is located at:
A. Savar B. Rangpur _—__C. Pabna D. Gazipur E. Kushtia
27. Which team won the UEFA Champions League in 2016-17?
A. Atletico Madrid B, Real Madrid C. FC Barcelona
D. Manchester City E. Bayern Munich
28. Which of the following banks is a fourth generation Bank in Bangladesh?
A. NRB Bank B. Sonali Bank C. Prime Bank
D. Karmashangsthan Bank E. IFIC Bank
29. In the year 2016 Bangladesh exported goods worth US$
A. 6.32 Billion B. 38.6 Billion C. 2.1 Billion D, 30.65 Billion E. 16.65 Billion
30. In terms of world population USA is ranked
A.5* B. 10" c. 3" D. 16" E7
Section II — Logical Reasoning/Critical Analysis and Mathematics (Questions 31-55)
31. The most important determinant of success in life is education: Een children from broken
or dysfunctional homes tend to establish themselves as good citizens so long as they
obtain a high school education. On the other hand, children who fail to earn a Bachelor’s
degree are much less likely to avoid prison, welfare of divorce, Which of the following
statements most effectively strengthens the above argument?
A. Children from functional homes are more likely to attain Bachelor’s degree while

B. Most prisoners receive a Bachelor’s degree while in jail.


C. Research indicates that college graduates from abusive homes are more likely to be
arrested,
D. Individuals with heart problems are more likely to have post graduate education.
E. One recent study demonstrated a link between education and lifetime earnings.

EMBA# 70 |
553
——3017| ;
<ion on
dmissi Test July. rore it; opensns in cinema
+ - It iswy MBA (Evening) Program, DU A ts bad reviews “et director 18 given
we found that a certain direct
or’s latest jig ? vie ane cessful. What can be
t halls, Consequently, very ie, fOr
few people pore nsucce
much less money to make his h also became "
next movie, whic
ae from this scenario?
- This director makes terrible movies
: ovie.
B. The movie reviewers were right
about the first a ity.
C. Movie review
ers exert influence on the movie
qu: reviews.
D. The general Public does not pay attention to movie :
movie. udent evaluation of teachers is not a
&- The ordirector
33. Profess class another
told nothis make
Khan will that the method of st + questionnaires at the end of the
valid measure of teaching quality. Students should fill « i folloyyageniftEre., provides
semester when courses have been completed. Which 0 a 1
support of Professor Khan’s proposal?
A. Professor Khan received low ratings from his students. wi)
B. Students filled out questionnaires after the midterm exam.
C. Students are interested In teacher evaluation.
D. Teachers are not obligated to use the survey results.
E. Student evaluation of teachers is voluntary.
34. An economist was quoted as saying that the Cost-of-Livingpier CO
index (
month because of a recent increase in the price of fruit and vegetables.
following cannot be inferred from the statement?
A. The cost of fruits and vegetables has risen sharply.
B. Food cost changes are reflected quickly in the COL.
C. The cost of fruits and vegetables is a major item in the COL.
D. Consumers have decreased their consumption of fruits and vegetables.
E. Other items that make up the COL have not significantly decreased in price.
35 . Many of the convenience foods on the market today, like cereals, have
less nutrients than
natural foods, which were dominant a decade fi or two ago.
Many nutritionists’ claim that
dry cereal gives less nourishment than natural foods like eggs or
bacon. Opponents of the
nutritionists’ views state that examination of High-
school students show less nutritional
deficiency than in their parents’ time. Which of the follo
wing, if true, would tend to
strengthen the opponent’s view?
A. High school children reported eating no breakfast
at all.
B. Consumers are not likely to sacrifice convenience
for nutrition.
C. Adults claim to eat convenience foods as well
as natural foods.
D. Convenience foods can be digested just as
quickly as natural foods
E. Fewer convenience foods were available to the parents.
1
36. At the end of a business conference, 10 people who were pre
each other once. How many handshakes will there be alto
gether? Le ee ih
A. 20 B. 35 C. 45 D. 55 E90
37. Find the least number which leaves a remainder of 3 wh os
leaves no remainder when divided by 9? €n divided by 5, 6, 7 and 8, but
A. 1483 B. 1458 C. 1598 D. 1683
38. One fourth of a number is equ to eee d ofn another numberot if 50 ie
] arger nu mber, it becomes two times the secon lied to the
A. 715 B. 85 C. 95 Ma ieig e is the smallest number?
E. 125
554
Wieeeteeeteheeteee eee eS TeV.
)68F]6U™”™F™”™mUC

39. A retailer buys two commodities for Tk. 1400. He sells one of them at a profit of 20%
and the other at a loss of 8%. He makes neither profit nor of loss in the end. Find the
selling price of the commodity that he sold at a loss.
A. Tk. 1,000 B. Tk. 1,100 C. Tk. 800 D. Tk. 950 E. TK. 920
40. In the first 10 overs of a cricket game, the run rate was only 3.2. What should the run rate
be in the remaining 40 overs to reach the target of 282 runs?
A. 6.25 B.6.5 _ C. 6.75 D.7 E. 6.8
4), There are deer and peacocks in a zoo. By counting heads they are 80. The number of their
legs is 200. How many peacocks are there?
A. 20 B. 30 C. 108 D. 112 E. 116
A farmer built a fence around
: his square plot. He used 27 fence poles on each side of the
44 square. How many poles did he need altogether?
A. 100 B. 104 C. 50 D. 60 E. 70
43, X is 3 years older to Y and 3 years younger to Z, while Y and V are twins. How many
years older is Z to V?
A.2 B.4 C.6 D. 8 E. 10
44. On selling 20 pens at Tk. 384, there is a loss equal to the cost price of 4 pens. The cost
price of a pen is:
A. Tk. 34 B. Tk. 30 C. TK. 16 D. TK. 20 E. TK. 24
45. In an orchard, there are 10 rows and 12 columns of mango trees, The distance between
two trees is 2 meters and a distance of one meter is left from all sides of the boundary of
the orchard. The length of the orchard is:
A. 20 meters B.22 meters C.24meters D.26meters E. 30 meters
46. Working 5 hours a day, 39 persons can repair a road in 12 days. In how many days will
30 persons, working 6 hours a day, complete the work?
A. 10 B. 13 C.15 D. 20 E. 25
47. If the difference between the length and breadth of a rectangle is 23 meter and its
i is 206 meter, what is its area?
A. 2200 sq. meters B. 2220 sq. meters C. 2350 sq. meters
D. 2520 sq. meters E. 2740 sq. meters
48. The value ofx for which
47 — 2” = 12, is:
A.2 B.3 C.-1 D.—2 E.1
49. If 16(4*) = 1, find the value of a.

A.-2 B.0 C. 5 D.3 E.2

l ‘ x
what is the value of
50. Ifx+ —=4, ert
x x” —3x+1
A.4 B. 3 C:2 D.1 E.0
51. Ifx = 10, which of the following has the minimum value?
x 2, x 2X
A.2-x
- B. —5 Cc -: (2-—x)(2-x) E.
D. (2-—x)(2-x) +=
-52

2. If 2x+3Y _> then find x;y?


3x+2y 6
A.4:5 B.6:8 C.8:3 D.5:6 E.4:3

555
OC,

MBA (Evening) Pro DU Admission Test Jul 2017

53. IfN is, positive, ~ M is equal to which of the fo Jlowing?


MN = 9, then
= 4 and
1
A. — 2 3 D. 6
E. 1

. has an area ofpag


54. A circle 4x. If the radiusmegwere doubled, the new 2 ea of the circle would be
Ow many times the original area?
E. 6 a kane?
A.2 Bp C.4 } D.5
55. A wheel of a car of at 600 RPM. What is the speed of the car Mh
radius 21 ems is rotating ;
A. 79.2 km/hr B. 47.52 ki/hr C. 7.92 km/hr D. 39.6 km/hr
Section HI: Writing Ability Test (questions 56-60) oe
(Write answers in the space provided after each question, no exér® P&PCr [5]
56. Translate the following sentences into E h.
A. WHIMGE af wie aeeane ee
B. Cfrt Ore HT STS HCE |
C.F OR SR FCT SEPT
D. “88 QS GET TT SF ANC |
E. SOR a $51 PSR Hos GTS BITS | [5]
57. Translate the following sentences into Bangla.
A. The weather will turn rough in the next twenty four hours. al ot
B. The girl surprised her parents with top performances in her examinations.
C. The thief had been planning to enter the house through the kitchen window.
D. Aren’t we wasting our time with too much video watching?
E. Nobody talks to him because he misbehaved with an elderly person.
58. Define/Explain (in English) the meaning of the following terms in 1-2 sentences; [5]
A. Eco tourism B. Black money C. Value added tax
D. ATM machine E. Export Processing Zone
[5]
59. Make a précis of the following passage and provide a title
Prevention is betthan
ter cure. It is recognized that the only way to get rid of malaria
complet
ely is to get rid of the mosquitoes which cause it. Malaria is always associated with
damp and marshy land. This is not because the land is damp, but. because to the Stagnant
water which forms in pools, drains and abandoned tires etc. where mosquitoes breed.
Mosq
begin
ui their to
life as larva
es in the stagnant water. Mala
does not
ri general
aly occur in
dry desert countries because mosquitoes cannot breed there. The only way we can be safe
ei uitoes is by preventing their breeding. This can be done if Stagnant water
is not
allowed isto tofrom
put aanywhe re,kerosen
film of and eby oildrainin g suchandwater
on pools drainsregularl y. byAnothe r way to ing
preventof
which, dep rivthe larva
air, kills them. a
= Qt
test:
!
Title: es.

Précis:
200 words on ‘The | mportance of .
_ Write a short essay in about
F Development and Capacity Building for a growing Bangladesh,’ uman_ Resource
[S}
556
orate
(E VE NI NG ) Pr og ra m (3 8-8 0°)
MBA ring-2018
Page # 572
DU, Admission Test Sp
INSTRUCTIONS
tes
s tion I and sect ion II) + 35 mi x (S ec ti on III) = 90 Minu
Test Duration : 55 min (Sec
= 80 Marks
Total Marks —: 55 (Section I and section II) + 25 (Section III)
The test is divided into Three sections:
; -30)
Section I— English Language & Current Affairs (Questions l ions 31-55
)
Section II — Logical Reasoning, Critical Analysis and Mathematics (Questions
Section III — Writing Ability Test (Questions 56-60)
Each question of the MCQ part carries one mark
For every wrong answer to a MCQ .25 mark will be deducted
Writing Ability test contains 5 questions carrying 5 marks for each (5 x 5 =25).
only in the space left after it; no additional paper will be supplied.
Write answer to a question
© Admit Card and photograph of each candidate would be checked
© No one will be allowed to sit for the test without admit card
e Use of calculator is not permitted
© Carrying of the script
a mobile phone or similar device during the exam will cause cancellation
e Check the question and answer script before start of the examination. It is advised to
change the script in case the printing is not clear or any blank page is found.

Section I: English Language (Questions 1-20) and Current Affairs (Questions 21-30)
Read the following passage and answer the questions that follow (Questions 1-5)
Photography is a series of actions involving light or electromagnetic radiation to record
images of objects on various surfaces. Photography allows people to communicate what is
important to them, helps to preserve history, facilitates communication enables le to
become artists and moves people in ways that words sometimes cannot In the i@be
eos
the first photographic experiments began, the field of photography seemed
stable oe to
define. A photograph was simply an image captured on film w;
with a camera, and
were described as a means of capturing or freezing ‘ ‘real life.” ; photographs .
rae
digital photography and graphic design software, Significant rea
inBeg ene .

forms. New methods of sending, posting, sharing, and al


media have made photography an ever more boric oy ae through interconnected
Changing field.

366
1, The passage is about: ae
ol.
A. The importance of photography in human life
B. The origin of photography C. The definition of photography
D, Photography and its evolution
02, Why is photography sometimes greater than words?
A. Words are often misleading B. It has a direct effect on human emotions
C. Words are never documented D. It is an easier communicative skill
3. In the past, the field of photography was stable because:
A. Photographs were easily captured
B. The manual cameras were not complicated
C. Photographs represented life in its true form
D. Photographs were the easiest forms of communication
04. The antonym of ‘digital’ is:
A. electrical B. mathematical __C. electronic D. analogue
05. The word ‘portable’ in the passage is a/an:
A. noun B. adverb C. adjective D. verb
Choose the appropriate preposition (questions 6-8):
06. We worked Monday to Wednesday. 99
A. on B. from C. at D. over
07. We walked the beach for two miles. en
A. along B. under C. at D. above
08. He is the CEO of a company that deals pharmaceutical products.
A. at B. in C. with ~ D. of
Choose the alternative that best reptaces the underlined pprtien of the sentences smi
(questions 9-10):
09. A sharp fall in prices ofjute have led the poor jute farmers to the brink of starvation.
A. has led the poor jute farmers. : B. have had led the poor jute farmers
C. are leading the poor jute farmers’ | ~D. were leading the poor jute farmers
10. Mr. Raihan, the president of the union and who is also a member of the community
group, will lead the delegation:
A. since he is amember of the‘community Group: ibe!
B. also being a member of the community group
C.a member of the community group |
D. in addition, who is a member of the community group
Select the appropriate word / words (Q: 11-12)
11. Our culture inspires us to take __ in our heritage.
A. proud B. prides C, pride D. proudly ©
12. He said I use his bike whenever I wanted. :
_ A. could B. can C. will D. would
Choose the correct form of the verb (Q;.13-14)..
13. She often shopping on the weekend. |
A. goes B. going C. to D. go

$67
14. My sister as a pilot. C KS
l A. is works B. work delicate
5. A ynonym of “intelligent” is ; : ‘ a
A. cleve ae c. imbecile eserved
16 An . B. duffer .
- n antonym of ‘hostile’ is : D.F
A. unsympathetic B. friendly C. inhospitat
a the correct sentence (questions 17-20)
- A. Neither Nasima or Krishna can sing well.
B. Neither Nasima nor Krishna can sing well.
C. Neither Nasima and Krishna can sing well.
a D. Both of Nasima and Krishna cannot sing well.
_ A. 1 am tired as I am working on 7 in the morning.
B. I am tired as I was working since 7 in the moming. _
: am tired as I have been working since 7 a aaa
- Lam tire
as I dhad been working sinc
7 ein the m oj
19. A. My mother is einen ser B. My siiothes areca
C. My mother is confined into bed. D. My mother 1s: ot.a prized
20. A. A few of the three boys got a prize. B. Each of the three boys £
C. Every of the three boys got a prized D. Few of the three boys g0°
Current Affairs (Questions 21-30)
21. What is the name of the severe hurricane that hit the United States in September 2017?
A. Katrina B. Irma ~ ~C. Maria... , D. Wilma
22. How much is 1 ounce of gold in grams?
A. 23.3 POTS FP Sas: Fer ee Bary Es.t
23. To the police of which country did the leader of the Catalonian independence movement
hand himself over in November 2017? s f svar S
A. Italy B. Spain C. Belgium... D.. France
24. The President of which country.was impeached this:year on grounds of cosruption and
immoral collusion with private entities? - A
_ A. The Philippines —_B. Myanmar
C. Indonesia _ D. Republic
assumed office of K:orea
this
from after President Trump
25. Which trade bloc the US has withdrawn _—
A. TPP B, NAFTA C. APTA.
26. Which one among the following is working to aid the Rohingyas flee; Pot
A. UNDP BWHO ...C, ioe Aecing Myanmar?
in Economics in 20179 ~UNESCQ
27. Who is the winner of the me Prize bh i
A. Michael Roshash B. ICAN C Doomedese
APEC has is: ay Clock D. Richard Thaler
28. The number of member countries
‘3... B, 23 C2 pe
29. Entomology is the science that studies SE Rae | |
A. Human behavior __B. Insects
4Summer Olyipi
30. Which city will host the B.202Tok s ) siaestomy D. For mat ion3 7
of4
rock s
yo C. Los Aiigét PIDVORT
A. Paris

* 368
<M... ©*MEILIDDDIVIY

section Il ~ Logical Reasoning/Critical Analysis and Mathematics (Questions


31-55)
31, Of those trig who typically work 8 hours at the office each day but sometimes say
that they will work at home on a particular day, 25% actually work less than an hour and
ore Se See Same employees believe that they are more productive working at
home than working in their office. If this is true, which of the following conclusions
about office employees best supports the statement?
A. He employees working at home for a day work fewer hours than those who work in
omice.
B. 10% employees are less productive working at home than those working in their office.
C. At ont 15% employees do not define productivity exclusively in terms of hours
D. At least 15% employees can complete an amount of work at home in 1 hour, which
they do in office in 8 hours.
32. Most students graduating from top high schools and having an intelligence quotient (IQ)
over 120, as well as all students with IQ over 150, when they apply, succeed in getting
admission in at least one of the departments of the University of Dhaka. If this is true,
which of the following best supports the above information?
A. Every top high school graduate with an IQ 150 gets admission in the University
of Dhaka.
B. A high school graduate with IQ less than 100 could not have. been a student at a top
high school. )
C. It is possible
that a student with IQ 130. and attending
the University of Dhaka is a top
high school graduate. !
D. At least one top high school graduate who applies for ‘admission is a student ef the
University of Dhaka.
33. According to some opinion, the increase in the number of newspaper articles as fabrication
serves to bolster the contention that publishers are more interested in boosting circulation
than in printingthe truth because even minor publications have staffs to check such obvious
fraud. Which.
of the following may be the assumption
of these who hold such opimon?
A. Newspaper stories exposed as fabrications are a recent phenomenon.
B. Everything a newspaper prints must be factually verifiable.
C. Fact checking is more comprehensive for minor publications.
D. The publishersof newspapers are the people who decide what to print im thear newspapers.
34. In the past, consumers would rarely walk into an ice cream store and order low-fat ice
cfeams. But today, an increasing health consciousness, combined with a much bigger
selection of tasty low-fat foods in all categories, has made low-fat ice cream a very
profitable item for ice cream store owners, This paragraph best supports the statement that -
A. Low-fat ice creams produce more revenue than other low-fat foods.
B. Ice cream store owners would be better off selling only low-fat ice cream.
C. Ice cream store owners no longer think that low-fat ice cream is an unpopular item.
_. D. Low-fat ice creams are more popular than other kinds of ice creams.
35. Fill in the blank in the series SCD, TEF, UGH, . WKL by choosing an option.
A. CMN « B. VIJ C. UHG | DUTs Sent

[EMBA #72 _
n;

MBA (Evening) Pro 3 5) Kb


s n the printed
Mathematics (Question® K at 10% discoun
36. A shopkeeper eams a profit of 12% on selling @ orn book 1s: 7:56
Price. The ratio of the cost price and the printed PUC® i D. 47°
A. 45:51 : B. P45:56 C. 47:51
os ence poles on eac h side of the
a7. A ae built a fence around his square ee used 27 i
208
Square.
Per How many Y po polesdies Be need altoge
altog C. 104 children D.are illiterate.
“ang a
38. Ina city, 40% of the adults are illiterate while 85% of the Ot ation is literate
ratio of the adults to children is 2:3, then what percent of D. 75%
A. 30% B. 33% ° C. 25% 9 a factory is Tk. 600. Ifa
39. The average weekly salary of 12 workers and 3 managers a er, then the average salary
manager whose salary is Tk. 720 is replaced with a new zat
of the team falls to Tk. 580. What is the salary of the new MANBE") Te 499
A. TK. 570 B. TK. 690 C. Tk, 640 f ‘xed nuts which is one-
40. In one box of mixed nuts, 30% are peanuts. In another " emptied into the same
half the size of the first one, 40% are peanuts. If both boxes are
bowl, what percentage of the mixed nuts in the bowl are PT 33.33%
A. 16.67% B. 20% C. 25% 7-3. then what is the
item is 2:3,
If the ratio of previous price and current price of an
41.
percentage increase compared to the previous price? .
A. 25% B. 33.33% C. 50% D. 66% .
42. An item is sold at a loss of 12%. If the item can be sold at Tk. 1,200 more, then there is a
profit of 8%. What is the cost price of the item? :
A. Tk. 5,000 B. Tk. 5,500 © -'C:'Tk. 6,000" ° D.'Tk. 6,500
43. A person makes a profit of 10% on 25% of the quantity and: a:loss of 20%. on the rest.
What is the percentage gain or loss on,the whole? He onoirrin
A. 10% B. 12.5% — C.-12% _ _ D.25%
How many times does the digit ‘4’ come to write numbers from 10 to 100?
A. 10 B. 11 CL Tat tered FUL Eerie
45. The population of Bangladesh is estimated at 1,60,000;000. Which one below is the
correct standard expression of the number?. :;
A. 16 x 10° B.1.6% 10’... 16x10"... D.1.6x 108
A fraction becomes 4 when 1 is added to both the numerator and
the
" becomes 7 when | is subtracted from both the humerator and thé denom inator and it
denominator The
numerator of the fraction is:
A.2 B.3 C.7 ) D.15 cB
47. 729 ml of mixture contains milk and water in the ration 7:2, How much
be added to get a new mixture containing milk and water in the rati “ RRC WBEIS to
much capital eapital would
How much
48. How wo yout stat
be Tk. 1040 inin 1515 years si D.90
; imi
A. Tk. 850 Bk 750 C. Th 650" annual
rate of interest of 4%?
49. Ifx =3 is a.s00k af tha aquetine 2 +kx+ 60, find the. value 2 $50) vo! »
A. -8 praia” C. 6 :
rornad.
6
seals Ny
50. The speed of a boat is 10 km per hour (kph) and th
boat travelled 5 hours in favor of the current and then antes ourent is 5 kph. The
was the time to return? the starting point. What
A. 12.5 hours B.10hours! > © 45ra
houtsy ;
fx=-1, then =
$1.1 x-1 '
1 1 9
A- > Bar c.-= D 7
. 1 1 3
52. Solve the the equation
eq idietternseeSe forx

1 l 1 1
Aa B. se D.
oa Ra Try 2-4
53. A rectangular park has a length of 620 meters and breadth of 480 meters. If a path of 4
meters wide passes around the park, what is the area of the path in square meters?
A. 4,416 B. 8,864 C. 8,800 D. 8,736
54. The length of room is 1.5 times of its breadth. If the area of the room is 216 square
meters, what is the perimeter of the room?
A. 60 B. 54 C. 48 D. 42
55. An angle which is less than 360° and larger than 180° is classified as:
A. acute angle B. reflex angle C. obtuse angle D. adjacent angle
Section III: Writing Ability Test (questions 56-60)
(Write answers in the space provided after each question, no extra paper will be supplied)
56. Translate the following sentences into English. S]
57. Translate the following sentences into Bangla. -_. S]
58. Define/Explain (in English) the meaning of the following terms in 1-2 sentences; [5]
A. Blue Economy B. Micro Credit C. Black money D.CSR E. Infrastructure
59. Make a précis of the following passage and provide a title [Sl
Trees give shade for the benefit of others, and while they themselves stand m the sun
and endure the scorching heat, they produce the fruit of. which others profit. The
character of good men is like that of trees. What is the use of this perishable body if no
use is made of it for the benefit of mankind? Sandalwood, the more it is rubbed, the
more scent does it yield. Sugarcane, the more it is peeled and cut up into pieces, the
more juice does it produce. The men who are noble at heart do not lose their qualities
even in losing their lives. What matters whether men praise them or not? What
difference does it make whether they die at this moment or whether lives are prolonged?
Happen what may, those who tread in the right path will not set foot in any other. Life
itself is unprofitable to a man who does not live for others. To live for the mere sake of
living one’s life is to live the life of dog and crows. Those who lay down their lives for
the sake of others will assuredly dwell forever in a world of bliss.
Title:
Précis;
60. Write a short essay in about 250 words on ‘Refugee crisis in Bangladesh.’ (5]
Note: 56 & 57 = Question-4 translation #4" Wey 5-f 47H Sentence at zrar facet; eet Collection
a HUT MAA OMT OMT TIAA a

571
“@ \ Program (39-84 0)
MBA(EVENING)n Test
Se ae _DU, Admissio Summer-2018 fy Page # 587

Affairs (Questions 21-30)


Section I: English Language (Questions 1-20) & Current
Read the following paragraph and answer the questions that follow (questions 1-5):
The government of Bangladesh has again decided to give funds for recapitalization of state-
owned banks (SoBs) amid criticism from economists about throwing public money ‘down the
drain’. The decision was taken in a meeting recently convened by the Financial Institutions
Division of the Ministry of Finance, which was attended by Managing Directors of SoBs and
representatives of Bangladesh Bank. Until last September, the state-run commercial ands
specialized banks had a combined capital shortage of nearly Taka 150 billion, which, officials
say, has increased to Taka 200 billion by now. The civil society observes that banks are not
making any attempt to recover the loans and the money misappropriated from the banks. The
bankers depend on easy money from the public exchequer without performing their duties
properly. The civil society opines that the government should take a tough stand agaist these
bad practices and instruct the bankers to recover the bad loans before they can expect any
help from the exchequer.
01. An appropriate title of the passage is:
A. Good money drives away bad money from circulation
B. Capital shortfall and the bankers’ role in recapitalization
C. The government should not help commercial banks
D. Recapitalization of state-owned banks
02. According to the civil society .
A. The government is doing everything to recover bad loans
B. The banks are indifference to the task of recovering loans
C. The government should help bankers to perform their duties properly
D. Bankers are looting the banks
03. Until 2017, state owned commercial banks had a shortfall of approximately Taka
A. 200 billion B. 180 billion C. 150 billion D. 190 billion
04. It is observed that about giving loans, the public banks are
A. Very much eager B. Not that much interested
C. Closely following Bangladesh Bank’ s:instructions
D. Eager to follow World bank’s guidelines
05. About recovery of loans the public banks seem to be
A. Following Bangladesh Bank’s instructions
B. Very much efficient ;
C. Creating a good image in the society
D. Not interested

581
| MB A (Evening) Program, DU AGmussion Test Summer, 2018]

:
Choose the alternatives that best replace the underlined porti ons of the sentences below
(questions 6-7)
06. Having the best record for attendance, a school awarded to him.
A. the school awarded him a medal.
medal was
B. he was awarded a medal by the school.
C. the school awarded a medal to him.
D. a medal was awarded to him
by the school.
07. The Chairman of the board of dire that this year’s
ctors have decided that this year 's
profits wil
Dros
for rese arch.
its will be useq
A. have decided on using this year’s profits for research.
B. has decided on using this year’s profits for researching. be
C. has decided that this year’s profits will be used for researen.
D. has decided that this year’s profits will be used researching.
Choose the appropriate preposition
(questions 8-10):
08. The gift was wrapped blue paper.
A. by B. on ound
09. Please look C. in ot
the issues urgently.
A. in B. on -
10. He can play all the musical instru C.out rine
ments the violin.
A. without B. except C. besides D. over
Identify the correct sentence
(questions 11-12)
11. A. One of my friends is a lawyer. ;
B. One of my friend is: a lawyer.
C. One of my friends are a lawyer. D. One of my friends are lawyers.
12. A. It had been raining all day today.
B. It rains all day today.
C. It was raining all day today.
D. It has been raining all day today.
Choose the correct form of verb
(questions 13-16)
13. I don’t know who the chair.
A. breaks ~ B. broke
14, My mother _____ TV news every C. breaking D. break
night.
15. ay rn fiat diene his bisa Gees
1. iam Chip ee Seating
meer

A. see B. seen Cc. seeing


Devise
Dawes
Select the appropriate word
(questions 17-18)
17. He is not the person to take your
failure
A. kindly B. mistakenly
18. Tamim was C ti
as he raced to a hundred in 60 bale
A. unable D. deeply
B. unstoppable
unplayable C.
Select the right synonym
of ‘hostile’ (question 19) D. unpredictabl
19. A. flexible B. belligerent Pea
C. deceiteil
D. happy
582
Sete
select the right antonym of ‘cowardly’ (qubatien 20)
96, 1 A COMERS B. panicky C. daring D, craven
Current Affairs (Questions
21-30)
ZA Which country is to deploy driver-less buses from 2022?
A. USA B. Japan C. Singapore D. South Korea
22. Who is the promoter of Chinese ‘One Belt One Road’ initiative?
A. Xi Jinping B. Chiang Kaisak C. Shinzo Abe D. Man Shetung
2: Recently which internatio nal body has decided to put sanctions against Myanmar generals?
A. United Nations _B. European Union C. SAARC D. UNDP
24. Which country has lifted restrictions on direct transport of
air cargo from Bangladesh?
A. United States B. United Kingdom —_C. Japan D. Australia
25: Who has received posthumous Ekushey Padak in 20187
A. Humayun Ahmed B. Humayun Azad — C, Humayun Kabir _D. Humayun Faridi
26. Who is the current President of India?
A. APJ Abul Kalam B. Pranab Mukharjee_ C. Pratibha Patil D, Ram Nath Kobind
27. What is the time line of achieving Sustainable Development Goals (SDGs)?
A. 2016-2030 B. 2016-2025 C. 2016-2040 D. 2016-2020
28. Which country is going to introduce its own ‘Encyclopedia’ by 2018?
A. Singapore B. Japan C. China D. India
The 2018 Winter Olympic was held in
A. Pyongyong, North Korea B. PyoingPang, Vietnam
C. PyeongChang, South Korea D. Peiking, China
What is the position of Bangladesh in terms of sending troops to the UN Peacekeeping
Force?
A. First B. Second . C. Third D. Fourth

Section II: Logical Reasoning / Critical Analysis (31-35) & Mathematics (36-55)
31. The cost of producing radios in country H is ten percent less than the cost of producing
radios in country T. Even after transportation fees and tariff charges are added, it is still
cheaper for a company to import radios from country H to country T than to produce
radios in country T. The statement above, if true, best support which of the following
assertions?
A. Labour costs in country H are ten percent below those in country T.
B. Importing radios from country H to T will eliminate 10% of the manufacturing jobs
in country T.
C. The tariff on a radio imported from country H to country T is less than 10% of the
cost of manufacturing radio in country T.
D. The fee for transporting a radio from country H to country T is more than 10% of
the cost of manufacturing radio in country H.

583
r
| Pac care 2018
— Summet
Admission Test wv to COV er services : required by
MBA (Evening) Program, DU
32. Company X is considering issuing a ne w insurance P ie its people. Premiums for the
that afflict Company X is concerned
elderly people who suffer from diseases Therefore, .

policy must be low enough to attract customers. ficient t0 pay for the nena that
that the income from the policies would not i" wt ould be most likely to minimize
would be made. Which of the following strategies W
Company X’s losses on the policies? eral: efits for many years,
A. iano middle age ee unlikely to submit anal diseases as children.
B. Insuring only those individuals who do not suffer a e are included in other policies
C. Include a greater number of services in the policy
of lower cost. mpanies for similar policies.
D. Insuri only ng
those individwho uals
were ene The number of cases
33. i
A rare disense is being
i i i
diagnosed i
witht h increasing
in aahequency. ‘our years ago. The
reported this year is more than double the num eported ; ;
ent and prevention of the disease.
government should now allocate more funds for treatm
, 28
All of the following, if true, would weaken the conclusion except
currently under-util -
A. funds already available for research in the disease are
se at a considerably
B. a new test employed for the first time this year detects the disea
earlier stage in the development of the disease. a
C. the number of cases reported this year represents the same fraction of the population
as reported in all of the last five years.
D. acommittee of experts reviewed the funding four years ago.
A financial consultant who prepared _a feasibility report for an investor was asked to
explain a particularly ambiguous proposition of the report. He responded, "when I wrote
that particular proposition, only Allah and I knew the meaning of that. Now only Allah
knows". What is the point of the consultant's response
A. Most investors are not able to understand feasibility reports
B. Allah is infinitely wiser than man.
C. The consultant has forgotten what he had originally meant by the particular proposition.
D. Consultants rarely know the source of their own creative inspiration.
35. Mr. X has twenty years of data entry experience: thus if you
are looking for an efficient
data entry operator, you don't need to look for other
s. Which of
assumptions make the statement valid?
cm ets
A. Twenty years of experience ensures data entry
efficiency
B. The required knowledge is identical to what Mr. X has
|
C. Mr. Xis an outstandingly fast and accurate data entry ‘
rhe
D. Mr. X seems to be a good fit for the company, foqunan a!

584
Mathematics (Questions 36-55)"
Pt ete.
36. By selling a bicycle for BDT 2,850, a shopkeeper gains 14%. If the profit is reduced to tt (
8%, then the selling price will be:
A. BDT 2,500 B. BDT 2,600 C. BDT 2,700 D. BDT 2,900
37. Naizel is twenty years older than Neketa. In three years, Naizel will be twice as old as
Neketa will be. How old is Neketa now?
A. 24 B. 17 C. 16 D. 8 |
38. If the price of the commodity is increased by 50%. By what fraction must its |
consumption be reduced so as to keep the same expenditure on its consumption? |
3 2 1 1
A. aa B. 3 Cc 3 D. 4

39. Inrecent survey of the students of a public university, it is found that 65% students are good in
Mathematics and 45% students are good in Statistics. How many students are good in both
Mathematics and Statistics of the public university?
A. 20 B. 59 C 15% D. 10%
40. A wall 8m long, 6m high and 22.5 cm thick is made up of bricks, each measuring 25
emx11.25cm x 6cm. The number of bricks required is:
A. 7,200 B. 6,400 C 6,000 D. 5,600
41. The length of a rectangle is increased by 25%. By what percentage should the width be
decreased so that the area of the rectangle remains unchanged?
A. 30% B. 25% C. 20% D. 35%
42. Inarow of trees, a tree is 7" from the left and 14" from the.right end. How many trees
are there in the row?
A119 B. 18 €. 21 ¢; . D. 20
43. The average age of 12' children is 15 years. If another child comes, the average age
comes to 13. What is the age of the new child? .
A. 11 years B. 7 years 8 CO'years) D-S years
44. A train 150 meter long and running at speed of 60 Sere eer 30 secondsto cross
a bridge. What is the lengthof the bridge? . ) . . 5:,.
A. 450 meter B. 500 meter Cc 350 meter D. 650 meter
45. A dishonest shopkeeper professes to sell ghee at his cost, price. But he uses a false
defective weight machine which reads of 950g for a kilogram. The gain of shopkeeper
in percentage is:
A 5.26 B.4 Crs D.45
46. A husband and wife have six married sons. Each of them has four children. The total
number in the family is:
A. 34 B, 32 C. 38 D. 36

[EMBA #74 wes


MBA (Evening) Program, DU Admi ission Test Summ er, 2018
full. What is the
47. A gas tank is th full and requires 32 gallons more to make it>

capacity of the tank esta gun D.145¢


: allons
A. 120 gallons B. 140 gallons
“g
48. A family had provision of food for 15 days. After 5 day: s 6 gu guests came and the
Provision lasted 6 days. How many are the members of the family
ily? ae
A. 10 B. 11 C12
49. Find the value of n, if 27" ad3 = 243,
A.2 B. 3
n't
C. 4 J
Determine the value of ae
3y »when
x = 2a and y=- a.
Sx+2y
8 5 13
A. —
13 B. — p=°
8 Cc. —
8
i 54+ 5345545345329 5
: :
A. 2575 B. 253 C525
One factor of x” ~ y+ 2y— 1 is x + y— D.5”
1. Another factor is:
A.xty-1 B.x-y-1_ “Cx-ytl 9 DP xtyt]
53. Simplify the 1 el ]
oe expression ——_ 1-4/1 ]
x+2 x-2 x4]
A.0 6x" —12 |
B. Les : ; ne Chih a Ds AOe De eV
Ix +1) 2) (x +2)
How many degrees are included
between hands of a clock at.4:00?
A. 50° B, 60° roe & 759 . ols) De 120°.
55. If the difference between
the circumference and diameter of
radius of the circle is: a citcle is 60cm, then the
A. 7cm |
B.9cm C. 10cm - D.ldem
‘ O ENING) Program (40-84 15)
Admission Test: 27Juty, 2018

billion, according to data from


Bangladesh will benefit to some extent from China's retaliatory 25 percent duty
on imports of
US cotton as there will be a supply glut of the white fibre in the international markets, the
fellow — However, the trade expert is cautious about the negative impact of the trade war
a hate two largest economies as Bangladesh has a significant stake in the

01. The most appropriate title of the passage is:


A. Trade war between large economics.
B. China’s loss in the trade war with USA
C. The World is getting ready for a trade war
D. Bangladesh stands to gain from the US-China trade war.
02. According to the information given in the passage, out of the total apparel imports by the
USA in 2017, China alone supplied approximately:
A. 20 percent B. 33 percent C. 27 percent D. 10 percent
03. In the year 2017, Bangladesh’s share in the USA apparel import was approximately:
A. 10 percent B. 12 percent C.7 percent D. 15 percent
may lead to:
04. Imposition of 25 percent import duty by the Chinese government on US cotton
A. Short term gains for Bangladesh B. Long term gains for Bangladesh
C. Short term gains for USA D. Long-term gains for USA
lead to:
05. If trade war escalates between USA and China, it might
economy
A. Negative impacts on Bangladesh’s
imports
B. Positive impacts on Bangladesh’s
C.D PositPosive impacts on US economy
itive i on all economies
Ct — oe raw best replace the underlined portions of the sentences below:

Sauk li
beled
destroyed numerous building and many that the city had
to eve
become a permanent disaster Zone. B. led many to believe that
A. many were led to belief D. many had been led to believe
C. the belief this led to was that

| [EMBA #75 593


ne ht
[MBA (Evening) Program, DU Admission Test July, 2018
ke to the was very rude,
07, The shopkeeper, who was a kind m
A. who was a kind man and he spoke to boy and he
B. spoke to the boy kindly and the boy
C. akind man, spoke to the boy who
D. who was a kind man and spoke to the boy and was
Choose the appropriate prepositions (question 8-10)
08. It is too risky to generalize
_ one set of results.
A. off B. from C. in D. about
09. Sonia decided to set some time every day for free hand exercise.
A. in B. up C. aside D.on
10. Artists have been painting nature centuries.
A. since B. by C. after D. for
Complete the following sentences (questions 11-13)
11. It is high time we
A. changing out eating habits B. change our eating habits
C. began changes our eating habits D. changed our eating habits.
12. The man is
A. reports to be a spy B. reported to be a spy
C. seen reporting to be a spy D. has been reporting to be spy
13. The patient’s blood pressure was
A. so high to ignore B. much higher to ignore
C. too high to ignore D. more high to ignore
Choose the correct form of verb questions (14-15)
14. The host was the number of guests.
A. count B. counted C. counting D. counts
15. When the thief entered the house, it was Suman who the police.
A. alerts B. was alerting C. is alerting D. alerted
Choose the appropriate word (questions 16-18)
16. He her on her beautiful new dress.
A. accomplished B. complimented C. appreciate D. reminder
17. The building is not safe and must be down.
A. razed B. pulled C. put D. pushed
18. We not hurry, we have got plenty of time
A. should B. need C. would D. must
19. Select the right synonym of ‘Irrelevant’
A. Opposite B. Applicable C. Irresponsible D. Unrelated
20. Select the right antonym for ‘Abstract’
ces

A. Pictorial B. Obtuse . C..Conerete D. Epherent


Curr
21. Besides Bangladesh, which country recognizes Bangla as one of its official languages?
A. Ghana B. Congo C. Mali D. Sierra leone
22. How many venues Cup were selected for FIFA World 2018 in Russia?
& @A—0e

A.11 dB. C. 14 D.9

594
ee eee. 8

23. Proposed
od ‘Garmentsve SER
Industry Park’ in Bangladesh will be co nstructed at-
A. Gajaria, Munshiganj B. Baushia, Munshiganj
C. Tangi, Gazipur D.Valuka, Mymensingh
24. Where has the world’s biggest automated container terminal been set up-
A. Shenzen B. Shanghai C. Philadelphia D.Armenia
25. Who is the present UN commissioner of human rights?
A. Kafi Anan B. Abu Zayed C. Zeid Raad Al Hussein D.Ban Ki Moon
26. The free trade area comprising of Argentina, Brazil, Paraguay, Uruguay, and Venezuela is called:
A.CAFTA B. COMECON C. SACU D. MERCOSUR
27. ‘Black Money’ is —
A. counterfeit currency —_B. money earned though understand dials
C. money eared illegally D. increase on which payment of as is usually evident.
28. Which sector has got the highest allocation in the development budget of Bangladesh for
the fiscal year 2018-19?
A. Transport and Communication B. Education
C. Agriculture D. Public Health
29. Who invented the genomics of jute?
A. Maksudul Alam B. Kudrat E- Khoda C. Jagadish Chandra Bose D.Muhahmmad Alam
30. Where does the Rakhain tribe mainly live?
A. Rangamati B.Bandarban C.Patuakhali D.Bogura
Section I : Logical Reasoning / Critical Analysis (31-35) & Mathematics (36-55)
31. Stock analyst: “We believe Company A’s stock will appreciate at 35% a year for the next
5 to 7 years. Company A just became the leader in its industry and we expect its sales to
grow at 8% a year.”
Commentator: “But how can the stock’s price be expected to grow more quickly than the
company’s underlying sales?
Which of the following, if true, would best support the stock analyst’s prediction?
A. The company’s expenses will be declining over the next 5 to 10 years.
B. The company just won a patent on a new product.
C. Company A’s stock is currently overvalued by a significant amount.
D. The 5 to 7 year time frame is too long for anyone to accurately forecast.
32. Evidence shows that people who live in the Antarctic score higher on happiness surveys than
those who live in Florida. Which is the best conclusion that can be drawn from this data?
A. Floridians would be happier if they moved to the Antarctic.
B. People in colder climates are happier than those in warmer climates.
C. There are only happy people in the Antarctic.
D. Those in the Antarctic who scored high on a happiness survey probably like snow.
33. Country X imposes heavy tariffs on imported manufactured goods. Company Y has
determined that it could increase its profits in the long term by opening a factory in
Country X to manufacture the goods that it currently produces in its home country for
sale in Country X.
os Company Y's determination to be true, which of the following assumptions must also
true? )

595
ee
[MBA (Evening) Program, DU Aniston Test ly, 201 shee
A. Company Y will be able to obtain all the necessary permits 10 OP tory in
Country X.
B. Company Y currently produces no goods outside its home en Country X
C. A sustainable market for Company Y's goods currently ear rted goods
D. Company Y's home country does not impose tariffs on mi, lll :ngh aie of the
34. Some 80,000 immigrants were living in a certain country 1n 2016. ied ee ie Se
immigrants were not employed in professional occupations, many © 7 ae of the
instance, many of them were engineers and many of them were nurses. Very Se
immigrants were librarians, another professional occupation. ? .
From the information given above, it can be validly Concluded that, in 2016, in the
country described above,
A. most immigrants were either engineers or nurses
B. it is not the case that some of the nurses were immigrants
C. some of the engineers were immigrants pat ED
D. most of those not employed in professional occupations were immigrants
35. A rapidly changing technical environment in government is promoting greater reliance on
electronic mail (e-mail) systems. As this usage grows, there are increasing chances of
conflict between the users’ expectations of privacy and public access rights. In some
investigations, access to all e-mail, including those messages stored in archival files and
messages outside the scope of the investigation, has been sought and granted. In spite of
this, some people send messages through e-mail that would never be said face-to-face or
written formally.
From the information given above, it cannot be validly concluded that
A. some e-mail messages that have been requested as part of investigations have
contained messages that would never be said face-to-face
B. some messages that people would never say face-to-face are sent in e-mail messages
C. some e-mail messages have been requested as part of investigations
D. e-mail messages have not been exempted from investigations
Mathematics (Questions 36-55)
. On selling 30 pens at Tk. 800, there is a loss equal to the cost The cost
price of 5
price of a pen is: pane
A.Tk. 23 B. Tk. 25 C. Tk. 32 D.Tk. 52
37. The ratio of female to male students in a class is f: m. If the number o! i
are x then the number of male students are of female studen
A= B. f[ C — p mtx
f x f |
38. In the first 20 overs of a cricket game, the run rate was only 3.5. Ww
rate in the remaining 30 overs to reach the target of 289 runs What should be the run
AT B. 7.2 C.7,3 PB arg
39. A sum of Tk. 24,500 amounts to Tk. 34,300 in 5 years at a simple interest rate. What is
the rate of interest? et Sie.
A4% B. 5% C. 6% D.8%

‘596
ee
ee ew Ue

4o, A bus is travelling with 48 passengers. When it arrives at a stop, x passengers get off and
ton, At the next stop half the
Find the value ofx. FECTS 8¢t off and 7 get on. There are now 22
messengers
A.26 B, 21 : C25 D22
4, Mr.
meet Lee
a left idhis fortune to his 3 sons, 4 daughters and his wife. Each son received twice
Goch wont enettdt
h as each daughter. His wife receive S $ 6000, which as quarter of the total money.m

A.$1200 B. $2400 C.$ 3200 D.$ 3600


4, A train travelling on an average speeds of 45 km per hour from Dhaka to Chittagong and
returned to Dhaka from Chittagong on an average speed 36 km per hour. What was the
average speed of train over the whole journey?
A. 38 kin B. 60 km C. 56 km D. 40km
43. The price of a loaf of bread was increased by 20%. How many loaves can be purchased
now for the amount of money that could be bought for 300 loaves earlier?
A.240 B. 250 C. 280 D.320
44. Out of 80 children, 35% can play only cricket, 45% can play only table tennis and
remaining children can play both the games. In all, how many children can play cricket?
A.55 B. 44 C. 36 D.28
45. Aman spends 75% of his income. His income is increased by 20% and he increased is
expenses by 10%. His saving are increased by —
A.10% B. 15% C. 25% D.50%
46. If the price of the commodity is increased by 50%, by what fraction must its consumption
be reduced so as to keep the same expenditure on its consumption?
at4 B. =3 .
c+B=) p.23
47. Five bells begin to toll together and tool respectively at interval of 6, 5, 7, 10 and 12
seconds. How many times will they toll together in one hour excluding the one at start?
A. 7 times B. 8 times C. 9 times D. 11 times
48. Shafi has all his money in currency noted of same denomination. He has as mamy notes as
the denomination of each note. Which of the following is NOT the possible amount of
money he has in taka? wa
A.100 B. 400 C. 500 D.1000
49. 32 =2* thenx is equal to:
AS CET e eee eat ; | D.5
50. Ifx*~4x + 3 equals
0, then what is the’value of (x =2/7
A3 B.2 C1 DO
51. What
is S the value
value
of0 x,? i Leda
5 , x
ds. Ly
x zy 1.25

A. 1.25 , B15. dats rea D.3.25


of x’ = ae tt xh = = 727 ©
°2. Find the value
x re ‘
A.170 B, 155 C. 140 D.125

597
[MBA (Evening) Program, DU Admission Test July, 201 8
$3. Ifx = 10, which of the following
has the minimum value?
Cae a c2 D. (2 =x)(2-x)
3 x
54. A tree of height 4 meter casts a sh
adow of length 6.5 meter, What is the
castings shadow 26 mete height of a house
r long?
A.14 meter B. 17 meter
°°. Two triangle having same height with areas C. 15 meter D.16 meter
of the smaller triangle is 6 cm. What of 18 cm? and 32cm? respectively. If the base
A. 10.66 cm wo ul d be the base of the larger triangle?
B. 7em C. 8.66cm D. 9cm
| ay a) Program (41
» Admission Test: Spring 2019
Section I: reer Language (Questions 1-20) and Current Affairs
(Questions 21-30)
Read the following passage and answer the questions that follow (questions
1-5):
ern retail, which repl; it
year sae 2014 inpatdateaee nding ne Brocery shops,
on changing hes been
consumer
growing st 15 percenta
behaviour, said a top official
of Nielson, a leading global market researcher. In 2018 the total turnover is estimated to
be Tk 2,180 crore, up from Tk. 1,342 crore in 2014. The factors that have led to the
changes in consumer behaviour include the advent and growth of e-commerce
advancement of technology, rising health awareness and consciousness about qoality
products and services. To these can also be asked the case and advantage of choice
shoppers find in modem retail chains such as Swapno, Meena Bazar or Agora. Population
growth, increased urbanization and rising income, particularly the growing middle class
population in the country, have been driving the growth of consumerism in the country,
which is expected to be the 26" largest economy in the world by 2030. However, despite
the growth of modern retail, traditional trade still remains dominant, and will remain so
far quite some time to come.

1. The passage is about


A. Changing consumer behaviour B. Modern retail chains in Bangladesh
C. Growth of consumerism in the country D. Growth of moder retail in Bangladesh
=
—o ;
2. According to the passage.
retail trade.
A. Modern retail is more profitable than traditional
economy in the world by 2030.
B. Modem retail will make Bangladesh the 26" largest to modern
C. Traditional retailtrade in the country hasn’t lost in dominant position

1n the country.
D. Modern retail is a driving consumerism
as a factor driving modem retail?
3. Which of the following the passage doesn’t suggest
A. Online shopping
B. Increasing health consciousness among the people
C. Retail chains in the rural areas ,
ng
D. Growth of consumer spendi
meaning of the word “Turnover”?
+. Which of the following words is closest to the
A. Profit me sate

5 C.ee
Investment
prs siding
or depending
on ith
«2
nd
wi
lin fh
ee
passage
2D anowertngast |
ing on ’
mem sen" |
on
A. based
C. makitig adjustments to

605
MBA (Evening) Pro DU Admission Test Sprin, 2019

ropriate w uestions 6-1

6. My brother get me his essay. iin


A. to writing B. to write C. wrote D. writing
7. Ifyou listened carefully, you it.
A. would understand B. understand —__C. have understood D. sais
8. People who don’t like boys with long hair or girls riding bikes are * saitind
A.Confident — B. Considerate C. Conniving Dp Naas
9. When something tastes disgusting, it is: ;
A. Teny B. Awful C. Spicy D. Expensive
10. Road the on the screen. :
A. instructions _B. instruments C. implements D. instructors
Cc the ropriate itions (questions 11-13

11. Its his great affection his grandchildren.


A. in B. after. C. with | D. for
12. He is better doing the work. ;
A. of B. off C. in ~ D. with
13. At the moment, she is recovering her injuries.
A. from B. at C. of D. with
Choose the correct form of verb (questions 14-15)

14. He claimed that he had never seen my uncle, but I know that .
A. he did B. he had ~ C. he was D. he hasn’t
15. I to Nepal until and my friend went'there last summer.
A. have never been B. was never been C. had never been D. were never been
16. Which sentence is correct?
A. She lives by an apartment. B. She lives on an apartment.
C. She lives in an apartment, D. She lives at an apartment.
17. Which word is wrongly spelt?
A. Deceive B. Receive C. Receipt D. Recede
18. Choose the appropriate pair of words matching the relationship s
FANS : GALLERIES, P Suggested in the pair
A. Teams : Goalposts _ B. Referee : Decision
C, Audience : Seats D. Tickets : Counters
19. Which one is a synonym for ‘Hostile’?
A. Unfriendly _B. Friendly C. Distant DD;
20. Which one is an antonym for ‘Diligent’? Disgusting
A. Active B, Prudent C. Careful «Dy Lazy

606
ts SWS & Benes

ai. A.Which ! country holds the first position of Global Innovation Index 2018?
Switzerland _ B. Finland Denm
92. re total number - ams Processing eae in ides oat et tr
3 C.
93. GDP growth rate of Bangladesh during 20 7-4 8 fiscal year n- ,
A. 9.18% B. 7.86% C. 6.52% D. 5.5%
4, Who was the first Vice Chancellor of the University of Dhaka?
A. G.H Langley B. Walter A. Jenkins
ree,

C. Philip J. Harrog D. Ramesh C. Majumder


25. The Bangladesh Securities and Exchange Commission (BSEC) was established as the
regulator of the country’s capital market through the enactment of the
A. S.E.C Act 1971 B. SEC Act 1973
C. S.E.C Act 1983 D. SEC Act 1993
on
26. BRICS is the acronym for an associati of five major emerging national
s. hich
economic W s
countrie does “C” and “S” stand for?
n n
A. Cameroo and Somalia B. Cameroo and South Africa
C. China and Saudi Arabia D. China and South Afprica
nshi
27. Which country won the SAFF under 18. women’s Champio in 2018?
e s h
A. Bhutan B. India C. Banglad D. Nepal
’ d
28. World teachers day is observe on-
A. 8th August —B. 6th November C. Ist January D. 5th October
29. The following rocket launched Bangabandhu Satellite 1 into the orbit.
A. Falcon 9 B. Ariane 5 C. Epsilon 8 D. Corona A3
30. Bangladesh became a member of ‘Commonwealth’ in-
A. 1965 B. 1975 C. 1971 ! D. 1972

Section I — ical Reasoning / Critical Anal sis and Mathem tics . 31

31. In the exam, David has lower marks than Dechen, but has more than Soma
, However, Karen also has more marks than David. Who has the lowest mark?
A. Karen B. Sonia C. David D. Dechen
32. The committee on sexual discrimination, in the workplace has highlighted Supremo
Company as a chief offender. Of the twenty senior executives in the firm, only one
is a woman. And of the forty junior executives, only five are female. Supremo could
best defend itself against the charges by showing that,
A. Male and female executives at'the same level have the same qualifications
B. They pay the same salary to senior men and senior women
C. Ten times more men than women apply for jobs with the company
D. All job applicant who were rejected had fewer qualifications than those accepted

607
Sprin 2019 |
(Evening) Pro DU Admission T est
MBA
ut iq ue ial re
nancees
filoy arch
sework
ca e i cs, a bo
ena ble s emp to
33. As a result of changes ini cultural norms and
flex-time,
company is considering implementing they are present at the officebe
during any time ofthe day from any location as long iSOok a aekcele curvemntby reust

ies help
ah believes this nip
inthe off eusbantond prove
hea een goals: ieee total costs, increase pr oductivity,
ment?
ki
Piet wa e ta
S . if true, most weakens the argument of the firm’s: manage
ting viastat Mcenth te
A. Some new costs will arise as a result of telecommu
ne
B. A similar firm tried a version of flex-time and abando
nt and extensive in
C. The firm in question performs: work that requires freque
- «
. 4

person collaboration
D. The firm in question recently lost its most experienced research analyst due to
his ion of the firm’s poor work life balance
An iesiratieseeanetc is considering a new policy cover service required by senior
citizens who suffer item diseases that affect elderly people. The premium for such
policy cover must be sufficiently low and that income from the policy should be
sufficient to pay the claims. Which policy strategy will be the best?
A. Attract middle-aged customers
B. Insure only those people who did not suffer from any serious disease
as children.
C. Include more services than are included in other low-cost policies
D. Include only those people who were rejected by other companies for similar polices
35. Unlike the wholesale price of raw wool, the wholesale price of raw cotton has fallen
considerably in the last year: Thus, ‘although ‘the retail price of cotton
clothing at
retail clothing stores has not yet fallen, it will inevitably fall.
Which of the following, if true, most seriously weakens the argument above?
A. The cost of processing raw cotton for cloth has increased
during the last year.
B. The wholesale price of raw wool is typically higher
than that of the same volume
of raw cotton.
C. The ; oper
theatintou
g costs
: of the average retail clothing store have
ve remained cons
, tant
D. Changes in retail prices always lag behind
changes in wholesale prices.

Mathematics (36-55)
Ifa garment factory produces 120 units of
shirts the total production cost is 6000$
and if the factory produces 150 unit the total
Prod
uctio:
exists a linear relationship between the production aii atson cone oa a
produced. What number the factory should prod chaste
selling price of per unit shirt is 65$.
ls

A. 75 B. 65 C. 70
— ll
a

608
37. One side of a rectangle is 3cm shorter than the other side. If we increase the length
of each side by 1 cm, then the area of the rectangle will increase by 18 cm’, Find the
length of all the sides.
A. 12cm and 9cm B. 7cm and 4cm C. 11cm and 8cm D. 10cm and 7cm
38. Two planes leave the same airport at 1:00 pm. How many miles apart will they be at
3:00 pm if one travels directly north at 150 mph and other travels directly west at
200 mph?
A. 350 miles B. 500 miles C. 600 miles D. 100 miles
39. A shop has a “buy 2 and get | free” offer. Sakib has Tk. 300 with him; he needs to
buy 6 dolls for his daughter. If each of the doll costs Tk. 55, how much does he save?
A. 30 B. 45 C. 60 D. 80
40. The total age of father and son at present is 74 years. 10 years ago the ratio of their
age was 7:2. What will be the ratio of their age after 10 years?
A. 2:2 B. 31:16 C. 12:16 D. 8:11
41. The purchase price of an article is Tk. 48. In order to include 15% of cost for
overhead and to provide Tk. 12 profit, the mark up should be:
A. 14% B. 25% C. 30% D. 40%
seen

42. Lima can do a job in 6 hours. If Sima works with her, the job will be compli? m2 |
te

hours. How many minutes would it take Sima to do the job alone?
A. 70 minutes B. 95 minutes |. C. 180 minutes. D. 205 minutes
43. In ten years time a company produces 500000 units of a commodity. From the second
year, the company produces further 1000 units of the commodity in each year than the
previous year. What would be the quantity that the company should produce im the first
year?
A. 45500 B. 50000 g tobe 900 o: FZ A D. 40000
For—; if 3 is added to the numerator, the resulting
$ fraction is > while if 4 is added

to the denominator the resulting fraction is : _ What is the value of = ?


y
A. 5/16 B, 2/10 C. 1/8 D. 4/14
45. A women says, “If you reverse the digits of my own age, the figures represent my
husband’s age. He is, of course, senior to me and the difference between our ages is
one-eleventh of our sum.” What is the age of the women?
A. 23 years B. 34 years C. 45 years D. 54 years
A waiter’s salary consists of his salary and tips. During one month his tips were 5/4
of his salary. What fraction of his income came form tips?
A, 4/9 B, 5/4 C, 5/8 D. 5/9

[EMBA #77] éoe


- :
Sprin, 019
MBA (Evening) Pro DU Admission Test
r to C, while B and D are, twins. What
47. A is 3 years older to B and 3 years younge
about the age difference between C and D? D. 6
A.9 caste aie hich of the following
48. Ifx+5>2
and x —3 <7, the value of x must be between W
pairs of numbers. 3 and4
A.—3and10 B.2and7 C.-3and4 D, !
3n ent+2
gl : 5
49. ‘Simplify the following: SA ge
: 3n
Zz n 3n D; 2
A. =n B. 2.5 C.5.2 5
62".5"*2 4 Gr gt L (3 x 2y°2.5 x 357.95" 5 ” 57, 5 ee 3 (C)
49. Sr! 93" 33" 33" an

50. if Vm +Vm—5 =5, what is the value of m?


A.8 B.7 CoB Pro D.9
Si. If x ¥ 0, 2x
= Sy, and 3z
= 7x, what is the ratio of zto y?_~
A.2to21 B.3to5 C. 14 to 15 - - -*D.35t06
52. Let x: y = 3: 4 and x: z=6:5 then z:y=? 1
A. 5:3 B.6:7 i C.5240% ~ p.5:8
53. if the radius of circle is increased by 30%, then its area ‘is increased by
A. 50% B.70%° | C.69% + D: 40%
A rectangular box has a length of 8 cm, a See psndie: and ace oR Ree, ae i
the length of the diagonal of the box, in cm?»
A. 10 B. 10/2 C.12 D.10 V3
55. In the following figure, ASB is a quarter of a circle. PQRS is a rectangle with sides PQ
=8 and PS = 6. What is the length of the arc AQB?
A

P Q..
4S R 'B
A. 5x B. 8x C. 10 D. 152

610
MBA (EVENING) Progr es Page
Page nx
# 623
Admission Test: Summer (1 March,
irs (Questions 21-30)
Section I: English Language (Questions 1 -20) and Current Affa
: ):
stions 1-5
th e questions that follow (q ue
i
s other forms of aa
hieraceeand tion were effective and even
For years,
aa it was
a believed
aidaathatatbribery easing the right palms, so
necessary tools for doing business in developing countries. nfs "Research undertaken by
e thinking went, firms achieved a competitive advantage. os
ie World Bank ree and others ‘canta far from lubricating business cei baa
actually fuels the growth of excessive and discretionary regulations. Bribery; ae
5
on itself, ; layer upon layer of bureaucracy eager toto getget
producing in on the action
4. Open
official
markets cannot work behind closed doors. Both private. capital flows ee: Ge
development assistance are increasingly discriminating with regard to policy performance
and institutional integrity. Investors today have too many options, and they are better able to
move their money to where the risks of corruption are less pronounced. And official agencies
and development institutions are seeking retums on their aid investments in the form of
poverty reduction and social development — with the same rigor with which private investors
look for financial returns. Perceptions in donor countries that corruption in recipient countries
sends their aid assistance down a black hole is one of the greatest threats of future aid. Again,
it is the poor who suffer. The World Bank, and other multilateral organizations are fully
aware that despite continued vigilance and’ state of the ‘art’ auditing’ and investigative
measures, the projects: that they support are not immune from the pressures of corruption.
There is simply no way to fully isolate individual projects and program lending from fraud if
it is pervasive throughout the environment in which they function. This calls for continued
efforts on the part of the Bank to pursue and prosecute fraud wherever they find
it, while
simultaneously strengthening the institutional structures . that.. will ultimately help, stop
corruption at the same source. It will be a difficult, long term struggle. But
make no mistake
it is a winnable fight, and one that must be fought.
Ol. The passage suggests that bribery leads to all the following except
A. It is difficult to isolate a WB project from fraud if corruption is pervasive
in the environment.
B. The fight against corruption will not be an easy one
C. Firms gain competitive advantage through corruption.....,..
»,
D. Bribery encourages the growth of discretionary regulat
ions
02. The phrase "greasing the right palms" means
A. Lubricating the business channels B. Paying bribes to relevant persons
C. Having an efficient work force D. Taking neve ; :
03. The author of the passage is most likely SSOTY Precouticns agaist bribe
A. A social researcher B. An anti-corruption officer
C. A bureaucrat in a developing country D. A World Bank official

618
o4, The passage suggests that corruption can be stopped at its source by «
"A, Isolating projects and programs lending from fraud
B. Simply maintaining constant vigilance and investigative measures
C. Increasing transparency in government transactions
D. Strengthening institutional structures and rooting out fraud
05. The author's attitude towards the fight against corruption can be described as-
A. Positive B. Over-enthusiastic C. Fanciful D. Pessimistic

choose the appropriate preposition (questions 6-8):


06. She was warned____the danger.
A. with B. of C. at D. for
07, Adil is accustomed hard work.
A. to B. at C. in D. on
08. The editor spent a lot of time the manuscript.
A. at B. in C. on D. with
Select the appropriate word (questions 9-13):
09. Iwill be in the office 5 pm.
A. on B. for C. until D. since
10. I will arrive sometime 8 and 9 a.m.
A. in B. next to C. between ~ D.on
11. Abed spent the entire afternoon the phone.
A. in B. at C..on D. by
12. My best friend Mohammad is named his grandfather.
A. after B. to C. about \“. D.of
13. All vehicles should stop the red light.
A. at B. in C. on : D. upon
Choose the correct form of verb (Questions 14-15):
14. I to the commentary on an exciting cricket match on my way to work.
A. listening B. was listening C.. used to listen D. listen
15. The watch was very expensive, I could not it.
A. buy B. bought C. buying ! D. to buy
16. Choose the correct sentence:
A. I shall wait for you till you finish your class.
B. I shall wait till you shall finish your class.
C. I will wait for you till you shall finish your class.
D. I shall wait for you till you will finish your class.
'7. Which one is a synonym for “Admonition"
A. Praise B. Warning C, Deference D. Elation
18. Which one is an antonym for "Judicious"?
A. Wise B, Simple C. Callous D. Miserly
19. Which word is wrongly spelt?
A. Infereor B. Decadence C. Sojourn D. Cyanide

619
LMBA (Evening) Program, DU Admission Test: Summer, 2019)
if FIRE: HOT
20. Choose the appropriate pair of words matching the relationship in theD, pasSmoke : Hazy
A. Water: Depth B. Air: Flow C. Ice; Cold
Current Affairs (Questions 21-30)
ai. Who was the first Budget proposer of Bangladesh?
A. Justice Abu Sayeed Chowdhury B, M. Saifur Rahman
C. Tajuddin Ahmed D. Abul Maal A. Muhith
ze Human Rights Watch is an advoc. oup located in:
A. New Delhi B. New York one C: WashingtonDC D. Loxkion
23 Which country tops the 2019 International Intellectual Property (IP) Index?
A. Germany B. United States C. Denmark D. United Kingdom
24 . Where is the Headquarter of Asian Infrastructure Investment Bank located? —
A. India B, China C. Japan D. Thailand
25. The tallest building of the world is located at the ‘level in the desert’, that isin
A. Dubai B. Istambul C, Sahara D. Saudi Arabia
26. Which country is in the position of Global Innovation Index?
A. Finland B. Netherland C. Switzerland D. Denmark
27. Which of the following is a monetary policy instrument?
A. Open market operation B. Tax
C. Savings D. Investment
28. The targeted electricity production capacity of Rampal Power Plant is
A.1100 MW B. 1240 MW C. 1200 MW D. 1320 MW
29. What was the GDP growth rate of Bangladesh in 2017-18 fiscal year?
A. 7.86% B.7.21% C. 7.01% D. 6.98%
30. Which Metro Rail route is expected to open within a year?
A.2 B.6 C.1 D. 12

Section IL. Logical Reasoning/Critical analysis and Mathematics (Questions 31-55)

31. Leena has been visiting friends in Chittagong for the past two weeks. She is leaving
tomorrow early in the morning by plane. Most of her friends live fairly close to the
airport. Monir lives ten miles away. Faruk lives five miles away, Sumona, seven miles.
wale
Alex is further away than Faruk but closer than Sumona. Approximately how far
from the airport is Alex?
B. Seven miles C. Eight miles D. Six Miles
A. Nine miles
32. At the Cricket match, Henry was sitting in seat 253. Marla was Sitting to the right of
to the left
to the left of Henry was George. Ali was sitting
Henry in scat 254. In the seat
of George. Which seat is Ali sitting in?
A. 251 B, 252 C, 255 D. 256

620
The demand for android phone experienced a growth of 25% every year. Market
33. analysis predicts that this growth will continue in the near future. Which of the
following statements will strengthen the statement above?
A. There is increasing threat of fake android phones in the market which has shaken
customers confidence.

ene
p, A large segment of people in the country find android phones more affordable than

ae =
ree
ever before due to its decreasing price.
c. Android phone increases the burden of electronic wastes and is a threat to
environment
D. The government is planning to impose import duty on android phone.
34. 42-40-38-35—33—3 1-28-, — Which pair of numbers comes next
A. 25-22 B.26-23 C. 26-24 D. 25-23
35. Jamal's grade point average puts him in the top three of the graduating class of college
A. Noman is in the top ten of the same class. Ethica has the same grade pomt average as
Noman. Nancy has a lower grade paint average than Ethica
If the information above is true, which of the following must also be true?
A. Noman and Jamal both have a higher grade point average than Nancy.
B. Jamal has a higher grade point average than Ethica
C. Jamal has a higher grade point average than Nancy.
D. Noman has a higher grade point average than Nancy.

Mathematics (Questions 36-55)

A sum of money is to be distributed among A, B, C, D in the proportion of 5:2:4:3. IfC


gets Tk.100 more than D, what is B's share? ih
A. Tk.500 B. Tk. 1,500 C. Tk. 1,000 D. Tk. 2,000
37. A father said to his son," I was as old as you are now at the time of your birth”. If the
father's age is 38 years now, the son's age five years back was:
A. 14 years B. 19 years C. 33 years D. 38 years
38. 6 people meet for a business lunch shaking hands with each other once. How many
handshakes take place?
A. 30 B. 21 C. 18 D.15
39. In a sport club with 30 members, 17 play badminton and 19 play tennis and 2 do not
play either. How many members play both badminton and tennis?
A.8 B.9 C. 10 D. 11
25% of a certain number is 15 less than 30% of the same number. What is that number?
A. 600 B. 300 C. 750 D. 135
4]. A bicycle agent allows 25% discount on his advertised price and
then makes a profit of
20% on his outlay. What is the advertised price on which he gains Tk. 40? ;
A. Tk. 25 B. Tk. 320 C. Tk 220 D. Tk.180
#
Z

621
MBA (Ev eni ng)
ni ng ) Progra m,
Prog ram, D U Admissis ion Test; Summer 2019 | sabi
LMBA2 (E ve !
er
42. If two digit positive integer has its digits reversed, the resulting integ
Original by 27. By how much do the two digits ee D6
A.3 B. 4 ae :
43. A dealer buys 40 tables, all at the same price. He sells 30 of them at dang 25% and
the rest at a loss of 10%. The dealer percentage of profit on this transac 5305
A. 16.25% B. 16% C. 15.25% ee td 2Flables
The price of 10 chairs is equal to that of 4 tables. The price of 15 chairs
together is Tk 4,000. The total price of 12 chairs and 3 tables is: 6.00
A. Tk. 3,500 B. Tk. 3.750 C.Tk. 3,840 D.Th eae
45. Ifa man can swim downstream at 6 km/h, upstream at 2 km/h, his speed in still water is:
A. 4 km/h B.2 km/h C.3 km/h D. 2.5 km/h
ed
A servant was encourag for TK.6 00 along with golde n ring for a year. He served for 9
months and received the ring and Tk. 420. What is the price of the ring?
A. Tk.120 B. Tk.130 C.Tk.140 D. Tk. 150
1
47. (4+12)4 x (x +3)’=?
A.2 B.0 C.2@+3) Dax
What is the sum of the squares of the roots of the equation 2x” + x — 6 = 0?
A.6 B.8 C. 10 D. 12
49. If 2* = 32, then x’ is equal to:
A.5 B. 10 C. 25 D. 100
Which of the following inequalities is the solution to the inequality (7x5) < (2x + 18)?
hice Rak Boss io
5 ; 9
S1. If mx + ny = 12my, and my = 0, then eo is equal to.
ym.
A. 12 B. 12mn Col2mn+12y. Dmx + ny
52. What is the angle that is half of its own complement? F
A. 30° B. 45° cea *! D. 90°
53. If x’ — 3x+ 1 =O, then the value of x’+ age
‘ x
A9 B.11 Cc. 18 D.7
A floor is covered by 600 tiles which are 10 cm by 10cm. How many 20cm by 20cm
tiles are needed to cover the same floor?
A. 120 B. 150 C. 180 D. 300
55. O is the center of the circle ABC. If ZOAB = 30° and ZOCB = 20° then find out the
value of x and y. -

A. 100° and 50° BB, 130° and 20° C. 50° and 130° _D. 50° and 100°

622)
Uh, MBA (EVENING) Program (44-87
*
Cy ak aay =rb) Ag (warty
Ay
ER DU, Admission Test: Spring'2026 °" ba Page # 634

section 1: English Language (Questions 1-20) and Current Affairs (Questions 21-30)
Read the following passage and answer the questions that follow (Questions 1-5): |
Marie Curie was one of the most accomplished scientists in history. Together witt
husband, Pierre, she discovered radium, an clement widely acu re od
studied uranium and other radioactive substances. Pierre and Marie's amicable collaboration
later helped to unlock the secrets of the atom. Marie was born in 1867 in Warsaw, Poland
where her father was professor of physics. At an early age, she displayed a brilliant mind and
a blithe personality. Her great exuberance for learning prompted her to continue with her
studies after high school. She became disgruntled, however, when she learned that the
university in Warsaw was closed to women. Determined to receive a higher education, she
defiantly left Poland and in 1891 entered the Sorbonne, a French university, where she earned
her Master's degree and Doctorate in physics. Marie was fortunate to have studied at the
Sorbonne with some of the greatest scientists of her day, one of whom was Pierre Curie,
Marie and Pierre married in 1895 and spent many productive years working together in the
physics laboratory. A short time after.they discovered radium, Pierre was killed by a horse-
drawn wagon in 1906. Marie was stunned by the horrible misfortune and endured
joy that
heartbreaking anguish. Despondently, ‘she recalled ‘their ‘close relationship’ and the
to fade when
they had shared in scientific research. Curie's feeling of desolation finally began
She was the first
she was asked to succeed her husband:as physics professor at the Sorbonne.
In 1914, she recetved the
woman to be given a professorship at the world-famous university.
Nobel Prize in the chemistry for isolating radium.
01. The Curies' collaboration helped to unlock the secrets of the atom.
B.competitive C. courteous ~ D. industrious
A. friendly
02. Marie had a bright mind and a___ personality.
B. humorous C. light-hearted D. strange
A. strong
the university in Warsaw, she felt
03. When she learned that she could not attend d
B. annoyed C. depressed D. worne
A. hopeless
enter the Sorbonne.
04. Marie by leaving Poland and traveling to France to
A. showed intelligence B. behaved ©. was distressed D. challenged authority
she remembered their joy together. 3
05. D. Dejectedly
A. Tearfully B. Happily C: Worriedly
preposition (Questions 6-8): .
Choose the appropriate
of interest,
06. This hak wo fe coportuliy to focus.__ your major area
B. of | C. about D.on —
A. in
07. He wants to stay a hotel tonight.
B. of C. at D. by
Ato
h et__‘__
tccrick
08. Did you wathe ‘TV last night? )
oe Boon” C. in D. to
A. by

629
2020
MBA (Evening) Program, DU Admissi on Test: Sprin.
Select the appropriate word (Questions 9 — 13):
09. Iwillbeinthe offices Sp.m. D. since

10. I will arrive sometime_____—._—:8 and 9 a.m. D. on


A. in B. next to C. between
11. Abed spent the entire afternoon the phone. D. by
Ain B. at C. on :
12. My best friend Mohammad is named___ his grandfather. D . of
A. after B. to C. about
13. All vehicles should stop the red light.
A.at B. in C. on paps

Choose the correct form of verb (Questions 14-15):

14. I to the commentary on an exciting cricket match on my way to work.


A. listening B. waslistening C.usedtolisten' _D.. listen
15. The watch was very expensive; I could not It.
A. buy B. bought C. buying D. to buy
Correct the underlined part of sentences (Questions 16-20):

16. Golf games often turn out to be more tiring than they
originally
seemed .
A. they orginally seemed
B. they seemingly would tire originally
C. it would have seemed originally
D. it originally seemed they would
17. She never visits any zoo because she is a strong opponent of the idea of setting animals
free into forest.
A. setting animals free into forest
B. watching the animals in their natural abode
C. going out of house on a holiday
D. holding the animals in captivity for our joy.
18. When studying an assignment, it is wise to read it over quickly at first, than see the
major points and finally outline the material.
A. first, than B. first, then C. first, though D. first-then
19. It is essential for an entrepreneur to keep his check upwards despite failures and setbacks.
A. to keep his chin upwards despite B. to keep his check up despite
C. to keep his chin up despite D, to keep his chin up despite of
20. In the cities, teenagers get more independence than
A. most suburbs : B. most suburbs do
C. they are in most suburbs D. they do in most suburbs

630
catia
rrent Affairs (Questions 21-
Which one of the following UN agenci
es recently became the first to make tra
21. nsactions
in crypto currency?
A. UNSECO ___B. UNICEF C. UNDP D. UNCHR
who developed ‘Bitcoin’?
92.
A. Satoshi Nakamoto B. Satoshi Furukawa C. Elon Mask D. Robert Elon
Which one of the following countries has announced to leave membership
23. of OPEC?
A. Ecuador B. Brazil C. Venezuela D. Chili
24. Proclamation of Bangladesh's independence is -
A. 3" schedule of the constitution B. 6th schedule of the constitution
C. 7" schedule of the constitution D. 8" schedule of the constitution
25. Which river of Bangladesh originates in Tibet?
A. Brahmaputra B. Padma C. Surma D. Jamuna
26. What is the time zone of Bangladesh?
A. UTC+5 B. UTC +6 C.UTIC+4 | D. UTC +3
21. Where is the headquarter of EU?
A. London B. Hague C. Brussels - D. Rome
28. Which country is the originator of the high speed Bullet Train?
A. USA 7 B. UK C. China D. Japan
29. The Champion for Skills Development for Young People award is conferred by -
A. ILO B.UNSECO C. UNHCR D. UNICEF
30. The Nobel Prize winner in Literature in 2019 is- ;
A. Dider Queloz B. Peter Handke C. Jim Peebles D. Abhijit Banerjee
Section II - Logical Reasoning/Critical Analysis and Mathematics (Questions 31-55).
31. If highways were restricted to cars and only those truck with capacity of less than $ tons,
most of the truck traffic would be forced to run outside highway. Such a reduction m the
amount of truck traffic would reduce the risk of collision on highways. The conclusion
drawn in the first sentence depends on which of the following assumptions?
A. Most trucks that are currently running on highways have a capacity of more than 8 tons.
B. The roads outside highways would be as convenient as highways for most truck drivers.
C. Most roads outside highways are not ready to handle truck traffic.
D. Cars are at greater risk of becoming involved in collisions than are trucks.
32. Our work proves to be very successful, In the past three years, each of our five clients
has experienced the fastest growth of sales in their history. Therefore, if your company
wants to increase sales, do not hesitate to call Sigma & Max, since we are the solution.
Which of the following, if true, most seriously jeopardizes the validity of the argument
by the speaker above? 4 :
A. Most of the consultants at Sigma & Max hold MBA degrees.
B. Even without the help of Sigma & Max, the five clients of Sigma & Max, will
achieve the same growth rate in sale.
C. Sigma & Max is one of the five leading management consulting companies.
D. Sigma & Max uses an updated accounting approach to help companies to cut cost.

631
a “Spring, 2020
MBA (Evening) Program, DU Admission Test mount of acreage
in ’ tural technology, the same @
agricul
Ba Today, because of improv ements
produces double the apples that it has in 1910.
A. double the apples that it has . yo times as ™
=» next?
C. as much as twice the apples ae
number should com
34. Look at the series: 7, 10, 8, 11,9,12,... What
ted in buil ding ar
s havi ng 9.8 floors. If a buil ing has more
uildding
—s y
35. Man busi ness offi ces ee
are loca
wing g must
h of the followin
a lift. If the above statements are truc, whic
than 3 floors, it has ae
,
be true? Oe
A. 2™ floors do not have lifts B. 7" floors have
C. Only floors above the 3 floors have lifts D. All floors may be reac

Mathematics (Questions 36-55)


36. If n—5 an even integer, what is the next larger consecutive even integer?
A.n-7 B.n-3 C.n-4 Dn+2
37. If (x° + |) =52, the value of (x + oP is
x x
A.4 B.-4 C.6 D. 10
38. + 3)’, then(- 2x - 6) is:
Ify =(x
A. 4y B. -2y C. 2y D. 27
39. Ifa man were to sell his chair for Tk. 720, he would lose 25%. To gain 25% he should
sell it for:
A. Tk. 1200 B.Tk. 1000 C. Tk. 960 D. Tk. 900
Rafi saw the flash of a lightning bolt and ‘heard the thunder 9 seconds later. If sound
travels at the speed of 350 meters per second, how far was Rafi standing from the place
the lightning struck?
A. 3500 B. 365 C. 3150 D. 4500
41. Which of the following is greater than = ?

A. 33 B. iB, CG. 3 D 13
50 1] 5 37
42. What is the arithmetic mean of 20 natural numbers 1, 2,3, ..... 209
A. 10 B, 11.5 C. 10.5 ~ pp
43. Simplify (a — c) - (2a + e- b) ifa=~1,b=2,c=~3
A. 12 B. 10 C.9
If the curve described by the equation y = x” + bx + ¢ cuts the x-axi Da
point does it cut the x-axis? at-4, at which other
A.-1 id » C.1 D4
45. Three times irst of three consecutive odd integers is 3 4; ;
third. Find the third integer. Gers 1s 3 times more than twice the
A.9 B. 11 C.13 at

632
46. The area of a circle is 225. The circumference of the circle is:
geen arte 7.30 C.50n D. 30n
2 A é' ia
47.

B C
AC = 17, AB=.15. The area of the right angle triangle
ABC is. . iy
- A.50 B. 34 C. 90 D. 60"
48. The angles at a point add up to: pe
A. 180° B. 90° bey ser | bs SOU” D. 270°
49. What is the object measured by Geometry
A. Map B. Graph _ C. Land D. Weight
50. The angles of a triangle are in the ratio 2 : 3 : 7. The measure of the smallest angle is:
A. 90° B. 60° C.45° D. 30°
51. The distance aroundacircle isthe; “°° © 0
A. Radius B. Diameter C. Chord — D. Circumference
52. What is the three times the square root of 144? "38
A. 12 B. 24 C. 36 D. 23
53. What is the sum of the first 5 prime numbers? a ae
A. 18 2 cise se De
54. Find the simple interest on Taka 5200 for 2 years at 6% per annum.
A. Tk. 350 BTR 450 © CC Tk. 524 D.Tk. 624
55. Ifa: b:c=3:4: 7, then the ratio (a+b +c): ¢ is equal to :
A.2:1 B.14:30 C72 D.1:2

56. Translation (Bangla to English). 57. Translation ena to Bangla)


58. Short Note 59. Précis 60. Essay

N.B. Writing
part seat ara |

EMBA# 80 |
Pr

(633
~~

FACULTY OF BUSINESS STUDIES


University of Dhaka

Executive MBA ‘Admission Test? Julv 2022

Duration : 90 minutes Total Marks - 80


MCQ__: 55 minutes MCQ Marks : 55 |
Written : 35 minutes _. Written Marks : 25 |

|
Instructions |

e Check the question paper and the answer sheet before the start
of the examination. You are advised to change the question
ble or
paper or the answer sheet should the printing be illegi
ained.
missing. Any complaint at a later stage shall not be entert

© Use of calculator of any kind is not permitted.

the circles.
© Use black ball point pen only to fill up

mark. For every


e Each question in the MCQ part carries one
ucted.
wrong answer to an MCQ, 0.25 marks shall be ded

EMBA# 81 641
bpm.
por
MCQ Part

English (Questions 1-20)

Read the following passage carefully and answer the questions that follow (Questions 1-5)
The economy of Bangladesh attained tremendous growth in the recent’ past. It is
undeniable that business enterprises and industries have played a significant rote in
achieving this growth. The other side of the coin is that business entities and industries
have been causing excessive environmental pollution while threatening the sustainability
of the emerging economic growth. It is reported that industrial waste has a substantial
impact on natural resources that lead to excessive air and: water pollution while
destroying aquatic and forest ecology. It is estimated that Bangladesh loses over $6.5
billion amounting to 3.4% of its Gross Domestic Product (GDP) dué to environmental
pollution every year. Due to such environmental pollution, Bangladesh is not only
incurring huge economic losses but also witnessing premature deaths amounting to 28%
of all deaths ia the country. It is true that companies contribute to society by employmg
people and producing products and services: However, we need to keep in mind that they
also make profits by exploiting natural resources: Since companies cause environmental
degradation, they cannot avoid the responsibility to mitigate and prevent such
degradation. In this regard, the corporate environmental responsibility becomes
significant. The corporate environmental responsibility encompasses legally or morally
binding responsibility of a company for its social and environmental performance. It
obligates the companies to look beyond the short-term profitability and to undertake
responsible business practices while considering the environmental impact of the
corporate actions. In this regard, the concept of environmental accountability needs to be
integrated into the relevant laws and policies. '

1. The major objective of the passage is to -


A. make the point that companies are behaving responsibly to minimize environmental
damage in Bangladesh.
B. describes the severity of environmental damage in Bangladesh and Suggests ways
to minimize the damage.
C. depict the contribution of companies towards the tremendous growth of Bangladesh.
|
|
D. explain the regulations to prevent the environmental damage in Bangladesh.

2. Which of the following titles best suits the passage above?


A. The rise of Bangladesh economy [ «@
B. Industrial pollution and global warming
C. Effective environmental regulations for sustainable growth in Bangladesh
D. The contribution of industries to the Bangladesh economy
_ toed e
a ail

642
4. According to the passage, companies in Bangladesh mostly-
"4. comply with environmental regulations.
n.
B. bears the costs to mitigate the environmental degradatio
g environmental damage.
C. do not behave responsibly for minimizin
D. focus on long run profitability.
4. According to the passage, all of the following are the outcomes of industrial
pollution except-
A. air pollution B. water pollution
C. premature death D. healthy food
5. Which of the following does the author suggest to mitigate the environmental
degradation?
A. The integration of environmental accountability to relevant laws and policies.
B. Imposition of additional corporate taxes.
C. Establishment of more industries that will contribute to GDP.
D. Encouraging import-substitute industries.
Fill in the blanks using appropriate Articles and Prepositions (Questions 6-10)
06. I am worried the examination.
A. in B. about C. on D. of

07. Rahima is afraid spiders. Jtey @


A. from B. in ! C. about _ .D.of

08. The teacher set some homework the end of the lesson. .
A. at B. in C. of D. about

09. Climate change is blame for ‘the majority of the heat waves bemg recorded
the planet. ei
A. at, in B. to, in C. to, around D. at, around
10. The Padma has the strongest current the water.
A. around B. at C. on D.m
Choose the correct underlined part of the sentences (Questions 11-15)
it wisely, leisure promotes health, efficiency and happiness.
Il. Using
A. Using it wisely B. If it is used wisely
C. Having used it wisely D. Because of its wise use
12. We cannot always convey ourselves in simple sentences.
A. cannot always told B. cannot always express
C. cannot always communicate D, cannot always say
-If you have told B. Had you been told
- Had you told D. If you would have told

643
— 2022
Executive MBA Admission Test, Jul ake employees
happy and
14. The first priority of any organization should be to ™
customer
satisfaction
naturally follow.
A. customer satisfaction naturally follow.
B. customer satisfaction will naturally follow.
C. customer satisfaction are naturally follow.
D. customer satisfaction to naturally follow. 4
evention as_m.
. ‘ an

15. The campaign is focusing on


and treatment. .
A. awareness and prevention as many as early detection
B. awareness and prevention as much as early detection
C. awareness and prevention as most as early detection
D. awareness and prevention as early detection
Choose the correct sentences (Questions 16-20)
16. A. The setting of a story effects the story’s plot.
B. The setting of a story affect the stories plot.
C. The seiting ofa story affects story’s plot.
D. The seiting of a story affects the story’s plot.
17. A. I heard my name was called.
B. I heard my name called.
C. I heard my name was being called.
D. Il heard my name is calling.
18. A. The greater the demand high the price.
B. The greater the demand higher the price.
C. The greater the demand the higher the price..,
D. The greater the demand so high the price.
19. A. Neither of those mattresses feel comfortable.
B. Neither of those mattresses are feeling comfo
rtable.
C. Neither of those mattresses feels comfortabl
e.
D. Neither of those mattresses have felt
comfortable.
20. A. When we eat lunch, the cell phone rang,
B. When we are eating lunch, the cell
phone rang.
C. As we were eating lunch, the cell phone rang,
D. As we eat lunch, the cell phone rang.
Current Affairs (Questions 21-30)
21. Justin Trudeau is the Prime Minister of whic
h coun f ot ‘ei
anti-vaccine protest? ae bi Se i a)
A. Italy B. Switzerland C. France D. Canada

644
likely to increase
ewer

22
valuation
en - of Taka is B, trade —".
C. tariff
oe

D. export
ion that regulates and supervises insurance companies
«

in Bangladesh j
23.
iM inistry of Finance + BSEC ge sou a
‘ch country will host the FIFA World Cup 2022?
94. Wink B. Qatar C.SouthKorea p. Australia
ich one of the following is the closest GDP per
capita in Bangladesh
25. Oe 300 B. $2300 C. $1700 D. $1600
onargaon in Narayanganj of Bangladesh is
declared as the ‘World Craft City’ for
26.
A. Maslin B. Jamdani C. Jute D. Handicraft
The ‘OBR’ initiative is launched by __.
27. a. Resin B. Japan C. China D. USA
The largest export destination of Bangladesh is?
28. A China
B. India C. USA D. UK
29. Which company constructed the Padma Bridge?
A. China Major Bridge Engineering Co. Ltd.
B. Sinohydro Corporation Ltd. C. Samsung C &T Corporation
D. Hyundai Engineering & Construction
30. Who is the first-ever female finance secretary of Bangladesh?
A. Sharifa Khan B. Fatima Yasmin
C. Sultana Afroz. . D.Easmin Akter
Logical Reasoning (Questions 31-35)
31. If the 5" date of the month comes 2 days fro m Tuesday, then which would be the day
on the day before 19™ of that moth? —
A. Monday B. Thursday C. Sunday D. Wednesday
32. Arif started from his home for the bus stop 15 minutes earlier then his usual time. He
took 10 minutes to reach the bus stop. He reached the bus stop at 08:40 hours. At
what time does he usually start for the bus stop?
A. 08: 40 hours B.08: 35 hours _C. 08: 45 hours D. 08: 55 hours
33. Find the word that suits the given set of words ~ Insect : Disease :: War:
A. Army B. Defeat _ C. Destruction _D.. Arsenal
IfS is taller than R, R is taller than V, and B is taller than both R and F, than which
of the following statements must be true?
A. S is taller than B B, R is shorter than F
C. § is taller than F D. B is taller than V

645
|
| Executive MBA Admission Test, July 2022
plus two does not
35. If one plus two equals three, than three plus two equals four jet
equal four. What can be concluded form the above aig 5 net equ ties
A. One plus two equals three. B. One plus two doe
C. Three plus two is greater than one plus two.
D. Three is greater than one.
Mathematic (Questions 36-55)
36. If (a+b) =— 1 and a” 24 + b’=
p= 2)25, then find the value of f(a (a — b)’ i
A. 16 B. 36 C.-12 D.
37. t(x) = 2x — 1 and p(x) = 1 - t(x)
The function of t and p are defined above. What is the value of p (0)?
A.0 B. -2 C.1 D. 2
38. A card is drawn from a well-shuffled pack of 52 cards. Find the probability of
getting a black card?

A. 3
l l 1 I
B. 4 C. 52 D. %6

39. If w is 10% less than x, and y is 30% less than z, then wy is what percent less than xz?
A. 39% B. 37% C.30% D: 25%
Simplify (2x!) +x 7-5
A. 4x° B. 4x C.4x7 D. 4x"
41. The ratio of boys to girls in a class is m : n. If the number of girls are x then the
number of boys are
x mx mn xn
A. ih C.rm D. —
m+n n x m-
42. Which of the following is equivalent to the pair. of inequalitiesx + 6 >10
andx—3<5?
A.245x<16 B.2<x<4 C.2<x<8 D.4<x<8
2246 4x-3
43. ifs = k4 “then the value ofx is:
A.9 B.6 C.3 D.}
When an item is sold for Tk. 18,700 the owner loses
15%. At what pri that
item be sold to get a gain of 15%? F se ae
A. Tk. 25,100 B, Tk.24,200 C. Tk. 25,300 D. Tk. 21,200
45. The average salary of all workers at a factory is Tk.8.0
workers is Tk. 12,000 and the a verage sala ry 000. The. average salary of 7
number of workers is 8 ry of the rest is Tk.6,000. The total
A. 19. Ba C23 Brie

646
umn” |

ifone-thitd of one-fourth of a number is 15, then three-ten


46.
A. 60 B. 54 C. 50 - ne number is
iday, 360 adults and children attended a show. The rat - |
47. show? ratio of adults to children
oo 1 How many children attended the

A certai n anima l in sj
in the zoo has consumed 39 pounds of fooditsCar .
48. will days. If as
same rate, in how many more days
continues to eat at the consumption
be 91 pounds?
A. 14 B. 42 C.7 D8

49. An item is sold at a loss of 12%. If the item can be sold by Tk. 1 more, then
there would have been a profit of 8%. Whatis the cost price of the oe
A. Tk 5,000 B. Tk 5,500 C. Tk. 6,000 D. Tk 6,500
50. A group of 202 people went on camping trip, taking 60 tents with them. Somie
of the
tents held 2 people each, and the rest held 4 people each. Assuming all the tents were
filled and every person got to sleep in a test, exactly how many of the tents were 2-
person tents?
A. 30 B. 20 C. 19 D. 18

oi. if Vx + J/9 = -/ 64, what is the value of x?


A. V5 B.5 C.25 D.55
32. A cricket team won 3 matches more than they lost. If a win gives them 2 pomts and
loss (-1) point, how many matches, in all, have they played if their score is 23? _
A. 37 B. 40 C.17 D. 20
53. The difference between the length and breadth of a rectangle is 23 m. If the permeter
of the rectangle is 206 m then its areais____.
2
A. 1520 m? B, 2420 m’ C.2480 m? D. 2520 m
54. What is the if its base and hypotenous measure 8 cm and 17 cm, respectively ?
A. 60 cm’ B. 70 cm? C. 65cm? D. 120 cm"
55. What is the area of a circle, whose diameter is 16 cm? j ;
A. 120m cm? B. 50n cm? C. 60cm’ DiGex cw

647
★★★Short
* *& * Short notes
notes for
for FBS
FBS admission★★★
admission** *

টা আসবে
55 DI BI

11 VAT:
VAT:

A
A value-added
value-added tax tax (VAT)
(VAT) isis aa consumption
consumption tax
tax placed
placed on on aa product
product whenever
whenever value
value is
is added
added atat
each
each stage
stage of
of the
the supply
supply chain,
chain, from production to
from production to the
the point
point of
of sale.
sale. The
The amount
amount ofof VAT
VAT that
that the
the
user pays
user pays is
is on
on the
the cost
cost of
of the
the product,
product, less
less any
any of
of the
the costs
costs of
of materials
materials used
used in
in the product that
the product that
have
have already been taxed.
already been taxed.

2) ATM
2) ATM MACHINE:
MACHINE:
An
An automated
automated teller
teller machine
machine (ATM)
(ATM) isis an
an electronic
electronic telecommunications
telecommunications device
device that
that enables
enables
customers
customers ofof financial
financial institutions
institutions to
to perform
perform financial
financial transactions,
transactions, such
such as
as cash
cash withdrawals,
withdrawals,
deposits, transfer funds, or obtaining account information, at any
deposits, transfer funds, or obtaining account information, at any time. time.
3)
3) Independence
Independence Day:
Day: aa holiday
holiday celebrating
celebrating the
the anniversary
anniversary of
of aa country’s
country’s independence
independence from
from
another
another country
country that
that ruled
ruled it
it in
in the
the past.
past.
4)
4) BANK
BANK RATE:
RATE:
A bank rate
A bank rate is
is the
the interest
interest rate
rate at
at which
which aa nation's
nation's central
central bank
bank lends
lends money
money to
to domestic
domestic banks.
banks.
5)
5) INTEREST
INTEREST RATE:
RATE:

The
The interest
interest rate
rate is
is the
the amount
amount charged,
charged, expressed
expressed as
as aa percentage
percentage of
of the principal, by
the principal, by aa lender
lender to
to
aa borrower for the use of assets.
borrower for the use of assets.
6)
6) Credit
Credit Transfer:
Transfer: A A way
way of
of sending
sending money
money directly
directly from
from one
one bank
bank account
account to
to another without
another without
using aa cheque
using cheque etc.
etc.
7)
7) Echo
Echo Tourism:
Tourism: The
The Definition:
Definition: Ecotourism
Ecotourism is:
is: "Responsible
"Responsible travel
travel to
to natural
natural areas
areas that
that
conserves the environment and improves the well-being of local people."
conserves the environment and improves the well-being of local people."
8)
8) Goal
Goal setting:
setting: The
The definition
definition of
of goal
goal setting
setting is
is the
the process
process of
of identifying
identifying something
something that
that you
you
want
want to
to accomplish
accomplish and and establishing
establishing measurable
measurable goals
goals and
and timeframes.
timeframes. When
When you
you decide
decide onon aa
financial
financial change
change to to save
save more
more money
money andand then
then set
set aa certain
certain amount
amount to
to save
save each
each month,
month, this
this is
is an
an
example
example ofof goal
goal setting.
setting.

9)
9) Bangladesh
Bangladesh Bank
Bank :: Bangladesh
Bangladesh Bank
Bank is
is the
the central
central bank
bank of
of Bangladesh
Bangladesh and
and is
is aa member
member of
of the
the
Asian
Asian Clearing
Clearing Union.
Union.
10)
10) Call
Call money:
money: Call
Call money
money is
is aa very
very short-term
short-term bank
bank loan
loan that
that does
does not
not contain
contain regular
regular
principal and
principal and interest payments. It
interest payments. It is
is often
often used
used by
by brokerage
brokerage firms
firms to
to finance
finance margin
margin accounts.
accounts.

11)
11) Inflation:
Inflation: Inflation
Inflation means
means that
that the
the general
general level
level of prices is
of prices is going
going up,
up, the
the opposite
opposite of
of
deflation.
deflation. More
More money
money will
will need
need to be paid
to be paid for
for goods
goods (like
(like aa loaf
loaf of bread) and
of bread) and services
12)
12) Black
Black money:
money: Black
Black money
money is
is income
income earned
earned secretly
secretly or
or illegally, usually in
illegally, usually in cash,
cash, and
and not
not
reported
reported to
to the
the government
government and
and taxes
taxes have
have not
not been paid.
been paid.
13)
13) Online
Online Banking:
Banking: Online
Online banking,
banking, also
also known
known asas internet banking, e-banking
internet banking, e-banking oror virtual
virtual
banking, is an electronic payment system that enables customers of a bank or other
banking, is an electronic payment system that enables customers of a bank or other financial financial
institution
institution to
to conduct
conduct aa range
range of
of financial
financial transactions
transactions through
through the
the financial
financial institution’s
institution’s website.
website.

14)
14) Private
Private sector
sector inin Bangladesh:
Bangladesh: The The Private
Private Sector
Sector is
is the
the part
part of
of the
the economy,
economy, sometimes
sometimes
referred
referred to
to as
as the
the citizen
citizen sector,
sector, which
which isis run
run by private individuals
by private individuals or
or groups, usually as
groups, usually as aa means
means
of enterprise for profit, and is not controlled by the State. (Areas of the economy
of enterprise for profit, and is not controlled by the State. (Areas of the economy controlled bycontrolled by
the
the state
state being referred to
being referred to as
as the
the Private
Private Sector
Sector
15)
15) Freedom:
Freedom: Freedom
Freedom is is the power or
the power or right
right to
to act,
act, speak,
speak, or
or think
think as
as one
one wants
wants without
without
hindrance
hindrance or
or restraint,
restraint, and
and the
the absence
absence of
of aa despotic
despotic government.
government.

16)
16) Democracy:The
Democracy:The definition
definition of
of aa democracy
democracy is
is aa form
form of
of government
government in
in which
which the
the people
people rule,
rule,
either
either directly
directly or
or through
through
elected
elected representatives. An example
representatives. An example of
of democracy
democracy is
is the
the type
type of
of government
government in
in the
the United
United States.
States.
17)
17) Rural
Rural Development
Development in
in Bangladesh:Rural
Bangladesh:Rural development
development is
is the
the process
process of
of improving
improving the
the quality
quality
of
of life
life and
and economic
economic well-being
well-being of people
of people

living
living in
in rural
rural areas,
areas,
18)
18) Honesty:
Honesty:

The
The definition
definition of
of honest
honest is
is someone
someone or
or something
something that
that is
is truthful,
truthful, trustworthy
trustworthy or
or genuine.
genuine.
19)
19)
Stock
Stock exchange:
exchange:
A
A stock
stock exchange
exchange is
is an
an exchange
exchange (or
(or bourse)
bourse) where
where stock
stock brokers
brokers and
and traders
traders can
can buy
buy and
and sell
sell
shares
shares of
of
stock,
stock, bonds,
bonds, and
and other
other securities
securities

20)
20) Technology:
Technology: The
The definition
definition of
of technology
technology is
is science
science or
or knowledge
knowledge put
put into practical use
into practical use to
to
solve
solve
problems or
problems or Invent
Invent useful
useful tools.
tools.

21)
21) Fiscal
Fiscal policy:
policy:

Fiscal
Fiscal policy
policy is
is the
the means
means by
by which
which aa government
government adjusts
adjusts its
its spending
spending levels
levels and
and tax
tax rates
rates to
to
monitor and
monitor and

influence
influence aa nation's
nation's economy.
economy.

22)
22) Monetary
Monetary policy:
It
It is
is how
how central
central banks
banks manage
manage liquidity
liquidity to
to create
create economic
economic growth.
growth.
23)
23) Culture:
Culture:

Culture
Culture is
is the
the characteristics
characteristics and
and knowledge
knowledge of
of aa particular
particular group
group of
of people,
people, encompassing
encompassing
language,
language,
religion,
religion, cuisine,
cuisine, social
social habits,
habits, music
music and
and arts
arts

24)
24) Automation:
Automation:

the
the technique,
technique, method,
method, or
or system
system of
of operating
operating or
or controlling
controlling aa process by highly
process by highly automatic
automatic
means, as by
means, as by
electronic
electronic devices,
devices, reducing
reducing human
human intervention
intervention to
to aa minimum.
minimum. 2.
2. A
A mechanical
mechanical device,
device, operated
operated
electronically, that functions
electronically, that functions automatically,
automatically, without
without continuous
continuous input
input from
from an
an operator.
operator. 3.
3. Act
Act or
or
process of
process of
automating.
automating.
25)
25) Green
Green financing:
financing:

Green
Green growth
growth is
is defined
defined as
as growth
growth generated
generated through
through the
the harmony
harmony between
between the
the economy
economy and
and
the
the environment
environment ........Green
........ Green finance
finance refers
refers to
to financial
financial support
support for
for green
green growth.
growth. •* Green
Green finance
finance
is
is

aa phenomenon
phenomenon that
that combines
combines the
the word
word of
of finance
finance and
and business
business with
with environmentally
environmentally friendly
friendly
behavior.
behavior.
26)
26) Globalization:
Globalization:
Globalization
Globalization is
is aa process
process of
of interaction
interaction and
and integration
integration among
among the people, companies,
the people, companies, and
and
governments
governments ofof

different
different nations,
nations, aa process
process driven
driven by
by international
international trade
trade and
and investment
investment and
and aided
aided by
by
information
information
technology.
technology.
27)
27) E-banking:
E-banking:
Online
Online banking,
banking, also
also known
known as
as internet
internet banking,
banking, e-banking
e-banking or
or virtual banking, is
virtual banking, is an
an electronic
electronic
payment
payment

system
system that
that enables
enables customers
customers of
of aa bank
bank or
or other
other financial
financial institution
institution to
to conduct
conduct aa range
range of
of
financial
financial

transactions
transactions through
through the
the financial
financial institution’s
institution’s website.
website.

28)
28) International
International Mother
Mother Language
Language Day:
International
International Mother
Mother Language
Language Day
Day (IMLD)
(IMLD) is
is aa worldwide
worldwide annual
annual observance
observance held
held on
on 21
21
February
February toto
promote awareness
promote awareness of
of linguistic
linguistic and
and cultural
cultural diversity
diversity and
and multilingualism.
multilingualism.

29)
29) Email:
Email:
email
email (or
(or e-mail)
e-mail) is
is defined
defined as
as the
the transmission
transmission of
of messages
messages over
over communications
communications networks
networks
Electronic
Electronic
mail
mail (or
(or e-mail
e-mail or
or email)
email) is
is an
an Internet
Internet service
service that
that allows
allows people
people who
who have
have an
an e-mail
e-mail address
address
(accounts) to
(accounts) to

send
send and
and receive
receive electronic
electronic letters.
letters. Several
Several formats
formats exist
exist for
for email
email addresses.
addresses.

30)
30) Digital
Digital Bangladesh:
Bangladesh:

"Digital
"Digital Bangladesh"
Bangladesh" means
means an
an ICT
ICT based
based modern
modern country
country where
where there
there will
will be
be adequate
adequate power
power
supply,
supply,
well-networked
well-networked infrastructure, use of
infrastructure, use of internet,
internet, use
use of
of open-source
open-source software,
software, e-banking,
e-banking,
ecommerce
ecommerce and
and

e-governance.
e-governance.
31)
31) Greenhouse
Greenhouse effect:
effect:
The
The greenhouse
greenhouse effect
effect is
is aa warming
warming of
of Earth's
Earth's surface
surface and
and the
the air
air above
above it.
it. It
It is
is caused
caused by
by gases
gases
in
in the
the

air
air that
that trap
trap energy
energy from
from the
the sun.
sun. These
These heat-trapping
heat-trapping gases
gases are
are called
called greenhouse
greenhouse gases.
gases. The
The
most
most

common
common greenhouse
greenhouse gases
gases are
are water
water vapor,
vapor, carbon
carbon dioxide,
dioxide, and
and methane.
methane.
32)
32) Metro
Metro Rail:
Rail:

Definition
Definition of
of Metrorail
Metrorail —– Any
Any of
of various
various (frequently
(frequently underground)
underground) citywide
citywide railway
railway systems.
systems.
33)
33) Padma
Padma Bridge:
Bridge:
The
The Padma
Padma Bridge
Bridge is
is aa multipurpose
multipurpose road-rail bridge across
road-rail bridge across the
the Padma
Padma River
River to
to be
be constructed
constructed in
in
Bangladesh.
Bangladesh. When
When completed
completed it
it will
will be
be the
the largest bridge in
largest bridge in Bangladesh
Bangladesh and
and the
the first
first fixed
fixed river
river
crossing
crossing

for
for road
road traffic.
traffic. It will connect
It will connect Louhajong,
Louhajong, Munshiganj
Munshiganj to
to Shariatpur
Shariatpur and
and Madaripur,
Madaripur, linking
linking the
the
south-west
south-west
of
of the
the country,
country, to
to northern
northern and
and eastern
eastern regions.
regions.

Main
Main Bridge
Bridge Length:
Length: 6.15
6.15 km
Construction
Construction end:
end: Expected
Expected in
in December
December 2018-Construction
2018-Construction start:
start: 77 December
December 2014
2014 Constructed
Constructed
by:
by:
China
China Major
Major Bridge
Bridge Engineering.
Engineering.

Viaduct:
Viaduct: 3.148
3.148 km
km (Road),
(Road), 532
532 m
m (Rail)
(Rail) &
& Approach
Approach Road:
Road: 12
12 km
km Total
Total
length:
length: 6,150
6,150 m
m (20,180
(20,180 ft)
ft)
34)
34) Bangladesh
Bangladesh aspiration
aspiration to
to be
be aa middle
middle income
income country:
country:
Middle-income
Middle-income countries
countries (MICs)
(MICs) are
are nations
nations with
with aa per-capita
per-capita gross
gross national
national income
income in
in 2012
2012
between
between
$1,036
$1,036 and
and $12,615.
$12,615. Middle-income
Middle-income countries
countries (MICs)
(MICs) are
are one
one of
of the
the income
income categories
categories that
that the
the
World
World

Bank uses to
Bank uses to classify
classify economies
economies for
for operational
operational and
and analytical
analytical purposes.
purposes.
35)
35) RMG
RMG in
in Bangladesh:
Bangladesh:
Bangladesh’s
Bangladesh’s export
export trade
trade is
is now
now dominated by the
dominated by the ready-made
ready-made garments
garments (RMG)
(RMG) industry.
industry. In
In
2012
2012
Bangladesh’s
Bangladesh’s garment
garment exports
exports —– mainly
mainly to
to the
the US
US and
and Europe
Europe —– made
made up
up nearly
nearly 80%
80% of
of the
the
country’s
country’s
export
export income.
income. …
... Subcontracting
Subcontracting is
is aa major
major component
component of
of the
the RMG
RMG industry
industry in
in Bangladesh.
Bangladesh.
36)
36) Global
Global Warming:
Warming:
Global
Global warming
warming is
is the process that
the process that causes
causes the
the Earth's
Earth's temperature
temperature to
to rise
rise and
and makes
makes the
the Earth
Earth
warmer.
warmer.
Humans play aa large
Humans play large role
role in
in the
the increase
increase in
in the
the Earth's
Earth's temperature.
temperature.

37)
37) Rural
Rural Development:
Development:
Rural
Rural development
development is
is the
the process
process of
of improving
improving the
the quality
quality of
of life
life and
and economic well-being of
economic well-being of
people
people
living
living in
in rural
rural areas,
areas, often
often relatively
relatively isolated
isolated and
and sparsely populated areas
sparsely populated areas .............................
..0....... ce eeeeeeeeeeees
Rural
Rural
development
development is
is aa comprehensive
comprehensive term.
term.
38)
38) Millennium
Millennium Development
Development Goal
Goal (MDG):
(MDG):

The
The Millennium
Millennium Development
Development Goals
Goals (MDGs)
(MDGs) are
are eight
eight goals
goals with
with measurable
measurable targets
targets and
and clear
clear
deadlines
deadlines

for
for improving
improving the
the lives
lives of
of the
the world's poorest people.
world's poorest people. To
To meet
meet these
these goals
goals and
and eradicate
eradicate poverty,
poverty,
leaders
of
of 189
189 countries
countries signed
signed the
the historic
historic millennium
millennium declaration
declaration at
at the
the United Nations Millennium
United Nations Millennium
Summit
Summit inin
2000.
2000.

39)
39) Public
Public Private
Private partnership
partnership (PPP):
(PPP):
Public-private
Public-private partnership
partnership (PPP)
(PPP) is
is aa funding
funding model
model for
for aa public
public infrastructure project such
infrastructure project such as
as aa
new
new
telecommunications
telecommunications system,
system, airport
airport or power plant.
or power plant. The public partner
The public partner is
is represented by the
represented by the
government
government

at
at aa local,
local, state
state and/or
and/or national
national level.
level. The
The private-sector partner assumes
private-sector partner assumes all
all risk.
risk.
40)
40) Rohingya
Rohingya

Rohingya
Rohingya definition:
definition: aa member
member of
of aa Muslim
Muslim people
people of
of W
W Myanmar
Myanmar
Leider
Leider believes
believes the
the Rohingya
Rohingya is
is aa political
political movement
movement that
that started
started in
in the
the 1950s
1950s to
to create
create "an
"an
autonomous
autonomous Muslim
Muslim zone"
zone" in
in Rakhine.
Rakhine. The
The government
government of
of Myanmar
Myanmar Prime
Prime Minister
Minister UU Nu,
Nu,
when
when

Myanmar
Myanmar was
was aa democracy
democracy from
from 1948-1962,
1948-1962, used
used the
the term
term "Rohingya".
"Rohingya".
Myanmar:
Myanmar: 500,000
500,000 (September
(September 2017)
2017)
Saudi
Saudi Arabia:
Arabia: 400,000
400,000 (2015)
(2015)
India:
India: 40,000
40,000 (September
(September 2017)
2017)
Bangladesh:
Bangladesh: 900,000+
900,000+ (September
(September 2017)
2017)
41)
41) EPZ:
EPZ:
The
The Bangladesh
Bangladesh Export
Export Processing
Processing Zone
Zone Authority
Authority is
is an
an agency
agency of
of the
the Government
Government of
of
Bangladesh and is
Bangladesh and is
administered
administered out
out of
of the
the Prime
Prime Minister's
Minister's Office.
Office. Its
Its objective
objective is
is to
to manage
manage the
the various
various export
export
processing
processing
zones
zones in
in Bangladesh.
Bangladesh.

42)
42) Money
Money laundering:
laundering:

Money
Money laundering
laundering is
is the process of
the process of transforming
transforming the
the profits
profits of
of crime
crime and
and corruption
corruption into
into
ostensibly
ostensibly

"legitimate"
"legitimate" assets.
assets.

43)
43) Fiscal
Fiscal Year:
A
A fiscal
fiscal year
year is
is aa period
period that
that aa company
company or
or government
government uses
uses for
for accounting purposes and
accounting purposes and
preparing
preparing
financial
financial statements
statements which
which vary
vary between
between countries.
countries. It
It is
is also
also used
used for
for financial
financial reporting
reporting by
by
business and
business and
other
other organizations.
organizations.
44)
44) Budget:
Budget:
A budget is
A budget is aa financial plan for
financial plan for aa defined period of
defined period of time,
time, usually
usually aa year.
year. It
It may
may also
also include
include
planned sales
planned sales

volumes
volumes and
and revenues,
revenues, resource
resource quantities,
quantities, costs
costs and
and expenses,
expenses, assets,
assets, liabilities
liabilities and
and cash
cash flows.
flows.

45)
45) Economic
Economic zone:
zone:

A
A special
special economic
economic zone
zone is
is an
an area
area in
in which business and
which business and trade
trade laws
laws are
are different
different from
from rest
rest of
of the
the
country.
country.

46)
46) Gross
Gross domestic product (GDP):
domestic product (GDP):
It
It is
is the
the monetary
monetary value
value of
of all
all the
the finished
finished goods
goods and
and services
services produced
produced within
within aa country's
country's
borders in
borders in a a
specific
specific time period.
time period.
47)
47) Gross
Gross national
national product
product (GNP):
(GNP):
The
The total
total market
market value
value of
of the
the final
final goods
goods and
and services
services produced by aa nation’s
produced by nation’s economy
economy during
during aa
specific
specific
period of
period of time
time (usually
(usually aa year).
year).

48)
48) Good
Good Governance:
Governance:
Good
Good Governance
Governance is
is an
an approach
approach to
to government
government that
that is
is committed
committed to
to making
making decisions which are
decisions which are
effective,
effective,

inclusive
inclusive and
and transparent.
transparent. It
It is
is aa commitment
commitment to
to doing
doing the
the best
best with
with the
the resources
resources available
available
within
within the
the
context
context of
of the
the challenges
challenges facing
facing governance.
governance.

49)
49) Mobile banking
Mobile banking

It
It is
is aa service provided by
service provided by aa bank
bank or
or other
other financial
financial institution
institution that
that allows
allows its
its customers
customers to
to
conduct
conduct

financial
financial transactions
transactions remotely
remotely using
using aa mobile
mobile device
device such
such as
as aa smart
smart phone
phone or
or tablet.
tablet.

50)
50) Remittance:
A
A remittance
remittance is
is aa transfer
transfer of
of money
money by
by aa foreign
foreign worker
worker to
to an
an individual
individual in
in his
his or
or her
her home
home
country.
country. Money
Money sent
sent home by migrants
home by migrants competes
competes with
with international
international aid
aid as
as one
one of
of the
the largest
largest

financial
financial inflows
inflows to
to developing
developing countries.
countries.
51)
51) Brexit:
Brexit:
Brexit
Brexit is
is the
the popular
popular term
term for
for the prospective withdrawal
the prospective withdrawal of
of the
the United
United Kingdom
Kingdom from
from the
the
European
European

Union.
Union. In
In aa referendum
referendum on
on 23
23 June
June 2016,
2016, 51.9%
51.9% of
of the participating UK
the participating UK electorate
electorate voted
voted to
to leave
leave
the
the
EU.
EU.

52)
52) Green
Green Banking:
Banking: ItIt is
is aa very
very general
general term
term which
which can
can cover
cover aa multitude
multitude of
of areas
areas from
from aa Bank
Bank
being environmentally
being environmentally friendly
friendly toto how
how their
their money
money is
is invested.
invested. Green
Green Banking
Banking considers
considers all
all the
the
social and environmental / ecological factors with an aim to protect the environment
social and environmental / ecological factors with an aim to protect the environment and and
conserve
conserve natural
natural resources.
resources.
50)
50) Micro
Micro Credit:
Credit: Microcredit
Microcredit isis the
the extension
extension of
of very
very small
small loans
loans to
to impoverished
impoverished borrowers
borrowers
who typically lack collateral, steady employment, or a verifiable credit history. It is designed
who typically lack collateral, steady employment, or a verifiable credit history. It is designed to
to
support
support entrepreneurship
entrepreneurship and
and alleviate
alleviate poverty.
poverty.
51)
51) L/C:
L/C: In
In modern
modern business
business practice,
practice, aa L/C
L/C also
also known
known asas aa Documentary
Documentary Credit
Credit is
is aa written
written
commitment
commitment by by aa bank
bank issued
issued after
after aa request
request by
by an
an importer
importer that payment will
that payment will be
be made
made to to the
the
beneficiary.
beneficiary.
53)
53) SDR:
SDR: Special
Special drawing
drawing rights
rights are
are supplementary
supplementary foreign-exchange
foreign-exchange reserve
reserve assets
assets defined
defined and
and
maintained
maintained byby the
the International
International Monetary
Monetary Fund.
Fund. The
The SDR
SDR is
is the unit of
the unit of account
account for for the
the IMF.
IMF. 52)
52)
Tax Holiday: A tax holiday is a temporary reduction or elimination of a tax. It
Tax Holiday: A tax holiday is a temporary reduction or elimination of a tax. It is synonymousis synonymous
with tax
with tax abatement,
abatement, tax
tax subsidy
subsidy oror tax reduction. Governments
tax reduction. usually create
Governments usually create tax
tax holidays
holidays as
as
incentives
incentives for
for business
business investment.
investment.
53)
53) Stagflation:
Stagflation:
A
A condition
condition ofof slow
slow economic
economic growth
growth and
and relatively
relatively high high unemployment
unemployment —– economic
economic stagnation
stagnation
—– accompanied
accompanied by by rising
rising prices,
prices, or
or inflation,
inflation, oror inflation
inflation and and aa decline
decline inin Gross
Gross Domestic
Domestic Product
Product
(GDP).
(GDP). 54)
54) Deflation:
Deflation: InIn economics,
economics, deflation
deflation is is aa decrease
decrease in in the
the general price level
general price level of
of goods
goods
and
and services.
services. Deflation
Deflation occurs
occurs when
when the
the inflation
inflation raterate falls
falls below
below 0%.0%. Inflation
Inflation reduces
reduces the
the real
real
value of money over time, but deflation increases
value of money over time, but deflation increases it. it.
55)
55) Blue
Blue chip:
chip: A
A blue
blue chip
chip is
is stock
stock in
in aa corporation
corporation with
with aa national
national reputation
reputation for
for quality,
quality,
reliability,
reliability, and
and the
the ability
ability to
to operate
operate profitably
profitably in
in good
good times
times and bad.
and bad.
54)
54) Blue
Blue economy:
economy: Blue
Blue economy
economy means
means the
the use
use of
of the
the sea
sea and
and its
its resources
resources for
for sustainable
sustainable
economic
economic development.
development. For
For others,
others, it
it simply
simply refers
refers to
to any
any economic
economic activity
activity in
in the
the maritime
maritime
sector, whether sustainable
sector, whether sustainable or
or not.
56)
56) General
General Agreement
Agreement onon Tariffs
Tariffs and
and Trade
Trade (GATT):
(GATT): General
General Agreement
Agreement on
on Tariffs
Tariffs and
and Trade
Trade
was
was aa legal
legal agreement between many
agreement between many countries,
countries, whose
whose overall purpose was
overall purpose was to promote
to promote
international
international trade
trade by
by reducing
reducing or
or eliminating
eliminating trade
trade barriers.
barriers.
57)
57) International
International Development
Development Association
Association (IDA):
(IDA): The
The International
International Development
Development Association
Association is
is
an
an international
international financial
financial institution
institution which
which offers
offers concessional
concessional loans
loans and
and grants
grants to
to the
the world's
world's
poorest developing
poorest developing countries.
countries.
58)
58) IFC:
IFC: The
The International
International Finance
Finance Corporation
Corporation (IFC)
(IFC) is
is an
an international
international financial
financial institution
institution that
that
offers investment, advisory, and asset-management services to the developing country.
offers investment, advisory, and asset-management services to the developing country.
59)
59) Excise
Excise duty: An excise
duty: An excise or
or excise
excise tax
tax is
is aa type
type of
of tax
tax charged
charged on
on goods produced within
goods produced within the.
the. It
It
is
is aa tax
tax on
on the production or
the production or sale
sale of
of aa good.
good. This
This tax
tax is
is now
now known
known as
as the
the Central
Central Value
Value Added
Added
Tax.
Tax.

60)
60) Liquidity:
Liquidity: Liquidity
Liquidity means
means how
how quickly
quickly you
you can
can get
get your
your hands
hands on
on your
your cash
cash
61)
61) Profitability:
Profitability: Profitability
Profitability is
is ability
ability of
of aa company
company to to use
use its
its resources
resources to
to generate
generate revenues
revenues in
in
excess
excess of
of its
its expenses.
expenses. 62)
62) Bail
Bail Out:
Out: AA bailout
bailout is
is the
the act
act of
of aa business,
business, anan individual,
individual, or
or
government providing money and resources (also known as a capital injection)
government providing money and resources (also known as a capital injection) to a failing to a failing
company.
Short Note
Short Note and
and short
short Essay
Essay Topics:
Topics:

1) Technology
1) Technology ::
The definition
The definition of
of technology
technology is
is science
science or
or knowledge
knowledge put
put into
into practical use to
practical use to solve
solve problems or
problems or
invent useful tools.
invent useful tools.
2) Digital
2) Digital Bangladesh:
Bangladesh:
"Digital Bangladesh"
"Digital Bangladesh" means
means an
an ICT
ICT based
based modern
modern country
country where
where there
there will
will be adequate power
be adequate power
supply, well-networked
supply, well-networked infrastructure,
infrastructure, use
use of
of internet,
internet, use
use of
of open-source
open-source software,
software, e-banking,
e-banking, e-
e-
commerce and
commerce and e-governance.
e-governance.
3) Freedom
3) Freedom ::
Freedom is
Freedom is the
the power
power or or right
right to
to act,
act, speak,
speak, or
or think
think as
as one
one wants
wants without
without hindrance
hindrance or
or restraint,
restraint,
and the
and the absence
absence ofof aa despotic
despotic government.
government.

4) Democracy:
4) Democracy:
The definition
The definition of
of aa democracy
democracy isis aa form
form of
of government
government in
in which
which the
the people
people rule,
rule, either
either directly
directly
or through
or through elected
elected representatives.
representatives. AnAn example
example of
of democracy
democracy isis the
the type
type of
of government
government in in the
the
United States.
United States.

5) Goal
5) Goal setting
setting ::
The definition
The definition of
of goal
goal setting
setting is
is the
the process
process of
of identifying
identifying something
something that
that you
you want
want to
to accomplish
accomplish
and establishing
and establishing measurable
measurable goals
goals and
and timeframes.
timeframes. When you decide on a financial change to
save more money and then set a certain amount to save each month, this is an example of goal
setting.
6) Stock
6) Stock exchange
exchange ::
A stock exchange is
A stock exchange is an
an exchange
exchange (or
(or bourse)
bourse) where
where stock
stock brokers
brokers and
and traders
traders can
can buy
buy and
and sell
sell
shares of stock, bonds, and other securities
shares of stock, bonds, and other securities

7) Bangladesh
7) Bangladesh Bank
Bank ::
Bangladesh Bank
Bangladesh Bank is
is the
the central
central bank
bank of
of Bangladesh
Bangladesh and
and is
is aa member
member of
of the
the Asian Clearing
Asian Clearing
Union.
Union.
8) Independence
8) Independence Day Day ::
aa holiday
holiday celebrating
celebrating the
the anniversary
anniversary of
of aa country’s
country’s independence
independence from
from another
another country
country that
that
ruled it
ruled it in
in the
the past
past

9) Credit
9) Credit Transfer
Transfer ::
AA way
way of
of sending
sending money
money directly
directly from
from one
one bank
bank account
account to
to another
another without
without using
using aa cheque
cheque etc.
etc.

10) Echo
10) Echo Tourism
Tourism ::
The Definition:
The Definition: Ecotourism
Ecotourism is:
is: "Responsible
"Responsible travel
travel to
to natural
natural areas
areas that
that conserves
conserves the
the
environment and
environment and improves
improves the
the well-being
well-being of
of local
local people."
people."

Page
Page 1 of 4
1of
11) Private
11) Private sector
sector inin Bangladesh
Bangladesh ::
The Private
The Private Sector
Sector isis the
the part
part of
of the
the economy,
economy, sometimes referred to as the citizen sector, which
which
is run
is run by
by private
private individuals
individuals oror groups,
groups, usually
usually as
as aa means
means of
of enterprise
enterprise for
for profit, and is
profit, and is not
not
controlled by
controlled by the
the State.
State. (Areas of the economy controlled by the state being referred to as the
Private Sector
12) Rural
12) Rural Development
Development in in Bangladesh
Bangladesh ::
Rural development
Rural development is is the
the process
process of
of improving
improving the
the quality
quality of
of life
life and
and economic
economic well-being
well-being of
of
people living in
people living in rural
rural areas,
areas,

13) Honesty
13) Honesty ::
The definition
The definition of
of honest
honest is
is someone
someone or
or something
something that
that is
is truthful,
truthful, trustworthy
trustworthy or
or genuine.
genuine.

14) Culture
14) Culture ::
Culture is
Culture is the
the characteristics
characteristics and
and knowledge
knowledge of of aa particular
particular group
group of
of people, encompassing
people, encompassing
language, religion,
language, religion, cuisine,
cuisine, social
social habits,
habits, music
music andand arts
arts

15) Automation
15) Automation ::
the technique,
the technique, method,
method, oror system
system of
of operating
operating or
or controlling
controlling aa process by highly
process by highly automatic
automatic
means, as by electronic devices, reducing human intervention to a minimum. 2.
means, as by electronic devices, reducing human intervention to a minimum. 2. A mechanical
device, operated electronically, that functions automatically, without continuous input from an
operator. 3. Act or process of automating.

16) Green
16) Green financing
financing ::
Green growth is defined as growth generated through the harmony between the economy and
Green finance
the environment. ... Green finance refers to financial
refers to financial support
support for
for green
green growth.
growth. •* Green
Green finance
finance is
is
aa phenomenon
phenomenon that
that combines
combines the
the word
word. of
of finance
finance and
and business
business with
with environmentally
environmentally friendly
friendly
behavior.
behavior.
17) Call
17) Call money
money ::
Call money
Call money isis aa_very short-term bank
very short-term bank loan
loan that
that does
does not
not contain
contain regular
regular principal and interest
principal and interest
payments.
payments. ItIt is
is often
often used
used by brokerage firms
by brokerage firms to
to finance
finance margin
margin accounts.
accounts.

18) Globalization
18) Globalization ::
Globalization is
Globalization is aa process
process of
of interaction
interaction and
and integration
integration among
among thethe people, companies, and
people, companies, and
governments of
governments of different
different nations,
nations, aa process
process driven
driven by
by international
international trade
trade and
and investment
investment and
and
aided by
aided by information
information technology.
technology.
19) E-banking
19) E-banking ::
Online banking,
Online banking, also
also known
known asas internet
internet banking,
banking, e-banking
e-banking or or virtual
virtual banking, is an
banking, is an electronic
electronic
payment system that enables customers of a bank or other financial institution to conduct aa range
payment system that enables customers of a bank or other financial institution to conduct range
of financial transactions through the financial institution’s website.
of financial transactions through the financial institution’s website.

Page
Page 2
2 of
of 4
20) International
20) International Mother
Mother Language
Language DayDay ::
International Mother
International Mother Language
Language Day
Day (IMLD)
(IMLD) is is aa worldwide
worldwide annual
annual observance
observance held
held on
on 21
21
February to
February to promote
promote awareness
awareness of
of linguistic
linguistic and
and cultural
cultural diversity
diversity and
and multilingualism.
multilingualism.
21) Email
21) Email ::
email (or
email (or e-mail)
e-mail) is
is defined
defined asas the
the transmission
transmission of of messages
messages over
over communications
communications networks
networks
Electronic mail (or e-mail or email) is an Internet service that allows people who have an e-mail
address (accounts) to send and receive electronic letters. ... Several formats exist for email
addresses.
22) Inflation
22) Inflation ::
Inflation means
Inflation means that
that the
the general
general level
level of
of prices
prices isis going
going up,
up, the
the opposite
opposite of
of deflation.
deflation. More
More
money will
money will need
need to
to be
be paid
paid for
for goods
goods (like
(like aa loaf
loaf of
of bread) and services
bread) and services

23) Black
23) Black money
money ::
Black money
Black money isis income
income earned
earned secretly
secretly oror illegally,
illegally, usually
usually in
in cash,
cash, and
and not
not reported
reported to
to the
the
government and
government and taxes
taxes have
have not
not been
been paid.
paid.
24) Online
24) Online Banking
Banking : :
Online banking,
Online banking, also
also known
known asas internet
internet banking,
banking, e-banking
e-banking oror virtual
virtual banking,
banking, isis an
an electronic
electronic
payment system that
payment system that enables
enables customers
customers ofof aa bank
bank oror other
other financial
financial institution
institution to
to conduct
conduct aa range
range
of financial
of financial transactions
transactions through
through the
the financial
financial institution’s
institution’s website.
website.

25) Greenhouse
25) Greenhouse effecteffect ::
The greenhouse
The greenhouse effect
effect is
is aa warming
warming ofof Earth's
Earth's surface
surface and
and the
the air
air above
above it.
it. It
It is
is caused
caused by
by gases
gases
in the
in the air
air that
that trap
trap energy
energy from
from the
the sun.
sun. These
These heat-trapping
heat-trapping gases
gases are
are called
called greenhouse
greenhouse gases.
gases.
The most common greenhouse gases are water vapor, carbon dioxide, and methane.
The most common greenhouse gases are water vapor, carbon dioxide, and methane.
26) Metro
26) Metro Rail
Rail ::
Definition of Metrorail
Definition of Metrorail —– Any of various
Any of various (frequently
(frequently underground)
underground) citywide
citywide railway
railway systems.
systems.

27) Padma
27) Padma Bridge
Bridge ::
The Padma
The Padma Bridge
Bridge is is aa multipurpose
multipurpose road-rail
road-rail bridge
bridge across
across the
the Padma
Padma River
River to
to be
be constructed
constructed inin
Bangladesh. When
Bangladesh. When completed
completed it it will
will be the largest
be the largest bridge
bridge inin Bangladesh
Bangladesh and and the
the first
first fixed
fixed river
river
crossing for
crossing for road
road traffic.
traffic. ItIt will
will connect
connect Louhajong,
Louhajong, Munshiganj
Munshiganj to to Shariatpur
Shariatpur and
and Madaripur,
Madaripur,
linking the
linking the south-west
south-west of of the
the country,
country, to to northern
northern and
and eastern
eastern regions.
regions.
Main Bridge
Main Bridge Length:
Length: 6.15 6.15 kmkm
Viaduct: 3.148
Viaduct: 3.148 km km (Road),
(Road), 532 532 m m (Rail)
(Rail)
Approach Road:
Approach Road: 12 12 kmkm
Total length
Total length: : 6,150
6,150 m m (20,180
(20,180 ft)ft)
Construction end
Construction end: : Expected
Expected in in December
December 2018 2018
Construction start
Construction start: : 7
7 December
December 2014 2014
Constructed by
Constructed by: : China
China Major
Major Bridge
Bridge Engineering...
Engineering...

Page
Page 3
3 of
of 4
28) Bangladesh
28) Bangladesh aspiration
aspiration to
to be
be aa middle
middle income
income country
country ::
Middle-income countries
Middle-income countries (MICs)
(MICs) are
are nations
nations with
with aa per-capita
per-capita gross
gross national
national income
income in
in 2012
2012
between $1,036
between $1,036 and
and $12,615.
$12,615. Middle-income
Middle-income countries
countries (MICs)
(MICs) are
are one
one of
of the
the income
income categories
categories
that the
that the World
World Bank
Bank uses
uses to
to classify
classify economies
economies for
for operational
operational and
and analytical
analytical purposes.
purposes.
29) RMG
29) RMG in in Bangladesh
Bangladesh ::
Bangladesh’s export
Bangladesh’s export trade
trade isis now
now dominated
dominated byby the
the ready-made
ready-made garments
garments (RMG)
(RMG) industry.
industry. In
In
2012 Bangladesh’s
2012 Bangladesh’s garment
garment exports
exports —– mainly
mainly to
to the
the US
US and
and Europe
Europe —– made
made up
up nearly
nearly 80%
80% of
of the
the
country’s export
country’s export income.
income. … ... Subcontracting
Subcontracting isis aa major
major component
component of
of the
the RMG
RMG industry
industry in
in
Bangladesh.
Bangladesh.
30) Global
30) Global Warming
Warming ::
Global warming
Global warming is
is the
the process
process that
that causes
causes the
the Earth's
Earth's temperature
temperature to
to rise
rise and
and makes
makes the
the Earth
Earth
warmer. Humans
warmer. Humans play
play a a large
large role
role in
in the
the increase
increase in
in the
the Earth's
Earth's temperature.
temperature.

31) Rural
31) Rural Development
Development ::
Rural development
Rural development is is the
the process
process ofof improving
improving the
the quality
quality of
of life
life and
and economic
economic well-being
well-being of
of
people living in
people living in rural
rural areas,
areas, often
often relatively
relatively isolated
isolated.and sparsely populated
and sparsely areas. ...
populated areas. ... Rural
Rural
development is
development is aa comprehensive
comprehensive term. term.
32) Millennium
32) Millennium Development
Development Goal Goal (MDG):
(MDG):
The Millennium
The Millennium Development
Development Goals Goals (MDGs)
(MDGs) are are eight
eight goals
goals with
with measurable
measurable targets
targets and
and clear
clear
deadlines for
deadlines for improving
improving the
the lives
lives of
of the
the world's
world's poorest
poorest people. To meet
people. To meet these
these goals
goals and
and
eradicate poverty,
eradicate poverty, leaders
leaders of
of 189
189 countries
countries signed
signed the
the historic
historic millennium
millennium declaration
declaration atat the
the
United
United Nations Millennium Summit
Nations Millennium Summit in in 2000.
2000.

33) Public
33) Public Private
Private partnership
partnership (PPP)
(PPP)
Public-private partnership (PPP) is
Public-private partnership (PPP) is aa funding
funding model
model forfor aa public
public infrastructure
infrastructure project
project such
such as as aa
new telecommunications
new telecommunications system,
system, airport
airport or
or power plant. The
power plant. The public
public partner
partner is
is represented
represented by by the
the
government at a local, state and/or national level. ... The private-sector partner assumes all risk.
government at a local, state and/or national level. ... The private-sector partner assumes all risk.
34) Rohingya
34) Rohingya
Rohingya definition: aa member
Rohingya definition: member of of aa Muslim
Muslim people
people of
of WW Myanmar
Myanmar
Leider believes
Leider believes the
the Rohingya
Rohingya is is aa political
political movement
movement that
that started
started in
in the
the 1950s
1950s to
to create
create "an
"an
autonomous Muslim
autonomous Muslim zone"
zone" in
in Rakhine.
Rakhine. The The government
government of of Myanmar
Myanmar Prime
Prime Minister
Minister UU Nu,
Nu,
when Myanmar was a democracy from 1948-1962, used the term "Rohingya".
when Myanmar was a democracy from 1948-1962, used the term "Rohingya".
Myanmar: : 500,000
Myanmar 500,000 (September
(September 2017)
2017)
India: 40,000 (September 2017)
India : 40,000 (September 2017)
Bangladesh: : 900,000+
Bangladesh 900,000+ (September
(September 2017)
2017)
Saudi Arabia: 400,000 (2015)
Saudi Arabia : 400,000 (2015)

Page
Page 4 of 4
4of
Short Note
Short and short
Note and short Essay
Essay Topics:
Topics:
1) Technology
1) Technology
2) Digital
2) Digital Bangladesh
Bangladesh
3) Freedom
3) Freedom
4) Democracy
4) Democracy
5) Goal setting
5) Goal setting
6) Stock
6) Stock exchange
exchange
7) Bangladesh Bank
7) Bangladesh Bank
8) Independence
8) Independence Day Day
9) Credit
9) Credit Transfer
Transfer
10) Echo
10) Echo Tourism
Tourism
11) Private
11) Private sector
sector in
in Bangladesh
Bangladesh
12) Rural
12) Rural Development
Development in in Bangladesh
Bangladesh
13) Honesty
13) Honesty
14) Culture
14) Culture
15) Automation
15) Automation
16) Green
16) Green financing
financing
17) Call
17) Call money
money
18) Globalization
18) Globalization
19) E-banking
19) E-banking
20) International
20) International Mother
Mother Language
Language Day Day
21) Email
21) Email
22) Inflation
22) Inflation
23) Black
23) Black money
money
24) Online
24) Online Banking
Banking
25) Greenhouse effect
25) Greenhouse effect
26) Metro
26) Metro RailRail
27) Padma
27) Padma Bridge
Bridge
28) Bangladesh
28) Bangladesh aspiration
aspiration toto be
be aa middle
middle income
income country
country
29) RMG in Bangladesh
29) RMG in Bangladesh
30) Global
30) Global Warming
Warming
31) Rural
31) Rural Development
Development
32) Millennium
32) Millennium Development
Development Goal Goal (MDG)
(MDG)
33) Public
33) Public Private
Private partnership
partnership (PPP)
(PPP)
34) Rohingya
34) Rohingya
35) Infrustucture
35) Infrustucture

Technology refers
Technology refers to
to methods,
methods, systems,
systems, and
and devices
devices which
which are
are the
the result
result of
of scientific
scientific knowledge
knowledge
being used
being for practical
used for practical purposes.
purposes.

Freedom
Freedom
The power
The power oror right
right to
to act,
act, speak,
speak, or
or think
think as
as one
one wants.
wants.
Absence of
Absence of subjection
subjection toto foreign
foreign domination
domination or or despotic
despotic government.\
government. \
The power
The power ofof self-determination
self-determination attributed
attributed to
to the
the will;
will; the
the quality
quality of
of being
being independent
independent of
of fate
fate or
or
necessity.
Stock Exchange
Stock Exchange
A stock
A stock exchange
exchange is
is an
an exchange
exchange where
where stock
stock brokers and traders
brokers and traders can
can buy and sell
buy and sell shares
shares of
of
stock, bonds,
stock, bonds, and
and other
other securities.
securities.

•¢ ICT
ICT inin Banking
Banking SectorSector
•¢ Agent
Agent Banking
Banking
•* Mobile
Mobile Banking
Banking
•* Digital
Digital Banking
Banking
•* Black
Black Money
Money
•* Call
Call Money/ / Call Call Money
Money Rate.
Rate.
•¢ Liquidity.
Liquidity.
•¢ Capital
Capital Market.
Market.
•¢ Money
Money Market.
Market.
•* Bank
Bank Rate.
Rate.
•° EPZ.
EPZ.
•* ATM
ATM
•* Debit
Debit Card.
Card.
•* Credit
Credit Card.
Card.
•* Inflation.
Inflation.
•* Deflation.
Deflation.
•* Devaluation
Devaluation of of Currency.
Currency.
•* Budget
Budget 2017-2018.
2017-2018.
•* GDP
GDP & & GNP.
GNP.
•* Letter
Letter of of Credit
Credit CardCard (L/C).
(L/C).
•* Money Laundering.
Money Laundering.
•* DSE
DSE & & CSE.
CSE.
•° IPO.
IPO.
•¢ Special
Special Economic
Economic Zones. Zones.
•* New
New VATVAT Act / VAT Act
Act / VAT Act 2012.
2012.
•* Cyber
Cyber Crime
Crime in in Social Network/
Social Network/
Cyber Security
Cyber Security in in Bangladesh.
Bangladesh.
•¢ Bank
Bank Loan
Loan Defaulters.
Defaulters.
•¢ Micro
Micro Credit.
Credit.
•¢ Monetary
Monetary Policy.
Policy.
•* Fiscal
Fiscal Policy.
Policy.
•* Remittance.
Remittance.
•¢ SDR.
SDR.
•¢ Share
Share // Stock
Stock market.
market.
•¢ Green
Green Banking.
Banking.
•¢ Blue
Blue Economy.
Economy.
•¢ WTO
WTO, , GATT GATT ,, IMF IMF, , IDA
IDA & & IFC.
IFC.
•¢ SME
SME, , FDI FDI, , TIN
TIN ,, Excise
Excise Duty
Duty & & ECNEC.
ECNEC.
•* Blue
Blue Chip.
Chip.
•* Agro
Agro Based
Based Industry.
Industry.
•* Economic
Economic Grouth.
•¢ Economic
Economic Depression.
Depression.
•* Fiscal
Fiscal Year.
Year.
•¢ Tax
Tax Holiday.
Holiday.
•* Mortgage
Mortgage Market.
Market.
•¢ Stagflation.
Stagflation.
•* Monopoly
Monopoly ,, Oligopoly.
Oligopoly.
•* Clearing
Clearing House.
House.
•* Cartel
Cartel && Treasury
Treasury Bill.
Bill.

DNA
DNA
Deoxyribonucleic acid,
Deoxyribonucleic acid, aa self-replicating
self-replicating material
material which
which isis present
present in
in nearly
nearly all
all living
living organisms
organisms
as the
as the main
main constituent
constituent of of chromosomes.
chromosomes. It It is
is the
the carrier
carrier of
of genetic
genetic information.
information.
e The fundamental
The fundamental andand distinctive
distinctive characteristics
characteristics oror qualities
qualities of
of someone
someone oror something,
something, especially
especially
when regarded as
when regarded as unchangeable.
unchangeable.

Mobile banking
Mobile banking
Mobile banking
Mobile banking isis a
a service
service provided
provided by
by aa bank
bank or
or other
other financial
financial institution
institution that
that allows
allows its
its customers
customers to
to
conduct financial
conduct financial transactions
transactions remotely
remotely using
using aa mobile
mobile device
device such
such as
as aa smartphone
smartphone oror tablet.
tablet.

Agent Banking means


Agent Banking means providing
providing limited
limited scale
scale banking
banking and
and financial
financial services
services to
to the
the
underserved population
underserved population through
through engaged
engaged agents
agents under
under aa valid
valid agency
agency agreement,
agreement,
rather than
rather than a
a teller/
teller/ cashier.
cashier. It
It is
is the
the owner
owner of
of an
an outlet
outlet who
who conducts
conducts banking
banking
transactions on
transactions on behalf
behalf of
ofaa bank.
bank.

Export processing
Export processing zones
zones (EPZs)
(EPZs) are
are areas
areas within developing countries
within developing countries that
that offer
offer
incentives and a barrier-free environment to promote economic growth by attracting
incentives and a barrier-free environment to promote economic growth by attracting
foreign investment
foreign investment for
for export-oriented
export-oriented production.
production.

What is
What is an
an 'Automated
'Automated Teller
Teller Machine
Machine -- ATM?'
ATM?'

An automated teller
An automated teller machine
machine (ATM)
(ATM) is is an
an electronic
electronic banking
banking outlet,
outlet, which
which allows
allows
customers to
customers to complete
complete basic
basic transactions
transactions without
without thethe aid
aid of
of aa branch
branch representative
representative or or
teller. Anyone
teller. Anyone withwith aa credit
credit card
card or
or debit
debit card
card can
can access
access most
most ATMs.
ATMs. TheThe first
first ATM
ATM
appeared in
appeared in London
London in in 1967,
1967, and
and in
in less
less than
than 50 50 years,
years, ATMs
ATMs spreadspread around
around thethe globe,
globe,
securing a
securing a presence
presence in in every
every major
major country
country andand even
even tiny
tiny little
little island
island nations
nations such
such asas
Kiribati and
Kiribati and thethe Federated
Federated States
States ofof Micronesia.
Micronesia.

Gross domestic
Gross domestic product
product
Gross domestic
Gross domestic product
product isis a
a monetary
monetary measure
measure of
of the
the market
market value
value of
of all
all final
final goods
goods and
and services
services
produced in
produced in a
a period
period of
of time.
time.

Gross national
Gross national product
Gross national
Gross national product
product is
is the
the market
market value
value of
of all
all the
the products
products and
and services
services produced
produced in
in one
one year
year by
by labor
labor
and property
and property supplied
supplied by
by the
the citizens
citizens of
of a
a country.
country.

Letter of
Letter of credit
credit
In modern
In modern business
business practice,
practice, a a letter
letter of
of credit
credit (LC)
(LC) also
also known
known asas a
a Documentary
Documentary Credit,
Credit, is
is a
a
written commitment
written commitment by by aa bank issued after
bank issued after aa request
request by
by an
an importer
importer (foreign
(foreign buyer)
buyer) that
that payment
payment will
will
be made
be made toto the
the beneficiary
beneficiary (exporter)
(exporter) provided
provided that
that the
the terms
terms and
and conditions
conditions stated
stated in
in the
the LC
LC been
been
met, as
met, as evidenced
evidenced byby the
the presentation
presentation of of specified
specified documents.
documents.

Initial public
Initial public offering
offering
Initial public
Initial public offering
offering or
or stock
stock market
market launch
launch is
is a
a type
type of
of public
public offering
offering in
in which
which shares
shares of
of a
a company
company
usually are sold to institutional investors that in turn, sell to the general public, on a securities exchange,
usually are sold to institutional investors that in turn, sell to the general public, on a securities exchange,
for the first
for the time.
first time.

Cybercrime
Cybercrime
Cybercrime, or
Cybercrime, or computer
computer related
related crime,
crime, is
is crime
crime that
that involves
involves a a computer
computer and and a
a network.
network. The
The computer
computer
may have
may have been
been used
used in
in the
the commission
commission ofof a
a crime,
crime, or
or itit may
may be
be the
the target.
target.

Freelancer
Freelancer
AA freelancer or freelance
freelancer or freelance worker is a
worker is a term
term commonly
commonly used
used for
for a
a person
person who
who is
is self-employed
self-employed and
and is
is not
not
necessarily committed
necessarily committed toto a
a particular
particular employer
employer long-term.
long-term.

Fiscal policy
In economics
In economics and
and political
political science,
science, fiscal
fiscal policy
policy is
is the
the use
use of
of government
government revenue
revenue collection
collection and
and
expenditure to
expenditure to influence
influence the the economy.
economy.

AA fiscal
fiscal year
year (FY)
(FY) is
is a
a period
period that
that a
a company
company or or government
government uses
uses for
for accounting
accounting
purposes and
purposes and preparing
preparing financial
financial statements.
statements. A A fiscal
fiscal year
year may
may not
not bebe the
the same
same as
as a
a
calendar year,
calendar year, and
and for tax purposes,
for tax purposes, the
the Internal
Internal Revenue
Revenue Service
Service (IRS)
(IRS) allows
allows
companies to
companies to be
be either
either calendar-year
calendar-year taxpayers
taxpayers oror fiscal-year
fiscal-year taxpayers.
taxpayers.

Outsourcing
Outsourcing
In business,
In business, outsourcing
outsourcing isis an
an agreement
agreement in in which
which one
one company
company contracts
contracts its
its own
own
internal activity
internal activity to
to a
a different
different company.
company. ItIt involves
involves the
the contracting
contracting out
out of
of a
a business
business
process and
process and operational,
operational, and/or
and/or non-core
non-core functions
functions to
to another
another party

You might also like